Sie sind auf Seite 1von 76

BAR EXAMINATION

QUESTIONS AND ANSWERS


(1980-2017)

CIVIL LAW

CREDIT TRANSACTIONS
AND
TORTS AND DAMAGES

Submitted to:
Atty. Stephanie Rachel Castro

Submitted by:

4A
1
Page

CIVIL LAW REVIEW 2 – 4A BAR EXAMINATION Q & A (1980-2017)


CREDIT TRANSACTIONS

Kevin signed a loan agreement with ABC Bank. To secure payment, Kevin
requested his girlfriend Rosella to execute a document entitled ―Continuing
Guaranty Agreement‖ whereby she expressly agreed to be solidarily liable for the
obligation of Kevin.
Can ABC Bank proceed directly against Rosella upon Kevin’s default even
without proceeding against Kevin first? Explain your answer. (3%)

SUGGESTED ANSWER:
Yes, ABC Bank may proceed directly against Rosella upon Kevin’s default even without
proceeding against Kevin first because Rosella is a surety after she bound herself
solidarily with the principal debtor.
Notwithstanding the use of the word ―guaranty‖ circumstances may be shown which
convert the contract into one of suretyship. Under the Civil Code, when the guarantor
binds himself solidarily with the principal debtor, the contract becomes one of suretyship
and not of guaranty proper. In a contract of suretyship, the liability of the surety is direct,
primary and absolute. He is directly and equally bound with the principal debtor. Such
being the case, a creditor can go directly against the surety although the principal
debtor is solvent and is able to pay or no prior demand is made on the principal debtor.
[Basis: Article 2047, Civil Code; Ong v. PCIB, 448 SCRA 705; discussed in pp. 810-812,
Vol. 2, Rabuya’s Civil Law Reviewer]

In this case, since Rosella is a surety, ABC Bank can go directly against her even
without proceeding against the principal debtor because the surety insures the debt,
regardless of whether or not the principal debtor is financially capable to fulfil his
obligation.
IV.

a. Distinguish antichresis from usufruct. (3%)

SUGGESTED ANSWER:
They are distinguished as follows:

Antichresis is always a contract while usufruct need not arise from a contract because it
may also be constituted by law or by other acts inter vivos, such as donation, or in a last
will and testament, or by prescription.
The subject matter of antichresis is always a real property while the subject matter of
usufruct may either be real property or personal property.
Antichresis is an accessory contract or contract of security while usufruct is a real right.
While in both, the fruits do not pertain to the owner, the usufructuary is entitled to enjoy
the fruits while the antichretic creditor has the obligation to apply the fruits to the
payment of the interest, if owing, and therefatre to the principal of the credit.

b. Distinguish commodatum from mutuum. (3%)


They are distinguished, as follows:

As to subject matter: The subject matter of commodatum is ordinarily non-consumable


while the subject matter of mutuum is either money or consumable;
As to compensation: Commodatum is essentially gratuitous while mutuum may be
gratuitous or with a stipulation to pay interest;
As to right in subject matter: In commodatum, there is no transmission fo ownership of
the thing loaned while in mutuum, the borrower acquires ownership of the thing
borrowed.
2

As to duty of borrower: In commodatum, the same thing borrowed is required to be


Page

returned while in mutuum, the borrower discharges himself, not by returning the

CIVIL LAW REVIEW 2 – 4A BAR EXAMINATION Q & A (1980-2017)


identical thing loaned, but by paying its equivalent in kind, quality and quantity.
[Discussed in pp. 725-726, Vol. 1, Rabuya’s Civil Law Reviewer]
III.

Josef owns a piece of land in Pampanga. The National Housing Authority (NHA) sought
to expropriate the property for its socialized housing project. The trial court fixed the just
compensation for the property at P50 million. The NHA immediately
deposited the same at the authorized depository bank and filed a motion for the
issuance of a writ of possession with the trial court. Unfortunately, there was delay in
the resolution of the motion. Meanwhile, the amount deposited earned interest.

When Josef sought the release of the amount deposited, NHA argued that Josef should
only be entitled to P50 million.

Who owns the interest earned? (3%)

SUGGESTED ANSWER:
The interest earned belongs to Josef because bank interest partakes of the nature of
civil fruits under Article 442 of the Civil Code and shall belong to the owner of the
principal thing.
When the National Housing Authority deposited the P50 Million as payment for the just
compensation with an authorized depositary bank for the purpose of obtaining a writ of
possession, it is deemed to be a constructive delivery of the said amount to Josef. Since
Josef is entitled to the P50 Million and undisputably the owner of the said principal
amount, the interest yield, as accession, in a bank deposit should likewise pertain to the
owner of the money deposited. Being an attribute of ownership (jus fruendi), Josef’s
right over the fruits, that is the bank interests, must be respected. [Basis: Republic v.
Holy Trinity Realty Development Corp., G.R. No. 172410, April 14, 2008]

I. Loan

The borrower in a contract of loan or mutuum must pay interest to the lender.
A. If there is an agreement in writing to the effect. (2012 BAR)
B. As a matter of course.
C. If the amount borrowed is very large.
D. If the lender so demands at the maturity date.

Siga-an granted a loan to Villanueva in the amount of P 540, 000.00. Such


agreement was not reduced to writing. Siga-an demanded interest which was paid
by Villanueva in cash and checks. The total amount Villanueva paid accumulated
to P 1, 200, 000.00. Upon advice of her lawyer, Villanueva demanded for the return
of the excess amount of P 660, 000.00 which was ignored by Siga-an. (2012 BAR)
Is the payment of interest valid? Explain. Answer:

SUGGESTED ANSWER:
NO. Art. 1956, provides that "no interest shall be due unless it has been expressly
stipulated in writing".
Is solutio indebiti applicable? Explain. Answer:

SUGGESTED ANSWER:
YES. Under Art. 1960, if the borrower pays interest when there has been no stipulation
3

thereof, the provisions of the Civil Code concerning solutio indebiti shall be applied.
Page

CIVIL LAW REVIEW 2 – 4A BAR EXAMINATION Q & A (1980-2017)


Villanueva paid in excess of P660.000 representing interest payment which is not due.
Therefore, he can demand its return.
Sarah had a deposit in a savings account with Filipino Universal Bank in the
amount of five Million pesos (P5,000,000.00). To buy a new car, she obtained a
loan from the same bank in the amount of P1,200,000.00, payable in twelve
monthly installments. Sarah issued in favor of the bank in post-dated checks,
each in the amount of P100,000.00 to cover the twelve monthly installment
payments. On the third, fourth and fifth months, the corresponding checks
bounced.
The bank then declared the whole obligation due, and proceed to deduct the
amount of one million pesos (P1,000,000.00) from Sarah's deposit after notice to
her that this is a form of compensation allowed by law. Is the bank correct?
Explain. (4%) (2009 Bar Question)

SUGGESTED ANSWER:
No the bank is not correct, while the Bank is correct about the applicability of
compensation, it was not correct as to the amount compensated.
A bank deposit is a contract of loan, where the depositor is the creditor and the bank the
debtor. Since Sarah is also the debtor of the bank with respect to the loan, both are
mutually principal debtors and creditors of each other. Both obligations are due,
demandable and liquidated but only up to the extent of P300,000 (covering the unpaid
third, fourth and fifth monthly installments). The entire one million was not yet due
because the loan has no acceleration clause in case of default. And since there is no
retention or controversy commenced by third persons and communicated in due time to
the debtor, then all the requisites of legal compensation are present but only up to the
amount of P300,000. The bank, therefore, may deduct P300,000 pesos from Sarah's
bank deposit by way of compensation.

Felipe borrowed $100 from Gustavo in 1998, when the Phil P - US$ exchange rate
was P56 - US$1. On March 1, 2008, Felipe tendered to Gustavo a cashier’s check
in the amount of P4,135 in payment of his US$ 100 debt, based on the Phil P - US$
exchange rate at that time. Gustavo accepted the check, but forgot to deposit it
until Sept. 12, 2008. His bank refused to accept the check because it had become
stale. Gustavo now wants Felipe to pay him in cash the amount of P5,600.
Claiming that the previous payment was not in legal tender, and that there has
been extraordinary deflation since 1998, and therefore, Felipe should pay him the
value of the debt at the time it was incurred. Felipe refused to pay him again,
claiming that Gustavo is estopped from raising the issue of legal tender, having
accepted the check in March, and that it was Gustavo’s negligence in not
depositing the check immediately that caused the check to become stale.
Can Gustavo now raise the issue that the cashier’s check is not legal tender?
(2%) (2008 Bar Question)
MAIN SUGGESTED ANSWER:
No, because Gustavo is guilty of estoppel by laches. He led Felipe to believe he could
pay by cashier’s check, and Felipe relied that such cashier’s check would be encashed
thus extinguishing his obligation. Because of Gustavo’s inaction of more than six
months the check became stale and Felipe will prejudiced if he will be required to pay
$100 at the exchange rate of P56 to $ 1.00. The exchange should be the rate at the
time of payment.
4
Page

CIVIL LAW REVIEW 2 – 4A BAR EXAMINATION Q & A (1980-2017)


ALTERNATIVE ANSWER:
Yes. The cashier’s check is not legal tender until it is encashed. (Art. 1249,CC). the
cashier’s check by itself is not legal tender. (Cuaycong v. Ruiz, 86 Phil. 170 [1950];
Belisario v. Natividad, 60 Phil. 156 [1934]).

b) Can Felipe validly refuse to pay Gustavo again? (2%) (2008 Bar Question)

SUGGESTED ANSWER:
Yes, if the payment is valid. Since the bank considered the cashier’s check as being
stale for not having been encashed on time, then the cashier’s check may be issued
again. At any rate, non-payment of the amount to Gustavo would constitute unjust
enrichment.

Can Felipe compel Gustavo to receive US$100 instead? (1%) (2008 Bar Question)

SUGGESTED ANSWER:
Yes. Felipe can compel Gustavo to pay US $ 100 instead. Under the prior law, RA 529,
as amended by R.A. 4100, payment can only be in Philippine currency as it would be
against public policy, null and void and of no effect. However, under RA 8183, payment
maybe made in the currency agreed upon by the parties, and the rate of exchange to be
followed is at the time of payment. [C.F. Sharp & Co. Inc vs. Northwest Airlines, Inc.,
381 SCRA 314 [2002]).
Multiple choice: Choose the right answer. (2% each)

The parties to a bailment are the:


bailor;
bailee;
comodatario;
all of the above;
letters a and b (2007 Bar Question)
SUGGESTED ANSWER:
e (letters a & b)

ALTERNATIVE ANSWER:
d (all of the above)

Cruz lent Jose his car until Jose finished his Bar exams. Soon after Cruz
delivered the car, Jose brought it to Mitsubishi Cubao for maintenance check up
and incurred costs of P8,000. Seeing the car's peeling and faded paint, Jose also
had the car repainted for P10,000. Answer the two questions below based on
these common facts. (2013 BAR)
After the bar exams, Cruz asked for the return of his car. Jose said he would
return it as soon as Cruz has reimbursed him for the car maintenance and
repainting costs of P 18,000.
Is Jose's refusal justified? (1%)

a. No, Jose's refusal is not justified. In this kind of contract, Jose is obliged to pay
for all the expenses incurred for the preservation of the thing loaned.
b. Yes, Jose's refusal is justified. He is obliged to pay forall the ordinary and
extraordinary expenses, but subject to reimbursement from Cruz.
5

c. Yes, Jose's refusal is justified. The principle of unjust enrichment warrants the
Page

reimbursement of Jose's expenses.

CIVIL LAW REVIEW 2 – 4A BAR EXAMINATION Q & A (1980-2017)


d. No, Jose's refusal is not justified. The expenses he incurred are useful for the
preservation of the thing loaned. It is Jose's obligation to shoulder these useful
expenses.

ANSWER:

NO CORRECT CHOICE – in commodatum, the bailee has no right of retention Article


1944 the bailee (Jose) has no right of retention even if it may be by reason of expenses,
Article 1951 he can only retain if he suffers damage by reason of a flaw or defect in the
thing
During the bar exam month, Jose lent the car to his girlfriend, Jolie, who parked
the car at the Mall of Asia's open parking lot, with the ignition key inside the car.
Car thieves broke into and took the car.
Is Jose liable to Cruz for the loss of the car due to Jolie's negligence? (1%) (2012
BAR)
A. No, Jose is not liable to Cruz as the loss was not due to his fault or negligence.
B. No, Jose is not liable to Cruz. In the absence of any prohibition, Jose could
lend the car to Jolie. Since the loss was due to force majeure, neither Jose nor
Jolie is liable.
C. Yes, Jose is liable to Cruz. Since Jose lent the car to Jolie without Cruz's
consent, Jose must bear the consequent loss of the car.
D. Yes, Jose is liable to Cruz. The contract between them is personal in nature.
Jose can neither lend nor lease the car to a third person.
ANSWER:
D – Commodatum is purely personal in nature (Article 1939) the bailee can neither lend
nor lease the object of the contract to a third person.
Before he left for Riyadh to work as a mechanic, Pedro left his Adventure van with
Tito, with the understanding that the latter could use it for one year for his
personal or family use while Pedro works in Riyadh. He did not tell Tito that the
brakes of the van were faulty. Tito had the van tuned up and the brakes repaired.
He spent a total amount of PI 5.000.00. After using the vehicle for two weeks. Tito
discovered that it consumed too much fuel. To make up for the expenses, he
leased it to Annabelle. Two months later. Pedro returned to the Philippines and
asked Tito to return the van. Unfortunately, while being driven by Tito, the van
was accidentally damaged by a cargo truck without his fault.
Who shall bear the P15,000.00 spent for the repair of the van? Explain. (2%) (2005
Bar Question)
SUGGESTED ANSWER:
The contract between Pedro and Tito is one of commodatum. Of the P15.000.00 spent,
Pedro, the bailor, shall bear the expenses for the repair of the faulty brakes, they being
extraordinary expenses incurred due to the non-disclosure by the bailor of the defect or
fault; Tito, on the other hand, shall shoulder that part of the P15,000.00 spent for the
tune-up, said expense being ordinary for the use and preservation of the van.

Who shall bear the costs for the van’s fuel, oil and other materials while it was
with Tito? Explain. (2%) (2005 Bar Question)

SUGGESTED ANSWER:
6

The costs for the fuel and other materials are considered ordinary expenses, and
Page

consequently Tito, the bailee, shall shoulder them. (Art. 1941, Civil Code)

CIVIL LAW REVIEW 2 – 4A BAR EXAMINATION Q & A (1980-2017)


Does Pedro have the right to retrieve the van even before the lapse of one year?
Explain. (2%) (2005 Bar Question)
SUGGESTED ANSWER:
No, Pedro cannot demand the return of the van until after the expiration of the one -year
period stipulated. However, if in the meantime he should have urgent need of the van,
he may demand its return or temporary use.

Who shall bear the expenses for the accidental damage caused by the cargo
truck, granting that the truck driver and truck owner are insolvent? Explain. (2%)
(2005 Bar Question)
SUGGESTED ANSWER:
Both Tito and Pedro shall bear equally the costs of the extraordinary expenses, having
been incurred on the occasion of actual use of the van by Tito, the bailee, even though
he acted without fault. (Art. 1949(2), Civil Code)

Distinguish briefly but clearly between: Mutuum and commodatum. (2004 Bar
Question)
SUGGESTED ANSWER:
In mutuum, the object borrowed must be a consumable thing the ownership of which is
transferred to the borrower who incurs the obligation to return the same consumable to
the lender in an equal amount, and of the same kind and quality. In commodatum, the
object borrowed is usually a non-consumable thing the ownership of which is not
transferred to the borrower who incurs the obligation to return the very thing to the
lender.
The parties in a contract of loan of money agreed that the yearly interest rate is
12% and it can be increased if there is a law that would authorize the increase of
interest rates. Suppose OB, the lender, would increase by 5% the rate of interest
to be paid by TY, the borrower, without a law authorizing such increase, would
OB’s action be just and valid? Why? Has TY a remedy against the imposition of
the rate increase? Explain. (5%) (2004 Bar Question)
SUGGESTED ANSWER:
OB’s action is not just and valid. The debtor cannot be required to pay the increase in
interest there being no law authorizing it, as stipulated in the contract. Increasing the
rate in the absence of such law violates the principle of mutuality of contracts.
ALTERNATIVE ANSWER:
Even if there was a law authorizing the increase in interest rate, the stipulation is still
void because there is no corresponding stipulation to decrease the interest due when
the law reduces the rate of interest.
Carlos sues Dino for (a) collection on a promissory note for a loan, with no
agreement on interest, on which Dino defaulted, and (b) damages caused by Dino
on his (Carlos’) priceless Michaelangelo painting on which Dino accidentally
spilled acid while transporting it. The court finds Dino liable on the promissory
note and awards damages to Carlos for the damaged painting, with interests for
both awards. What rates of interest may the court impose with respect to both
awards? Explain. (5%) (2002 Bar Question)
SUGGESTED ANSWER:
With respect to the collection of money or promissory note, it being a forbearance of
7

money, the legal rate of interest for having defaulted on the payment of 12% will apply.
Page

With respect to the damages to the painting, it is 6% from the time of the final demand

CIVIL LAW REVIEW 2 – 4A BAR EXAMINATION Q & A (1980-2017)


up to the time of finality of the decision and 12% of the total amount from finality of
judgment until judgment credit is fully paid. The court considers the latter as a
forbearance of money. (Eastern Shipping Lines, Inc. v. CA, 234 SCRA 78 [1994]; Art
2210 and 2211, CC)
Samuel borrowed P300.000.00 housing loan from the bank at 18% per annum
interest. However, the promissory note contained a proviso that the bank
―reserves the right to increase interest within the limits allowed by law." By virtue
of such proviso, over the objections of Samuel, the bank increased the interest
rate periodically until it reached 48% per annum. Finally. Samuel filed an action
questioning the right of the bank to increase the interest rate up to 48%. The bank
raised the defense that the Central Bank of the Philippines had already
suspended the Usury Law. Will the action prosper or not? Why? (5%) (2001 Bar
Question)
SUGGESTED ANSWER:
The action will prosper. While it is true that the interest ceilings set by the Usury Law are
no longer in force, it has been held that PD No. 1684 and CB Circular No. 905 merely
allow contracting parties to stipulate freely on any adjustment in the interest rate on a
loan or forbearance of money but do not authorize a unilateral increase of the interest
rate by one party without the other’s consent (PNB v. CA, 238 SCRA 20 [1994]]). To say
otherwise will violate the principle of mutuality of contracts under Article 1308 of the Civil
Code. To be valid, therefore, any change of interest must be mutually agreed upon by
the parties (Dizon v. Magsaysay, 57 SCRA 250 [1974]). In the present problem, the
debtor not having given his consent to the increase in interest, the increase is void.
Distinguish usufruct from commodatum and state whether these may be
constituted over consumable goods. [2%] (1998 Bar Question)
SUGGESTED ANSWER:
Usufruct is a right given to a person (usufructuary) to enjoy the property of another with
the obligation of preserving its form and substance. (Art. 562, Civil Code)
On the other hand, commodatum is a contract by which one of the parties (bailor)
delivers to another (bailee) something not consumable so that the latter may use it for a
certain time and return it.
In usufruct, the usufructuary gets the right to the use and to the fruits of the same, while
in commodatum, the bailee only acquires the use of the thing loaned but not its fruits.
Usufruct may be constituted on the whole or a part of the fruits of the thing. (Art. 564,
Civil Code). It may even be constituted over consumables like money (Altman v. Veloso,
52 Phil. 545). On the other hand, in commodatum, consumable goods maybe subject
thereof only when the purpose of the contract is not the consumption of the object, as
when it is merely for exhibition. (Art. 1936, Civil Code)
ANOTHER ANSWER:
There are several points of distinction between usufruct and commodatum. Usufruct is
constituted by law, by contract, by testamentary succession, or by prescription (Art.
1933, Civil Code). Usufruct creates a real right to the fruits of another's property, while
commodatum creates only a purely personal right to use another's property, and
requires a stipulation to enable the bailee to ―make use‖ of the fruits (Arts. 1939 & 1940,
Civil Code). Usufruct may be onerous while commodatum is always or essentially
gratuitous (Arts. 1933 &: 1935, Civil Code). The contract constituting usufruct is
consensual, while commodatum is a real contract (perfected only by delivery of the
subject matter thereof). However, both involve the enjoyment by a person of the
property of another, differing only as to the extent and scope of such enjoyment [jus
8
Page

fruendi in one and jus utendi in the other); both may have as subject matter either an

CIVIL LAW REVIEW 2 – 4A BAR EXAMINATION Q & A (1980-2017)


immovable or a movable; and, both may be constituted over consumable goods (Arts.
574 & 1936, Civil Code).

A consumable thing may be the subject-matter of an abnormal usufruct but in a normal


usufruct, the subject- matter maybe used only for exhibition. A commodatum of a
consumable thing may be only for the purpose of exhibiting, not consuming it.
In order to secure a bank loan, XYZ Corporation surrendered its deposit
certificate, with a maturity date of 01 September 1997 to the bank. The
corporation defaulted on the due repayment of the loan, prompting the bank to
encash the deposit certificate. XYZ Corporation questioned the above action
taken by the bank as being a case of pactum commissorium. The bank disagrees.
What is your opinion? (1997 Bar Question)

SUGGESTED ANSWER:
We submit that there is no pactum commissorium here. Deposits of money in banks and
similar institutions are governed by the provisions on simple loans (Art. 1980, Civil
Code). The relationship between the depositor and a bank is one of creditor and debtor.
Basically this is a matter of compensation as all the elements of compensation are
present in this case [BPI vs. CA, 232 SCRA 302).
ADDITIONAL ANSWER:
Where the security for the debt is also money deposited in a bank, it is not illegal for the
creditor to encash the time deposit certificates to pay the debtor's overdue obligation.
(Chu vs. CA, et al., G.R. 78519. September 26, 1989).
A, upon request, loaned his passenger Jeepney to B to enable B to bring his sick
wife from Paniqui. Tarlac to the Philippine General Hospital in Manila for
treatment. On the way back to Paniqui, after leaving his wife at the hospital,
people stopped the passenger jeepney. B stopped for them and allowed them to
ride on board, accepting payment from them just as in the case of ordinary
passenger jeepneys plying their route. As B was crossing Bamban, there was an
onrush of lahar from Mt. Pinatubo. The jeep that was loaned to him was wrecked.
What do you call the contract that was entered into by A and B with respect to the
passenger jeepney that was loaned by A to B to transport the latter’s sick wife to
Manila?
Is B obliged to pay A for the use of the passenger jeepney?

Is B liable to A for the loss of the jeepney? (1993 Bar Question)

SUGGESTED ANSWER:
A. The contract is called ―commodatum‖. (Art. 1933, Civil Code)

B. No, B is not obliged to pay A for the use of the passenger jeepney because
commodatum is essentially gratuitous. (Art. 1933, Civil Code)
Yes, because B devoted the thing to a purpose different from that for which it has been
loaned (Art. 1942, par. 2, Civil Code)
ALTERNATIVE ANSWER:

No, because an obligation which consists in the delivery of a determinate thing shall be
extinguished if it should be lost or destroyed without the fault of the debtor, and before
he has incurred in delay. (Art. 1262, Civil Code)
9
Page

CIVIL LAW REVIEW 2 – 4A BAR EXAMINATION Q & A (1980-2017)


II. Deposit

Due to the continuous heavy rainfall, the major streets in Manila became flooded.
This compelled Cris to check-in at Square One Hotel. As soon as Crisgot off from
his Toyota Altis, the Hotel’s parking attendant got the key of his car and gave him
a valet parking customer’s claim stub. The attendant parked his car at the
basement of the hotel. Early in the morning, Cris was informed by the hotel
manager that his car was carnapped. (2014 BAR)
What contract, if any, was perfected between Cris and the Hotel when Cris
surrendered the key of his car to the Hotel’s parking attendant?
Answer:
The contract between Cris and Square One Hotel is one of necessary deposit. Deposit
of effects made by travelers or guests in hotels or inns is considered a necessary
deposit (Art. 1998). This includes not only the personal effects brought inside the hotel
premises but also vehicles or animals and articles which have been introduced or
placed in the annexes of the hotel.

What is the liability, if any, of the Hotel for the loss of Cris’ car?

Answer:

In the case of Durban Apartments v. Pioneer Insurance (G.R. No. 179419 January 12,
2011), the Supreme Court held the hotel liable for the loss of the vehicle of the guest
after its valet parking attendant parked the vehicle in front of a bank near the hotel
premises. The court ruled that the bank’s parking area became an annex of the hotel
when the management of the bank allowed the hotel to park vehicles there on the night
in question. The contract of deposit was perfected when the guest surrendered the keys
to his vehicle to the parking attendant and the hotel is under obligation of safely keeping
and returning it. Ultimately, Square One Hotel is liable for the loss of the vehicle.
Who enjoys the Right of Retention? (1%)
(A) Depositary until full payment of what may be due him in deposit.

B. Lessee if he advances the expenses for the repair of the leased premises.
C. Bailee if bailor owes him something.
D. Builder in bad faith for the recovery of necessary and useful expenses.
Answer:
Letter A – depositary (Article 1994)
Multiple choice: Choose the right answer. (2% each)

1. x x x
A deposit made in compliance with a legal obligation is:
A. an extrajudicial deposit;
B. a voluntary deposit;
C. a necessary deposit;
D. a deposit with a warehouseman;
E. letters a and b (2007 Bar Question)
10

SUGGESTED ANSWER:
c (necessary deposit)
Page

CIVIL LAW REVIEW 2 – 4A BAR EXAMINATION Q & A (1980-2017)


X and Y staged a daring bank robbery in Manila at 10:30 A.M. in the morning of a
regular business day, and escaped with their loot of two (2) bags, each bag
containing P50.000.00.
During their flight to elude the police. X and Y entered the nearby locked house of
A, then working in his Quezon City office. From A's house. X and Y stole a box
containing cash totalling P50.000.00 which box A had been keeping in deposit for
his friend B.
In their hurry, X and Y left in A's bedroom one (1) of the bags which they had
taken from the bank.
With X and Y now at large and nowhere to be found, the bag containing
P50.000.00 is now claimed by B, by the Mayor of Manila, and by the bank.
B claims that the depository, A, by force majeure had obtained the bag of money
in place of the box of money deposited by B.
The Mayor of Manila, on the other hand, claims that the bag of money should be
deposited with the Office of the Mayor as required of the finder by the provisions
of the Civil Code.
A. The bank resists the claims of B and the Mayor of Manila.

To whom should A deliver the bag of money? Decide with reasons. (1992 Bar
Question)
SUGGESTED ANSWER:

B would have no right to claim the money. Article 1990 of the Civil Code is not
applicable. The law refers to another thing received in substitution of the object
deposited and is predicated upon something exchanged.
The Mayor of Manila cannot invoke. Article 719 of the Civil Code which requires the
finder to deposit the thing with the Mayor only when the previous possessor is
unknown. In this case, a must return the bag of money to the bank as the previous
possessor and known owner (Arts. 719 and 1990 Civil Code).

Ana rented a safety deposit box at the Alto Bank, paid the rental fee and was
given the key. Ana put her jewelry and gold coins in the box. Days after, three
armed men gained entry into the Alto Bank, opening its vault and several safety
deposit boxes, including Ana’s and emptied them of their contents.
Could Ana hold the Alto Bank liable for the loss of the contents of her deposit
box? Explain. (1987 Bar Question)
SUGGESTED ANSWER:
No, because under Article 1990 of the Civil Code, if the depository by force majeure
loses the thing and receives money or another thing in its place, he shall deliver the
sum or other thing to be depositor. There being no showing that there was anything
received in place of the things deposited the Alto Bank is not liable for the contents of
the safety box.
ALTERNATIVE ANSWER:
The Alto Bank is not liable because the contract is not a deposit but a rental of the
safety deposit box. Hence, the Alto Bank is not liable for the loss of the contents of the
11

box.
Page

CIVIL LAW REVIEW 2 – 4A BAR EXAMINATION Q & A (1980-2017)


III. Guaranty and Suretyship

Define, Enumerate or Explain. (2% each)

What is the difference between ―guaranty‖ and ―suretyship‖? (2010 Bar Question)

SUGGESTED ANSWER:
Guaranty and Suretyship distinguished:

The obligation in guaranty is secondary; whereas, in suretyship, it is primary.


In guaranty, the undertaking is to pay if the principal debtor cannot pay;
whereas, in suretyship, the undertaking is to pay if the principal debtor does not
pay.
In guaranty, the guarantor is entitled to the benefit of excussion; whereas, in
suretyship the surety is not so entitled.
Liability in guaranty depends upon an independent agreement to pay the
obligations of the principal if he fails to do so; whereas, in suretyship, the surety
assumes liability as a regular party.
The Guarantor insures the solvency of the principal debtor; whereas, the surety
insures the debt.
In a guaranty, the guarantor is subsidiarily liable; whereas, in a Suretyship, the
surety binds himself solidarily with the principal debtor. (Art. 2047, Civil Code)

TRUE or FALSE. Answer TRUE if the statement is true, or FALSE if the statement
is false. Explain your answer in not more than two (2) sentences. (5%)
x x x
[d] An oral promise of guaranty is valid and binding. (2009 Bar Question)
SUGGESTED ANSWER:
FALSE. An oral contract of guaranty, being a special promise to answer for the debt of
another, is unenforceable unless in writing (Article 1403 [2] b, NCC).
ANOTHER SUGGESTED ANSWER:
TRUE. An oral promise of guaranty is valid and binding. While the contract is valid,
however, it is unenforceable because it is not in writing. Being a special promise to
answer for the debt, default, or miscarriage of another, the Statute of Frauds requires it
to be in writing to be enforceable (Article 1403 [2] b, NCC). The validity of a contract
should be distinguished from its enforceability.
In an action brought to collect a sum of money based on a surety agreement, the
defense of laches was raised as the claim was filed more than seven years from
the maturity of the obligation. However, the action was brought within the ten-
year prescriptive period provided by law wherein actions based on written
contracts can be instituted.
Will the defense prosper? Reason. (3%)

What are the essential elements of laches? (2%) (2000 Bar Question)

SUGGESTED ANSWER:
No, the defense will not prosper. The problem did not give facts from which laches may
be inferred. Mere delay in filing an action, standing alone, does not constitute laches
[Agra v. PNB, 309 SCRA 509).
The four basic elements of laches are: (1) conduct on the part of the defendant or of
12

one under whom he claims, giving rise to the situation of which complainant seeks a
remedy; (2) delay in asserting the complainant’s rights, the complainant having had
Page

CIVIL LAW REVIEW 2 – 4A BAR EXAMINATION Q & A (1980-2017)


knowledge or notice of the defendant’s conduct and having been afforded an
opportunity to institute suit; (3) lack of knowledge on the part of the defendant that the
complainant would assert the right on which he bases his suit; and (4) injury or
prejudice to the defendant in the event relief is accorded to the complainant, or the suit
is not held to be barred.
AB sold to CD a motor vehicle for and in consideration of P120,000.00, to be paid
in twelve monthly equal installments of P10,000.00, each installment being due
and payable on the 15th day of each month starting January 1997.
To secure the promissory note, CD (a) executed a chattel mortgage on the subject
motor vehicle, and (b) furnished a surety bond Issued by Philamlife. CD failed to
pay more than two (2) Installments
AB went after the surety but he was only able to obtain three-fourths (3/4) of the
total amount still due and owing from CD. AB seeks your advice on how he might.
If at all, recover the deficiency.
How would you counsel AB? (1997 Bar Question)

SUGGESTED ANSWER:
Yes, he can recover the deficiency. The action of AB to go after the surety bond cannot
be taken to mean a waiver of his right to demand payment for the whole debt. The
amount received from the surety is only payment pro tanto, and an action may be
maintained for a deficiency debt.

IV. Pledge

Donna pledged a set of diamond ring and earrings to Jane for P200,000.00 She
was made to sign an agreement that if she cannot pay her debt within six months,
Jane could immediately appropriate the jewelry for herself. After six months,
Donna failed to pay. Jane then displayed the earrings and ring set in her jewelry
shop located in a mall. A buyer, Juana, bought the jewelry set for P300,000.00.
(2015 BAR)
Was the agreement which Donna signed with Jane valid? Explain with legal
basis.

Answer:
NO, the agreement to appropriate the jewelry upon default of Donna is considered
pactum commissorium and it is considered void by law ( Art. 2088).
Can Donna redeem the jewelry set from Juana by paying the amount she owed
Jane to Juana? Explain with legal basis.
Answer:
NO, Donna cannot redeem it from Juana because the pledge contract is between her
and Jane. Juana is not a party to the pledge contract (Art. 1311)
Give an example of a pledge created by operation of law. Answer:

One example of a pledge created by operation of law is the right of the depositary to
retain the thing deposited until the depositor shall have paid him whatever may be due
to the depositary by reason of the deposit (Art. 1994). Another is the right of the agent to
retain the thing which is the object of the agency until the principal reimburses him the
expenses incurred in the execution of the agency. (Art. 1914)
13

Ozamis Paper Corporation secured loans from ABC Universal Bank in the
Page

aggregate principal amount of P100 M, evidenced by several promissory notes,

CIVIL LAW REVIEW 2 – 4A BAR EXAMINATION Q & A (1980-2017)


and secured by a continuing guaranty of its principal stockholder Menandro
Marquez; a pledge of Marquez’s shares in the corporation valued at P45 M; and a
real estate mortgage over certain parcels of land owned by Marquez.
The corporation defaulted and the bank extra-judicially foreclosed on the real
estate mortgage. The bank, which was the sole bidder for P75 M, won the award.
x x x
Can the bank foreclose on the pledged shares of Marquez and recover the
deficiency from the corporation? (2010 Bar Question)
SUGGESTED ANSWER:
If the bank forecloses the pledge, it cannot recover the deficiency because the
foreclosure extinguishes the principal obligation, whether or not the proceeds from the
foreclosure are equal to the amount of the principal obligation.
Rosario obtained a loan of P100,000.00 from Jennifer, and pledge her diamond
ring. The contract signed by the parties stipulated and if Rosario is unable to
redeem the ring on due date, she will execute a document in favor of Jennifer
providing that the ring shall automatically be considered full payment of the loan
Is the contract valid? Explain. (3%) (2009 Bar Question)

SUGGESTED ANSWER:
The contract is valid because Rosario has to execute a document in favor of Jennifer to
transfer the ownership of the pledged ring to the latter. The contract does not amount to
a pactum commissorium because it does not provide for the automatic appropriation by
the pledge of the thing pledged in case of default by the pledgor.
Will your answer to [a] be the same if the contract stipulates that upon failure of
Rosario to redeem the ring on due date, Jennifer may immediately sell the ring
and appropriate the entire proceeds thereof for herself as full payment of the?
Reasons. (3%) (2009 Bar Question)
SUGGESTED ANSWER:

No, my answer will be different. While the contract of pledge is valid, the stipulation
authorizing the pledge to immediately sell the thing pledged is void under Article 2088 of
the new
Civil Code which provides that: ―the creditor cannot appropriate the things given by way
of pledge or mortgage, or dispose of them x xx‖. Jennifer cannot immediately sell by
herself the thing pledge.
It must be foreclosed by selling it at a public auction in accordance with the procedure
under Article 2112 of the New Civil Code.
ABC loaned to MNO P40.000 for which the latter pledged 400 shares of stock in
XYZ Inc. It was agreed that if the pledgor failed to pay the loan with 10% yearly
interest within four years, the pledgee is authorized to foreclose on the shares of
stock. As required, MNO delivered possession of the shares to ABC with the
understanding that the shares would be returned to MNO upon the payment of
the loan. However, the loan was not paid on time.
A month after 4 years, may the shares of stock pledged be deemed owned by
ABC or not? Reason. (5%) (2004 Bar Question)
SUGGESTED ANSWER:
14

The shares of stock cannot be deemed owned by ABC upon default of MNO. They have
Page

to be foreclosed. Under Article 2088 of the Civil Code, the creditor cannot appropriate

CIVIL LAW REVIEW 2 – 4A BAR EXAMINATION Q & A (1980-2017)


the things given by way of pledge. And even if the parties have stipulated that ABC
becomes the owner of the shares in case MNO defaults on the loan, such stipulation is
void for being a pactum commissorium.

Distinguish a contract of chattel mortgage from a contract of pledge. (2%) (1999


Bar Question)
SUGGESTED ANSWER:

In a contract of chattel mortgage possession belongs to the creditor, while in a contract


of pledge possession belongs to the debtor.
A chattel mortgage is a formal contract while a pledge is a real contract.

A contract of chattel mortgage must be recorded in a public instrument to bind third


persons while a contract of pledge must be in a public instrument containing description
of the thing pledged and the date thereof to bind third persons.
In 1982, Steve borrowed P400.000.00 from Danny, collateralized by a pledge of
shares of stock of Concepcion Corporation worth P800.000.00. In 1983, because
of the economic crisis, the value of the shares pledged fell to only P 100,000.00.
Can Danny demand that Steve surrender the other shares worth P700.000.00?
(1994 Bar Question)
SUGGESTED ANSWER:
No. Bilateral contracts cannot be changed unilaterally. A pledge is only a subsidiary
contract, and Steve is still indebted to Danny for the amount of P400.000.00 despite the
fall in the value of the stocks pledged.
No. Danny’s right as pledgee is to sell the pledged shares at a public sale and keep the
proceeds as collateral for the loan. There is no showing that the fall in the value of the
pledged property was attributable to the pledger’s fault or fraud. On the contrary, the
economic crisis was the culprit. Had the pledgee been deceived as to the substance or
quality of the pledged shares of stock, he would have had the right to claim another
thing in their place or to the immediate payment of the obligation. This is not the case
here.
What do you understand by ANTICHRESIS? How is it distinguished from pledge
and mortgage? (1989 Bar Question)
SUGGESTED ANSWER:
Antichresis is a contract whereby the creditor acquires the right to receive the fruits of
an immovable of his debtor with the obligation to apply them to the payment of interest if
owing and thereafter to the principal.
Pledge is an accessory and real contract whereby the debtor delivers to the creditor
movable property as security for the performance of a principal obligation upon the
fulfilment of which the thing pledged shall be returned to the debtor.
A real estate mortgage is an accessory contract whereby the debtor guarantees the
performance of the principal obligation by subjecting real property or real right as
security for the performance of such obligation.
ALTERNATIVE EXTENDED ANSWER:
By the contract of antichresis the creditor acquires the right to receive the fruits .of an
immovable of his debtor, with the obligation to apply them to the payment of the interest,
15

if owing* and thereafter to the principal of his credit.


Page

Antichresis distinguished from pledge:

CIVIL LAW REVIEW 2 – 4A BAR EXAMINATION Q & A (1980-2017)


Antichresis is consensual, pledge is a real contract.

Antichresis involves real property, pledge involves personal property.

In antichresis, the principal and the interest must be provided in writing for validity. In
pledge, the date and description of the pledge must be in a public instrument to affect
third persons.
Antichresis distinguished from mortgage:

In antichresis the fruits that are applied to .the interest and thereafter to the principal.
In mortgage the fruits are not applied to the principal obligation.
In antichresis, the creditor is in possession. In mortgage, the debtor is in possession.
The principal and interest must be in writing tor validity. In mortgage, registration is
required to bind third persons.
In antichresis, the creditor pays the taxes. In mortgage, taxes are not imposed on
the creditor.
RECOMMENDATION OF THE COMMITTEE:

If the above alternative answer is given, two (2) distinctions for each should be given full
credit.
A diamond ring and a female cow were pledged to secure a loan in the amount of
P100,000. The pledge appeared in a public instrument. A month later, the cow
gave birth. When the amount of the loan was not paid upon its maturity date, the
pledged caused to be sold at a public auction the ring, the cow and the cow’s
offspring and the amount of P150,000. as realized. The pledgor, upon learning of
the sale, demanded from the pledgee the excess in the price over and above the
amount of the principal obligation, claiming that he is entitled to the excess and
that the offspring was not included in the pledge. The pledgee refused to comply
with the demand. How would you decide this conflict? Give your reasons. (1989
Bar Question)
SUGGESTED ANSWER:
Debtor/pledgor is not entitled to the excess unless the contrary is agreed upon. The
offspring shall pertain to the pledgor but is subject to the pledge if there is no stipulation
to the contrary.

V. Real Mortgage

Mr. Bong owns several properties in Pasig City. He decided to build a


condominium named Flores de Manila in one of his lots. To fund the project, he
obtained a loan from the National Bank (NB) secured by a real estate mortgage
over the adjoining property which he also owned. During construction, he built
three (3) pumps on the mortgaged property to supply water to the condominium.
After one (1) year, the project was completed and the condominium was turned
over to the buyers. However, Mr. Bong failed to pay his loan obligation to NB.
Thus, NB foreclosed the mortgaged property where the pumps were installed.
During the sale on public auction of the mortgaged property, Mr. Simon won in
the bidding. When Mr. Simon attempted to take possession of the property, the
condominium owners, who in the meantime constituted themselves into Flores
de Manila Inc. (FMI), claimed that they have earlier filed a case for the declaration
of the existence of an easement before the Regional Trial Court (RTC) of Pasig
City and prayed that the easement be annotated in the title of the property
16

foreclosed by NB. FMI further claimed that when Mr. Bong installed the pumps in
his adjoining property, a voluntary easement was constituted in favor of FMI.
Page

CIVIL LAW REVIEW 2 – 4A BAR EXAMINATION Q & A (1980-2017)


Will the action prosper? (2014 BAR)

Answer:
NO, the action will not prosper. The essence of a mortgage is that it immediately
subjects the property upon which it is imposed, and whoever the possessor may be, to
the fulfillment of the obligation for whose security it was constituted (Art. 2126). There
was no voluntary easement in this case because at the time the water pumps were
constructed, the subject lot where the water pumps were constructed and the
condominium belong to the same person. No one can have an easement over his own
property (Bogo- Medellin v. CA G.R. 124699, July 31, 2003). Even of the assumption
that an easement was created in favor of FMI, that alone will not defeat the right of the
mortgagee to enforce the security if the debtor defaults.
Lito obtained a loan of P1,000,000 from Ferdie, payable within one year. To secure
payment, Lito executed a chattel mortgage on a Toyota Avanza and a real estate
mortgage on a 200-square meter piece of property. (2013 BAR)
Would it be legally significant - from the point of view of validity and
enforceability - if the loan and the mortgages were in public or private
instruments?
Answer:
From the point of view of validity and enforceability, there would be legal significance if
the mortgage was in public or private instrument. As for the loan, there is no legal
significance except if interest were charged on the loan, in which case the charging of
interest must be in writing. A contract of loan is a real contract and is perfected upon the
delivery of the object of the obligation (Art. 1914). Thus, a contract of loan is valid and
enforceable even if it is neither in a private nor in a public document. As a rule,
contracts shall he obligatory in whatever form they may have been entered into
provided all the essential requisites for their validity are present. With regard to its
enforceability, a contract of loan is not among those enumerated under Art. 1403 (2),
which are covered by the Statute of Frauds. It is important to note that under Art. 1358,
all other contracts where the amount involved exceeds five hundred pesos must appear
in writing, even a private one. However, the requirement is not for the validity of the
contract, but only for its greater efficacy. With regard the chattel mortgage, Act No.
1508, the Chattel Mortgage Law, requires an affidavit of good faith stating that the
chattel mortgage is supposed to stand as security for the loan; thus, for validity of the
chattel mortgage, it must be in a public document and recorded in the Chattel Mortgage
Register in the Registry of Deeds. A real estate mortgage under the provisions of Art.
2125 requires that in order that a mortgage may be validly constituted the document in
which it appears be recorded. If the instrument is not recorded, the mortgage is
nevertheless valid and binding between the parties. Hence, for validity of both chattel
and real estate mortgages, they must appear in a public instrument. But for purposes of
enforceability, it is submitted that the form of the contract, whether in a public or private
document, would be immaterial (Mobil Oil v. Diocares, G.R. No. L-26371, September
30, 1969). Also, under Art. 1358, acts and contracts which have for their object the
creation or transmission of real rights over immovable property must be in a public
document fur greater efficacy, and a real estate mortgage is a real right over immovable
property.

Lito's failure to pay led to the extra-judicial foreclosure of the mortgaged real
property. Within a year from foreclosure, Lito tendered a manager's check to
Ferdie to redeem the property. Ferdie refused to accept payment on the ground
that he wanted payment in cash: the check does not qualify as legal tender and
does not include the interest payment. Is Ferdie's refusal justified?
17
Page

CIVIL LAW REVIEW 2 – 4A BAR EXAMINATION Q & A (1980-2017)


Answer:
Ferdie’s refusal is justified. A check, whether a manager’s check or ordinary check, is
not legal tender, and an offer of a check in payment of a debt is not a valid tender if
payment and may be refused receipt by the obligee or creditors (Philippine Airlines v.
CA and Amelia Tan, GR. No. L-49188, January 30, 1990). Mere delivery of checks does
not discharge the obligation under a judgment. A check shall produce the effect of
payment only when they have been cashed or when through the fault of the creditor,
they have been impaired (Art. 1249). However, it is not necessary that the right of
redemption be exercised by delivery of legal tender. A check may be used for the
exercise of right of redemption, the same being a right and not an obligation. The tender
of a check is sufficient to compel redemption but is not in itself a payment that relieves
the redemptioner from his liability to pay the redemption price (Bianca v. Gimenez, G.R.
No. 132768, September 9, 2005, citing Fortunado v. CA). Redemption within the period
allowed by law is not a matter of intent but a question of payment of valid tender of full
redemption price within the said period. Whether the redemption is being made under
Act 3135 or under the General Banking Law, the mortgagor or his assignee is required
to tender payment to make said redemption valid (Heirs of Quisumbing v. PNB and
SLDC, GR. No. 178242, January 20, 2009). Moreover, Ferdie's refusal was justified on
the ground that the amount tendered does not include interest. In order to effect the
redemption of the foreclosed property, the payment to the purchaser must include the
following sums: (a) the bid price; (b) the interest on the bid price, computed at one per
centum (1%) per month; and (c) the assessments or taxes, if any, paid by the
purchaser, with the same rate of interest (Sec. 28, Rules of Court). Unless there is an
express stipulation to that effect, the creditor cannot be compelled to receive partial
payment of the prestation (Art. 1248).
Amador obtained a loan of P300,000 from Basilio payable on March25, 2012. As
security for the payment of his loan, Amador constituted a mortgage on his
residential house and lot in Basilio's favor. Cacho, a good friend of Amador,
guaranteed and obligated himself to pay Basilio, in case Amador fails to pay his
loan at maturity. (2013 BAR)
If Amador fails to pay Basilio his loan on March 25, 2012, can Basilio compel
Cacho to pay? (1%)
A. No, Basilio cannot compel Cacho to pay because as guarantor, Cacho can invoke
the principle of excussion, i.e., all the assets of Basilio must first be exhausted.
B. No, Basilio cannot compel Cacho to pay because Basilio has not exhausted
the available remedies against Amador.
C. Yes, Basilio can compel Cacho to pay because the nature of Cacho's undertaking
indicates that he has bound himself solidarily with Amador.
D. Yes, Basilio can compel Cacho who bound himself to unconditionally pay in case
Amador fails to pay; thus the benefit of excussion will not apply.
ANSWER:
B – Basilio has in his favor a REM and he should exhaust his legal remedies against
Amador. (Art. 2058)

If Amador sells his residential house and lot to Diego, can Basilio foreclose the
real estate mortgage? (1%)
a. Yes, Basilio can foreclose the real estate mortgage because real estate
18

mortgage creates a real right that attaches to the property.


Page

CIVIL LAW REVIEW 2 – 4A BAR EXAMINATION Q & A (1980-2017)


b. Yes, Basilio can foreclose the real estate mortgage. It is binding upon Diego as the
mortgage is embodied in a public instrument.
c. No, Basilio cannot foreclose the real estate mortgage. The sale confers ownership on
the buyer, Diego, who must therefore consent.
d. No, Basilio cannot foreclose the real estate mortgage. To deprive the new owner of
ownership and possession is unjustand inequitable.
ANSWER:
A- Art. 2126 The mortgage directly and immediately subjects the property upon which it
is imposed, whoever the possessor may be to the fulfillment of the obligation for whose
security it was constituted.
Asiong borrowed P1 million from a bank, secured by a mortgage on his land.
Without his consent, his friend Boyong paid the whole loan. Since Asiong
benefited from the payment, can Boyong compel the bank to subrogate him in its
right as mortgagee of Asiong's land? (2011 BAR)
A. No, but the bank can foreclose and pay Boyong back.
B. No, since Boyong paid for Asiong’s loan without his approval.
C. Yes, since a change of creditor took place by novation with the bank’s consent.
D. Yes, since it is but right that Boyong be able to get back his money and, if not, to
foreclose the mortgage in the manner of the bank.
The right of a mortgagor in a judicial foreclosure to redeem the mortgaged
property after his default in the performance of the conditions of the mortgage
but before the sale of the mortgaged property or confirmation of the sale by the
court, is known as (2011 BAR)
a. accion publiciana.
b. equity of redemption.
c. pacto de retro.
d. right of redemption.
Does the right to request for the issuance of a writ of possession over a
foreclosed real property prescribe in five (5) years? (2012 BAR)
Answer:

NO, the purchaser’s right to request for the issuance of the writ of possession of the
land never prescribes. The right to possess a property merely follows the right of
ownership, and it would be illogical to hold that a person having ownership of a parcel of
land is barred from seeking possession thereof (Spouses Edralin v. Philippine Veteran’s
Bank, G.R. No. 168523, March 9, 2011).
X obtained a P10 M loan from BBB Banking Corporation. The loan is secured by
REM on his vacation house in Tagaytay City. The original Deed of REM for the
P10 M was duly registered. The Deed of REM also provides that ―The mortgagor
also agrees that this mortgage will secure the payment of additional loans or
credit accommodations that may be granted by the mortgagee…‖ Subsequently,
because he needed more funds, he obtained another P5 M loan. On due dates of
both loans, X failed to pay the P5 M but fully paid the P10 M. BBB Banking
Corporation instituted extrajudicial foreclosure proceedings.
a. Will the extrajudicial foreclosure prosper considering that the additional P5
M was not covered by the registration?
19
Page

CIVIL LAW REVIEW 2 – 4A BAR EXAMINATION Q & A (1980-2017)


What is the meaning of a ―dragnet clause‖ in a Deed of Real Estate
Mortgage? Under what circumstances will be ―dragnet clause‖ applicable?
(2012 Bar Question)

SUGGESTED ANSWER:
Yes. X executed a REM containing a ―blanket mortgage clause‖. Mortgages given to
secure future advancements are valid and legal contracts, and the amounts names as
consideration in said contracts do not limit the amount for which the mortgage may
stand as security if from the four corners of the instrument the intent to secure future
and other indebtedness.
Generally, a dragnet clause is a clause in a deed of REM stating that the mortgage
secures all the loans and advances that the mortgagor may at any time owe to the
mortgagee. The word ―dragnet‖ is a reference to a net drawn through a river or across
ground to trap fish or game. It is also known in American jurisprudence as a ―blanket
mortgage clause‖ or an ―anaconda clause‖. A mortgage with a dragnet clause enables
the parties to provide continuous dealings, the nature or extent of which may not be
known or anticipated at the time, and they avoid the expense and inconvenience of
executing a new security on each new transaction. It operates as a convenience and
accommodation to the borrower as it makes available additional funds to him without his
having to execute additional security documents, thereby saving time, travel, costs of
extra legal services, recording fees, etc.
The ―dragnet clause‖ may not apply to other loans extended by the mortgagee to the
mortgagor for which other securities were given. In the case of Prudential Bank v.
Alviar, the Supreme Court adopted the ―reliance on the security test‖ to the effect that
―when the mortgagor takes another loan [from the mortgagee] for which another security
was given, it could not be inferred that such loan was made in reliance solely on the
original security with the ―dragnet clause‖, but rather, on the new security given‖. This
means that the existence of the new security must be respected and the foreclosure of
the old security should only be for the other loans not separately collateralized and for
any amount not covered by the new security for the new loan.
X, at Y’s request, executed a Real Estate Mortgage (REM) on his (X’s) land to
secure Y’s loan from Z. Z successfully foreclosed the REM when Y defaulted on
the loan but half of Y’s obligation remained unpaid. May Z sue X to enforce his
right to the deficiency? (2011 Bar Question)
A. Yes, but solidarily with Y.
B. Yes, since X’s is deemed to warrant that his land would cover the whole
obligation.
C. No, since it is the buyer at the auction sale who should answer for the
deficiency.
D. No, because X is not Z’s debtor.
SUGGESTED ANSWER:

d. No, because X is not Z’s debtor.

Eduardo was granted a loan by XYZ Bank for the purpose of improving a building
which XYZ leased from him. Eduardo, executed the promissory note (―PN‖) in
favor of the bank, with his friend Recardo as cosignatory. In the PN, they both
acknowledged that they are ―individually and collectively‖ liable and waived the
need for prior demand. To secure the PN, Recardo executed a real estate
mortgage on his own property. When Eduardo defaulted on the PN, XYZ stopped
payment of rentals on the building on the ground that legal compensation had set
20

in. Since there was still a balance due on the PN after applying the rentals, XYZ
foreclosed the real estate mortgage over Recardo’s property.
Page

CIVIL LAW REVIEW 2 – 4A BAR EXAMINATION Q & A (1980-2017)


Recardo opposed the foreclosure on the ground that he is only a co-signatory;
that no demand was made upon him for payment, and assuming he is liable, his
liability should not go beyond half the balance of the loan. Further, Recardo said
that when the bank invoked compensation between the rentals and the amount of
the loan, it amounted to a new contract or novation, and had the effect of
extinguishing the security since he did not give his consent (as owner of the
property under the real estate mortgage) thereto.
x x x
Can Recardo’s property be foreclosed to pay the full balance of the loan? (2%)
(2008 Bar Question)

MAIN SUGGESTED ANSWER:


No, because there was no prior demand on Ricardo, depriving him of the right to
reasonably block the foreclosure by payment. The waiver of prior demand in the PN is
against public policy and violates the right to due process. Without demand, there is no
default and the foreclosure is null and void. Since the mortgage, insofar as Ricardo is
concerned is not violated, a requirement under Act 3135 for a valid foreclosure of real
estate mortgage is absent.
In the case of DBP vs. Licuanan (516 SCRA 644 [2007]), it was held that: ―the issue of
whether demand was made before the foreclosure was effected is essential. If demand
was made and duly received by the respondents and the latter still did not pay, then
they were already in default and foreclosure was proper. However, if demand was not
made, then the loans had not yet become due and demandable. This meant that
respondents had not defaulted in their payment and the foreclosure was premature‖.
ALTERNATIVE ANSWER:
No. Although the principal obligation of loan is due and demandable without need of
further demand the foreclosure of the accessory contract of real estate mortgage, there
is a need of notice and demand.
ANOTHER ANSWER:
Yes. Recardo’s property can be foreclosed to pay the full balance of the loan. He is
admittedly ―individually and collectively‖ liable. His liability is solidary. He and Eduardo
have waived notice for a prior demand as provided in the promissory note.
Industry Bank, which has a net worth of P1 Billion, extended a loan to celestial
Properties Inc. amounting to P270 M. the loan was secured by a mortgage over a
vast commercial lot in the Fort Bonifacio Global City, appraised at P350 M. After
audit, the BSP gave notice that the loan to Celestial Properties exceeded the
single borrower’s limit of 25% of the bank’s net worth under a recent BSP
Circular. In light of other previous similar violations of the credit limit
requirement, the BSP advised Industry Bank to reduce the amount of the loan to
Celestial Properties under pain of severe sanctions. When Industry Bank
informed Celestial Properties that it intended to reduce the loan by P50 M,
Celestial Properties counter-proposal, and referred the matter to you as counsel.
How would you advise Industry Bank to proceed, with its best interest in mind?
(2008 Bar Question)
SUGGESTED ANSWER:
I shall advise Industry Bank that the mortgage is indivisible. Therefore, Celestial
Properties cannot ask for a partial release of the mortgage so long as the loan has not
been completely paid.
21

To secure a loan obtained from a rural bank, Purita assigned her leasehold rights
over a stall in the public market in favor of the bank. The deed of assignment
Page

provides that in case of default in the payment of the loan, the bank shall have the

CIVIL LAW REVIEW 2 – 4A BAR EXAMINATION Q & A (1980-2017)


right to sell Purita's rights over the market stall as her attorney-in-fact, and to
apply the proceeds to the payment of the loan.
Was the assignment of leasehold rights a mortgage or a cession? Why? (3%)
Assuming the assignment to be a mortgage, does the provision giving the bank
the power to sell Purita’s rights constitute pactum commissorium or not? Why?
(2%) (2001 Bar Question)
SUGGESTED ANSWER:
The assignment was a mortgage, not a cession, of the leasehold rights. A cession
would have transferred ownership to the bank. However, the grant of authority to the
bank to sell the leasehold rights in case of default is proof that no such ownership was
transferred and that a mere encumbrance was constituted. There would have been no
need for such authority had there been a cession.
No, the clause in question is not a pactum commissorium. It is pactum commissorium
when default in the payment of the loan automatically vests ownership of the
encumbered property in the bank. In the problem given, the bank does not automatically
become owner of the property upon default of the mortgagor. The bank has to sell the
property and apply the proceeds to the indebtedness.
Cesar bought a residential condominium unit from High Rise Co. and paid the
price in full. He moved into the unit, but somehow he was not given the
Condominium Certificate of Title covering the property. Unknown to him. High
Rise Co. subsequently mortgaged the entire condominium building to Metrobank
as security for a loan of P500 million. High Rise Co. failed to pay the loan and the
bank foreclosed the mortgage. At the foreclosure sale, the bank acquired the
building, being the highest bidder. When Cesar learned about this, he filed an
action to annul the foreclosure sale insofar as his unit was concerned. The bank
put up the defense that it relied on the condominium certificates of title presented
by High Rise Co., which were clean. Hence, it was a mortgagee and buyer in good
faith. Is this defense tenable or not? Why? (5%) (2001 Bar Question)
SUGGESTED ANSWER:

Metrobank’s defense is untenable. As a rule, an innocent purchaser for value acquires a


good and a clean title to the property. However, it is settled that one who closes his
eyes to facts that should put a reasonable man on guard is not an innocent purchaser
for value. In the present problem the bank is expected, as a matter of standard
operating procedure, to have conducted an ocular inspection, of the promises before
granting any loan. Apparently, Metrobank did not follow this procedure, otherwise, it
should have discovered that the condominium unit in question was occupied by Cesar
and that fact should have led it to make further inquiry. Under the circumstances,
Metrobank cannot be considered a mortgagee and buyer in good faith.
A. X borrowed money from Y and gave a piece of land as security by way of
mortgage. It was expressly agreed between the parties in the mortgage contract
that upon non-payment of the debt on time by X, the mortgaged land would
already belong to Y. If X defaulted in paying, would Y now become the owner of
the mortgaged land? Why? (3%)
Suppose in the preceding question, the agreement between X and Y was that if X
failed to pay the mortgage debt on time, the debt shall be paid with the land
mortgaged by X to Y. Would your answer be the same as in the preceding
question? Explain. (3%) (1999 Bar Question)
22
Page

CIVIL LAW REVIEW 2 – 4A BAR EXAMINATION Q & A (1980-2017)


SUGGESTED ANSWER:
No, Y would not become the owner of the land. The stipulation is in the nature of
pactum commissorium which is prohibited by law. The property should be sold at public
auction and the proceeds thereof applied to the indebtedness. Any excess shall be
given to the mortgagor.
No, the answer would not be the same. This is a valid stipulation and does not
constitute pactum commissorium. In pactum commissorium, the acquisition is automatic
without need of any further action. In the instant problem another act is required to be
performed, namely, the conveyance of the property as payment (dacion en pago).
Various buyers of lots in a subdivision brought actions to compel either or both
the developer and the bank to release and deliver free and clear the titles to their
respective lots.
The problem arose because notwithstanding prior sales mostly on installments—
made by the developer to buyers, developer had mortgaged the whole
subdivision to a commercial bank. The mortgage was duly executed and
registered with the appropriate governmental agencies. However, as the lot
buyers were completely unaware of the mortgage lien of the bank, they
religiously paid the installments due under their sale contracts.
As the developer failed to pay its loan, the mortgage was foreclosed and the
whole subdivision was acquired by the bank as the highest bidder.
May the bank dispossess prior purchasers of individual lots or, alternatively,
require them to pay again for the paid lots? Discuss.

What are the rights of the bank vis-à-vis those buyers with remaining unpaid
installments? Discuss. (1999 Bar Question)
SUGGESTED ANSWER:
No. The bank may not dispossess the prior purchasers of the individual lots, much less
require them to pay for the paid lots. The bank has to respect the rights of the prior
purchasers of the individual lots. The purchasers have the option to pay the installments
of the mortgagee.
The bank has to respect the rights of the buyers with remaining unpaid installments.
The purchaser has the option to pay the installments to the mortgagee who should
apply the payments to the mortgage indebtedness.
Borrower obtained a loan against the security of a mortgage on a parcel of land.
While the mortgage was subsisting, borrower leased for 50 years the mortgaged
property to Land development Company (LDC). The mortgagee was duly advised
of the lease.
Thereafter, LDC constructed on the mortgaged property an office condominium.

Borrower defaulted on his loan and mortgagee foreclosed the mortgage. At the
foreclosure sale, the mortgagee was awarded the property as the highest bidder.
The corresponding Certificate of Sale was executed and after the lapse of 1 year,
title was consolidated in the name of the mortgagee.
Mortgagee then applied with the RTC for the issuance of a writ of possession not
only over the land but also the condominium building. The mortgagee contended
23

that the mortgage included all accessions, improvements and accessories found
on the mortgaged property.
Page

CIVIL LAW REVIEW 2 – 4A BAR EXAMINATION Q & A (1980-2017)


LDC countered that it had built on the mortgaged property with the prior
knowledge of mortgagee which had received formal notice of the lease.
How would you resolve the dispute between the mortgagee and LDC?

Is the mortgagee entitled to the lease rentals due from LDC under the lease
agreement? (1999 Bar Question)
SUGGESTED ANSWER:

a. The mortgagee has a better right than LDC. The mortgage extends to the
improvements introduced on the land, with the declarations, amplifications, and
limitations established by law, whether the estate remains in the possession of the
mortgagor or passes into the hands of a third person. The notice given by LDC to
the mortgagee was not enough to remove the building from coverage of the

mortgage considering that the building was built after the mortgaged was
constituted and the notice was only as regards the lease and not as to the
construction of the building. Since the mortgagee was informed of the lease and did
not object to it, the mortgagee became bound by the terms of the lease when it
acquired the property as the highest bidder. Hence, the mortgagee steps into the
shoes of the mortgagor and acquires the rights of the lessor under Article 1678 of
the Civil Code. This provision gives the lessor the right to appropriate the
condominium building but after paying the lessee half of the value of the building at
that time. Should the lessor refuse to reimburse said amount, the lessee may
remove the improvement even though the land will suffer damage thereby.
The lease rentals belong to the mortgagor. However, the mortgage extends to
rentals not yet received when the obligation becomes due and the mortgagee may
ran after the said rentals for the payment of the mortgage debt.
Debtor purchased a parcel of land from a realty company payable in 5 yearly
installments. Under the contract of sale, title to the lot would be transferred upon
full payment of the purchase price.
But even before full payment, debtor constructed a house on the lot. Sometime
thereafter, debtor mortgaged the house to secure his obligation arising from the
issuance of a bond needed in the conduct of his business. The mortgage was
duly registered with the proper chattel mortgage registry.
5 years later after completing payment of the purchase price, debtor obtained title
to the lot. And even as the chattel mortgage on the house was still subsisting,
debtor mortgaged to a bank the lot and improvement thereon to secure a loan.
This real estate mortgage was duly registered and annotated at the bank of the
title.
Due to business reverses, debtor failed to pay his creditors. The chattel mortgage
was foreclosed when the debtor failed to reimburse the surety company for
payments made on the bond. In the foreclosure sale, the surety company was
awarded the house as the highest bidder.
Only after the foreclosure sale did the surety company learn of the real estate
mortgage in favor of the lending investor on the lot and the improvement thereon.
Immediately, it filed a complaint praying for the exclusion of the house from the
real estate mortgage. It was submitted that as the chattel mortgage was executed
and registered ahead, it was superior to the real estate mortgage.
24

On the suggestion that a chattel mortgage on a house—a real property—was a


Page

nullity, the surety company countered that when the chattel mortgage was

CIVIL LAW REVIEW 2 – 4A BAR EXAMINATION Q & A (1980-2017)


executed, debtor was not yet the owner of the lot on which the house was built.
Accordingly, the house was a personal property and a proper subject of a chattel
mortgage.

Discuss the validity of the position taken by the surety company.

Who has a better claim to the house, the surety company or the lending investor?
Explain.
Would the position of the surety company be bolstered by the fact that it acquired
title in a foreclosure sale conducted by the Provincial Sheriff. Explain. (1999 Bar
Question)
SUGGESTED ANSWER:
The house is always a real property even though it was constructed on a land not
belonging to the builder. However, the parties may treat it as personal property and
constitute a chattel mortgage thereon. Such mortgage shall be valid and binding but
only on the parties. It will not bind or affect third parties.
The lending investor has a better claim to the house. The real estate mortgage covering
the house and lot was duly registered and binds the parties and third persons. On the
other hand, the chattel mortgage on the house securing the credit of the surety
company did not affect the rights of third parties such as the lending investor despite
registration of the chattel mortgage.
No. The chattel mortgage over the house which was foreclosed did not affect the rights
of third parties like the lending investor. Since third parties are not bound by the chattel
mortgage, they are not also bound by any enforcement of its provisions. The foreclosure
of such chattel mortgage did not bolster or add anything to the position of the surety
company.
Distinguish between a contract of real estate mortgage and a contract of sale with
right of repurchase. (1989 Bar Question)
SUGGESTED ANSWER:
Real estate mortgage is an accessory contract. A contract of sale with right of
repurchase is a principal contract.
Real estate mortgage involves no transfer of title. A contract of sale involves a
conditional transfer of title.
Real estate mortgage involves no transfer of possession. A contract of sale involves a
conditional transfer of possession.
In a real estate mortgage the creditor has no rights to the fruits. In a contract of sale, the
vendee is entitled to the fruits.
In a real estate mortgage, upon default the creditor is not the owner. In a contract of
sale, upon consolidation, the vendee is the owner.
RECOMMENDATION OF THE COMMITTEE:

Any three (3) of the foregoing distinctions should be given full credit.

Does an action to foreclose a real estate mortgage affecting registered land under
the Torrens System prescribe? Give your reasons. (1989 Bar Question)
SUGGESTED ANSWER:
Even if the property given as collateral is covered by a Torrens Title, the right to
foreclose a real estate mortgage thereon prescribes. This is really an action to enforce
25

collection of the loan.


Page

CIVIL LAW REVIEW 2 – 4A BAR EXAMINATION Q & A (1980-2017)


What do you understand by ANTICHRESIS? How is it distinguished from pledge
and mortgage? (1989 Bar Question)
SUGGESTED ANSWER:
Antichresis is a contract whereby the creditor acquires the right to receive the fruits of
an immovable of his debtor with the obligation to apply them to the payment of interest if
owing and thereafter to the principal.
Pledge is an accessory and real contract whereby the debtor delivers to the creditor
movable property as security for the performance of a principal obligation upon the
fulfilment of which the thing pledged shall be returned to the debtor.
A real estate mortgage is an accessory contract whereby the debtor guarantees the
performance of the principal obligation by subjecting real property or real right as
security for the performance of such obligation.
ALTERNATIVE EXTENDED ANSWER:
By the contract of antichresis the creditor acquires the right to receive the fruits .of an
immovable of his debtor, with the obligation to apply them to the payment of the interest,
if owing* and thereafter to the principal of his credit.
Antichresis distinguished from pledge:

Antichresis is consensual, pledge is a real contract.

Antichresis involves real property, pledge involves personal property.

In antichresis, the principal and the interest must be provided in writing for validity. In
pledge, the date and description of the pledge must be in a public instrument to affect
third persons.
Antichresis distinguished from mortgage:

In antichresis the fruits that are applied to .the interest and thereafter to the principal. In
mortgage the fruits are not applied to the principal obligation.

In antichresis, the creditor is in possession. In mortgage, the debtor is in possession.


The principal and interest must be in writing tor validity. In mortgage, registration is
required to bind third persons.

In antichresis, the creditor pays the taxes. In mortgage, taxes are not imposed on the
creditor.
RECOMMENDATION OF THE COMMITTEE:
If the above alternative answer is given, two (2) distinctions for each should be given full
credit.
Union Corporation was declared insolvent by order of the court. All creditors of
Union were asked to file their claims and attend a meeting to elect the assignee in
insolvency. Merchant Finance Corporation (MFC) has a claim for P500,000, which
is secured by a mortgage on a piece of land worth P1 M. MFC seeks your advice
as counsel whether it should participate in the foregoing proceedings.
What advice would you give MFC? (1987 Bar Question)

SUGGESTED ANSWER:
26

I would advise MFC that, having a contractual mortgage (the value of the mortgaged
property being well over the secured obligation), it should refrain from participating in
Page

the proceedings and instead pursue its preferential right to foreclose the mortgage.

CIVIL LAW REVIEW 2 – 4A BAR EXAMINATION Q & A (1980-2017)


A, as guarantor, executed a real estate mortgage in the amount of P50,000.00 to
secure payment of the indebtedness of XYZ Transit Co. for the purchase of two
GM trucks with a total value of P152,000.00. XYZ Transit Co. paid BMC Motors
Co., the seller of the trucks, the sum of P92,000.00, thus leaving a balance of
P60,000.00. The obligation guaranteed is further secured by a deed of chattel
mortgage on the trucks executed by XYZ Transit Co., in favor of BMC Motors Co.
To collect the balance of P60,000.00, BMC Motors Co. later filed an action against
XYZ Transit Co. with CFI, Manila to foreclose the chattel mortgage. The suit
resulted in the sale of the trucks at public auction in the amount of P50,000.00.
A, the real estate mortgagor, filed an action for the cancellation of the real estate
mortgage above-mentioned.
Will the action prosper? Give reasons. (1978 Bar Question)

SUGGESTED ANSWER:
Yes. A’s action for the cancellation of the real estate mortgage will prosper. The rule is
to the effect that the foreclosure of the chattel mortgage on the thing sold, bars further
recovery by the vendor of any unpaid balance of the price. Any agreement to the
contrary is void. A’s real estate mortgage is invalid, being a flagrant circumvention of the
prohibition of the law.
Include: Act 3135, as amended by R.A. No. 4118

DMP Corporation (DMP) obtained a loan of P20 M from National Bank (NB)
secured by a real estate mortgage over a 63,380-square meter land situated in
Cabanatuan City. Due to the Asian Economic Crisis, DMP experienced liquidity
problems disenabling it from paying its loan on time. For that reason, NB sought
the extrajudicial foreclosure of the said mortgage by filing a petition for sale on
June 30, 2003. On September 4, 2003, the mortgaged property was sold at public
auction, which was eventually awarded to NB as the highest bidder. That same
day, the Sheriff executed a Certificate of Sale in favor of NB.
On October 21, 2003, DMP filed a Petition for Rehabilitation before the RTC.
Pursuant to this, a Stay Order was issued by the RTC on October 27, 2003.
On the other hand, NB caused the recording of the Sheriff’s certificate of Sale on

December 3, 2003 with the Register of Deeds of Cabanatuan City. NB executed an


Affidavit of Consolidation of Ownership and had the same annotated on the title
of DMP.
Consequently, the Register of Deeds cancelled DMP’s title and issued a new title
in the name of NB on December 10, 2003.
NB also filed on March 17, 2004 an Ex-Parte Petition for Issuance of Writ of
Possession before the RTC of Cabanatuan City. After hearing, the RTC issued on
September 6, 2004 an Order directing the Issuance of the Writ of Possession,
which was issued on October 4, 2004.
DMP claims that all subsequent actions pertaining to the Cabanatuan property
should have been held in abeyance after the Stay Order was issued by the
rehabilitation court. Is DMP correct? (2014 Bar Question)
SUGGESTED ANSWER:
No. DMP is not correct. Since the foreclosure of the mortgage and the issuance of the
27

certificate of sale in favor of the mortgagee were done prior to the appointment of a
Rehabilitation Receiver and the issuance of the Stay Order, all the actions taken with
Page

CIVIL LAW REVIEW 2 – 4A BAR EXAMINATION Q & A (1980-2017)


respect to the foreclosed mortgaged property which were subsequent to the issuance of
the Stay Order were not affected by the Stay Order. Thus, after the redemption period
expired without the mortgagor redeeming the foreclosed property, the mortgagee
becomes the absolute owner of the property and it was within its right to ask for
consolidation of title and the issuance of new title in its favor. The writ of possession
procured by the mortgagee despite the subsequent issuance of Stay Order in the
rehabilitation proceeding instituted is also valid.
X defaulted in his loan with Y. Y instituted extra-judicial foreclosure of the
property subject to a real estate mortgage that secured the loan. X has 1 year
within which to redeem the property. After the foreclosure, X filed an action
questioning the validity of the extra-judicial foreclosure sale. Which statement is
most accurate? (2012 Bar Question)
A. The 1 year period within which to redeem will be interrupted by the filing of an
action questioning the validity of the foreclosure;
B. The 1 year period will not be interrupted by the filing of the action;
C. The 1 year period will be extended for another year because of the filing of
an action questioning the validity of the foreclosure sale;
D. If the action which questions the validity of the foreclosure prospers, the
period will be interrupted.
SUGGESTED ANSWER:
b. The 1 year period will not be interrupted by the filing of the action.
What is the effect if the proceeds in an extra-judicial foreclosure sale is not
sufficient to pay for the obligation? (2012 Bar Question)
A.The mortgagee can claim for deficiency judgment from the debtor;
B. The mortgagee can claim for deficiency judgment from the mortgagor even
though it is a third party mortgage;
C. The mortgagee has no more recourse or claim against the debtor;
D. The mortgagee cannot claim for deficiency judgment from the debtor
because it’s an extrajudicial foreclosure.
SUGGESTED ANSWER:

a) The mortgagee can claim for deficiency judgment from the debtor.

X mortgaged her residential house and lot in favor of ABC Bank. X defaulted in
her loan and so the bank foreclosed the real estate mortgage on the residential
house. Y then bought the residential house and lot before the expiration of the
redemption period. Can Y now take possession of the property? (2012 Bar
Question)
a. No, because it is still covered by the redemption period and the purchaser is
not yet entitled as a matter of right to take possession of the property;
b. Yes, the purchaser is now entitled to the possession of the house;
c. No, because there is a need to talk to X to leave the house;
d. No, because Y was not the one who foreclosed the mortgage on the property.

SUGGESTED ANSWER:

No, because it is still covered by the redemption period and the purchaser is not yet
entitled as a matter of right to take possession of the property
Which phrase best completes the statement—When a debt is secured by a real
28

estate mortgage, upon default of the debtor: (2012 Bar Question)


Page

CIVIL LAW REVIEW 2 – 4A BAR EXAMINATION Q & A (1980-2017)


A. The only remedy of the creditor is to foreclose the real estate mortgage;
B. Another remedy is filing an action for collection and then foreclose if
collection is not enough;
C. The creditor can foreclose the mortgage and demand collection for any
deficiency;
D. None of the above.
SUGGESTED ANSWER:

c) The creditor can foreclose the mortgage and demand collection for any deficiency.

X obtained a loan for P50 M from SSS Bank. The collateral is his vacation house
in Baguio City under a real estate mortgage. X needed more funds for his
business so he again borrowed another P10 M, this time from BBB Bank, another
bank, using the same collateral. The loan secured from SSS Bank fell due and X
defaulted.
If SSS Bank forecloses the real estate mortgage, what rights, if any, are left with
BBB Bank as mortgagee also?
If the value of the Baguio property is less than the amount of loan, what would be
the recourse of SSS bank? BBB Bank?
If the value of the property is more that the amount of the loan, who will benefit
from the excess value of the property?
If X defaulted with its loan in favor of BBB Bank but fully paid his loan with SSS
Bank, can BBB foreclose the real mortgage executed in its favor?
Does X have any legal remedy after the foreclosure in the event that later on he
has the money to pay for the loan?
If SSS Bank and BBB Bank abandoned their rights under the real estate
mortgage, is there any legal recourse available to them? (2012 Bar Question)
SUGGESTED ANSWER:

BBB Bank, as junior mortgagee, would have a right to redeem the foreclosed
property, together with X, his successors in interest, any judicial or judgment
creditor of X, or any other person or entity having a lien on the vacation house
subsequent to the real estate mortgage in favor of SSS Bank.
In case of a deficiency, SSS Bank could file suit to claim for the deficiency. BBB
Bank could file an ordinary action to collect its loan from X. if it does so, it would be
deemed to have waived it mortgage lien. If the judgment in the action to collect is
favorable to BBB Bank, and it becomes final and executor, BBB Bank could
enforce the said judgment by execution. It could even levy execution on the same
mortgaged property, but it would not have priority over the latter.
If the value of the property is more than the amount of the loan, the excess could
benefit and be claimed by BBB Bank, any judicial or judgment creditor of X, any
other junior mortgagee, and X.
If X defaulted in respect of his loan from BBB Bank but fully paid his loan from SSS
Bank, BBB Bank could now foreclose the mortgaged property as it would be the
only remaining mortgagee of the same.
Yes, X could redeem the property within 1 year from the date of registration of the
sheriff’s certificate of foreclosure sale.
SSS Bank and BBB Bank could each file an ordinary action to collect its loan from X.
On X’s failure to pay his loan to ABC Bank, the latter foreclosed the Real Estate
29

Mortgage he executed in its favor. The auction sale was set for Dec. 1, 2010 with
Page

the notices of sale published as the law required. The sale was, however,

CIVIL LAW REVIEW 2 – 4A BAR EXAMINATION Q & A (1980-2017)


cancelled when Dec. 1, 2010 was declared a holiday and rescheduled to Jan. 10,
2011 without republication of notice. The auction sale then proceeded on the new
date. Under the circumstance, the auction sale is (2011 Bar Question)
A. Rescissible.
B. Unenforceable.
C. Void.
D. Voidable.
SUGGESTED ANSWER:

c. Void.

Ozamis Paper Corporation secured loans from ABC Universal Bank in the
aggregate principal amount of P100 M, evidenced by several promissory notes,
and secured by a continuing guaranty of its principal stockholder Menandro
Marquez; a pledge of Marquez’s shares in the corporation valued at P45 M; and a
real estate mortgage over certain parcels of land owned by Marquez.
The corporation defaulted and the bank extra-judicially foreclosed on the real
estate mortgage. The bank, which was the sole bidder for P75 M, won the award.
Can the bank sue Marquez for the deficiency of P25 M? Explain. (2010 Bar
Question)
SUGGESTED ANSWER:
Yes, the bank can sue Marquez for the deficiency of P25 M. in extrajudicial foreclosure
of a real estate mortgage, if the proceeds of the sale are insufficient to pay the debt, the
mortgagee has the right to sue for the deficiency.
If the bank opts to file an action for collection against the corporation, can it
afterwards institute a real action to foreclose the mortgage? Explain. (2010 Bar
Question)
SUGGESTED ANSWER:
No, the bank can no longer file an action to foreclose the real estate mortgage. When it
filed a collection case, it was deemed to have abandoned the real estate mortgage.
Can the bank foreclose on the pledged shares of Marquez and recover the
deficiency from the corporation? (2010 Bar Question)
SUGGESTED ANSWER:
If the bank forecloses the pledge, it cannot recover the deficiency because the
foreclosure extinguishes the principal obligation, whether or not the proceeds from the
foreclosure are equal to the amount of the principal obligation.
On December 4, 2003, RED Corporation executed a real estate mortgage in favor
of BLUE Bank. RED Corporation defaulted in the payment of its loan.
Consequently, on June 4, 2004, BLUE Bank extrajudicially foreclosed the
property. Being the highest bidder in the auction sale conducted, the Bank was
issued a Certificate of Sale which was registered on August 4, 2004.
Does RED Corporation still have the right to redeem the property as of September
14, 2007? Reason briefly. (2007 Bar Question)
SUGGESTED ANSWER:
No, RED Corporation has lost its right to redeem the property. Juridical persons whose
property is sold pursuant to an extrajudicial foreclosure, shall have the right to redeem
30

the property until registration of the certificate of sale with the Register of Deeds, which
Page

shall in no case be more than 3 months after foreclosure, whichever is earlier.

CIVIL LAW REVIEW 2 – 4A BAR EXAMINATION Q & A (1980-2017)


A real estate mortgage may be foreclosed judicially or extrajudicially. In what
instance may a mortgage extrajudicially foreclose a real estate mortgage? (2006
Bar Question)

SUGGESTED ANSWER:
A mortgage may extrajudicially foreclose a real estate mortgage when the right to
foreclose extrajudicially has been expressly stipulated in the deed of mortgage or there
is a special power in the real estate mortgage authorizing it.
Primetime Corporation (the Borrower) obtained a P10 M, 5-year term loan from
Universal Bank (the Bank) in 1996. As security for the loan and as required by the
Bank, the Borrower gave the following collateral security in favor of the Bank:
A real estate mortgage over the land and building owned by the Borrower and
located in Quezon City;
The joint and several promissory note of Mr. Primo Timbol, the President of
the Borrower; and
A real estate mortgage over the residential house and lot owned by Mr.
Timbol, also located in Quezon City
Because of business reverses, neither the Borrower nor Mr. Timbol was able to
pay the loan. In June 2001, the Bank extrajudicially foreclosed the two real estate
mortgages, with the Bank as the only bidder in the foreclosure sale. On
September 16, 2001, the certificates of sale of the two properties in favor of the
Bank were registered with the Register of Deeds of Quezon City.
10 months later, both the Borrower and Mr. Timbol were able to raise sufficient
funds to redeem their respective properties from the Bank, but the Bank refused
to permit redemption on the ground that the period for redemption had already
expired, so that the Bank now has absolute ownership of both properties. The
Borrower and Mr. Timbol came to you today, September 15, 2002, to find out if the
position of the Bank is correct. What would be your answer? State your reasons.
(2002 Bar Question)
SUGGESTED ANSWER:
With respect to the real estate mortgage over the land and building owned by the
Borrower, Primetime Corporation, a juridical body, the period of redemption is only 3
months, which period already expired.
As to the real estate mortgage over the residential house and lot owned by Mr. Timbol,
the period of redemption is 1 year from the date of registration of the sale, which period
has not yet expired in this case.
Debtor ―A‖ issued a promissory note in the amount of P10 M in favor of a
commercial bank ―Y‖ secured by mortgage of his properties worth P30 M. When
―A‖ failed to pay his indebtedness, despite demands made by bank ―Y‖, the latter
instituted a collection suit to enforce payment of the P10 M account.
Subsequently, bank ―Y‖ also filed foreclosure proceedings against ―A‖ for the
security given for the account. If you were the judge, how would you resolve the
two cases? (2001 Bar Question)
SUGGESTED ANSWER:
The case for collection will be allowed to proceed. But the foreclosure proceedings have
to be dismissed. In instituting a foreclosure proceedings, after filing a collection case
involving the same account or transaction, bank ―Y‖ is guilty of splitting a cause of
action. The loan of P10 M is the principal obligation while the mortgage securing the
31

same is merely an accessory to said loan obligation. The collection of the loan and the
Page

foreclosure of the mortgage securing said loan constitute one and the same cause of

CIVIL LAW REVIEW 2 – 4A BAR EXAMINATION Q & A (1980-2017)


action. The filing of the collection case bars the subsequent filing of the foreclosure
proceedings.
Are the right of redemption and the equity of redemption given by law to a
mortgagor the same? Explain. (2%) (1999 Bar Question)
SUGGESTED ANSWER:
The equity of redemption is different from the right of redemption. Equity of redemption
is the right of the mortgagor after judgment in a judicial foreclosure to redeem the
property by paying to the court the amount of the judgment debt before the sale or
confirmation of the sale. On the other hand, right of redemption is the right of the
mortgagor to redeem the property sold at an extra-judicial foreclosure by paying to the
buyer in the foreclosure sale the amount paid by the buyer within one year from such
sale.
―X‖ mortgaged his land to the Philippine National Bank (PNB) to secure a
promissory note. He defaulted in the payment of the loan so that the land was
sold at public auction on January 20, 1960, for P3,500 with the PNB as the highest
bidder. On January 20, 1970,
―X‖ offered to redeem the property in the amount of P3,500. He enclosed a postal
money order for PI,000 as partial payment and stated that the balance is to be
paid in 12 monthly installments. The PNB then discovered that the sheriffs
certificate of sale prepared after the public auction of the land was not registered
so that it cause the same to be registered on January 30, 1970. The PNB refused
the offer of ―X‖ contending that the offer to redeem was beyond the one-year
period provided under Act No. 3135 and that it was not accompanied by an actual
and simultaneous tender of the entire repurchase price. In view of the refusal of
the PNB, ―X‖ filed an action to repurchase on
February 20, 1970. Will the action prosper? Give your reasons. (1989 Bar Question)
SUGGESTED ANSWER:
Yes, the action should prosper. The one (1) year period of redemption is counted from
the registration of the sheriff’s certificate of sale hence the action has not yet prescribed.
However, there need not be a tender of the redemption price because the filing of the
judicial action to enforce the right of redemption within the redemption period suffices.
ABC Corporation has been experiencing liquidity problems. Anticipating that it
would be unable to pay maturing obligations, it filed with the SEC a petition for
suspension of payments with a prayer for the appointment of a rehabilitation
receiver.
During the pendency of the case, ABC Corporation defaulted in the payment of its
debt to XYZ Corporation, a mortgage creditor. Thereupon, XYZ Corporation
initiated proceedings for the extrajudicial foreclosure of the mortgaged property.
If you were counsel for ABC Corporation, what argument would you raise to
resist the foreclosure? If you were the hearing officer of the SEC, would you
permit the foreclosure? Why? (1984 Bar Question)
SUGGESTED ANSWER:
Counsel for ABC Corp. should contend that by virtue of the appointment of a
rehabilitation receiver by the SEC the proceedings for the extrajudicial foreclosure of the
mortgage should not be permitted since PD 902-A, as amended, provides that ―all
actions for claims against‖ ABC Corp. ―pending before any court, tribunal, board or body
shall be suspended accordingly.‖ It is the intent of the law to cover not only those
32

actions actually pending at the time of the appointment of the receiver but those that
may be brought subsequent thereto, so long as the distressed corporation remain under
Page

receivership, in order to prevent the dissipation of the assets of the corporation.

CIVIL LAW REVIEW 2 – 4A BAR EXAMINATION Q & A (1980-2017)


As the hearing officer of the SEC, I would permit the foreclosure for the following
reasons:
The extrajudicial foreclosure proceedings does not constitute an ―action‖ within the
meaning of the Rules of Court; 2) Proceedings for the foreclosure of a mortgage is not
an ―action for claims against‖ ABC Corp., but is against the property subject of the
mortgage; 3) Applying the law literally, the foreclosure proceedings were not ―pending‖
at the time of the appointment of the rehabilitation receiver. The default by ABC Corp.
and the consequent initiation of the foreclosure proceedings took place after the
appointment of the rehabilitation receiver; 4) The extrajudicial foreclosure of the
mortgage is not one ―pending before any court, tribunal, board or body: and 5) To
prevent the extrajudicial proceedings for foreclosure may result in a violation of the
constitutional prohibition against impairment of the obligations of contracts.

VI. Antichresis

Multiple choice: Choose the right answer. (2% each)

A contract of antichresis is always:

a. a written contract;
b. a contract with a stipulation that the debt will be paid through receipt of
the fruits of an immovable;
c. involves the payment of interests, if owing;
d. all of the above;
e. letters a and b.
SUGGESTED ANSWER:

d (all of the above)

Olivia owns a vast mango plantation which she can no longer properly manage
due to a lingering illness. Since she is indebted to Peter in the amount of
P500,000.00, she asks Peter to manage the plantation and apply the harvest to the
payment of her obligation to him, principal and interest, until her Indebtedness
shall have been fully paid. Peter agrees.
What kind of contract is entered into between Olivia and Peter? Explain.
What specific obligations are imposed by law on Peter as a consequence
of their contract?
Does the law require any specific form for the validity of their contract?
Explain
May Olivia re-Require the plantation before her entire indebtedness shall
have been fully paid? Explain. (1994 Bar Question)
SUGGESTED ANSWER:
a. A contract of antichresis was entered into between Olivia and Peter. Under Article
2132 of the New Civil Code, by a contract of antichresis the creditor acquires the
right to receive the fruits of an immovable of his debtor, with the obligation to apply
them to the payment of the interest, and thereafter to the principal of his credit.
b. Peter must pay taxes and charges upon the land and bear the necessary
expenses for preservation and repair which he may deduct from the fruits. (Art.
2135, NCC)
The amount of the principal and interest must be specified in writing, otherwise the
antichresis will be void. (Art. 2134, NCC)
33

No. Art. 2136 specifically provides that the debtor cannot re-acquire the enjoyment
of the immovable without first having totally paid what he owes the creditor.
Page

However, it is potestative on the part of the creditor to do so in order to exempt him

CIVIL LAW REVIEW 2 – 4A BAR EXAMINATION Q & A (1980-2017)


from his obligation under Art. 2135, NCC. The debtor cannot re-acquire the
enjoyment unless Peter compels Olivia to enter again the enjoyment of the
property
What do you understand by ANTICHRESIS? How is it distinguished from pledge
and mortgage? (1989 Bar Question)
SUGGESTED ANSWER:
Antichresis is a contract whereby the creditor acquires the right to receive the fruits of
an immovable of his debtor with the obligation to apply them to the payment of interest if
owing and thereafter to the principal.
Pledge is an accessory and real contract whereby the debtor delivers to the creditor
movable property as security for the performance of a principal obligation upon the
fulfilment of which the thing pledged shall be returned to the debtor.
A real estate mortgage is an accessory contract whereby the debtor guarantees the
performance of the principal obligation by subjecting real property or real right as
security for the performance of such obligation.
ALTERNATIVE EXTENDED ANSWER:
By the contract of antichresis the creditor acquires the right to receive the fruits .of an
immovable of his debtor, with the obligation to apply them to the payment of the interest,
if owing* and thereafter to the principal of his credit.
Antichresis distinguished from pledge:

Antichresis is consensual, pledge is a real contract.


Antichresis involves real property, pledge involves personal property.
In antichresis, the principal and the interest must be provided in writing for
validity. In pledge, the date and description of the pledge must be in a public
instrument to affect third persons.
Antichresis distinguished from mortgage:

In antichresis the fruits that are applied to .the interest and thereafter to the
principal. In mortgage the fruits are not applied to the principal obligation.
In antichresis, the creditor is in possession. In mortgage, the debtor is in
possession.
The principal and interest must be in writing tor validity. In mortgage,
registration is required to bind third persons.
In antichresis, the creditor pays the taxes. In mortgage, taxes are not imposed
on the creditor.

RECOMMENDATION OF THE COMMITTEE:

If the above alternative answer is given, two (2) distinctions for each should be given full
credit.
VII. Chattel Mortgage

Which phrase best completes the statement—A chattel mortgage can be


constituted to secure: (2012 Bar Question)
A. Obligation both past and future;
B. Obligation existing at the time the mortgage is constituted;
34

C. Future obligations only;


Page

D. Past obligations only.

CIVIL LAW REVIEW 2 – 4A BAR EXAMINATION Q & A (1980-2017)


SUGGESTED ANSWER:

b. Obligation existing at the time the mortgage is constituted.

X constituted a chattel mortgage on a car (valued at P1 M) to secure a P500,000


loan. For the mortgage to be valid, X should have (2011 Bar Question)
A. The right to mortgage the car to the extent of half its value.
B. Ownership of the car.
C. Unqualified free disposal of his car.
D. Registered the car in his name.
SUGGESTED ANSWER:

c. Unqualified free disposal of his car.

X constructed a house on a lot which he was leasing from Y. Later, X executed a


chattel mortgage over said house in favor of Z as security for a loan obtained
from the latter. Still later, X acquired ownership of the land where his house was
constructed, after which he mortgaged both house and land in favor of a bank,
which mortgage was annotated on the Torrens Certificate of Title. When X failed
to pay his loan to the bank, the latter, being the highest bidder at the foreclosure
sale, foreclosed the mortgage and acquired X’s house and lot. Learning of the
proceedings conducted by the bank, Z is now demanding that the bank reconvey
to him X’s house or pay X’s loan to him plus interests. Is Z’s demand against the
bank valid and sustainable? Why? (2003 Bar Question)
SUGGESTED ANSWER:
No, Z’s demand is not valid. A building is immovable or real property whether it is
erected by the owner of the land, by a usufructuary, or by a lessee. It may be treated as
a movable by the parties to a chattel mortgage but such is binding only between them
and not on third parties (Evangelista v. Alto Surety Co., Inc., 103 Phil. 401 [1958]). In
this case, since the bank is not a party to the chattel mortgage, it is not bound by it. As
far as the Bank is concerned, the chattel mortgage does not exist. Moreover, the chattel
mortgage is void because it was not registered. Assuming that it is valid, it does not bind
the Bank because it was not annotated on the title of the land mortgaged to the bank. Z
cannot demand that the Bank pay him the loan Z extended to X, because the Bank was
not privy to such loan transaction.
ANOTHER SUGGESTED ANSWER:
No, Z’s demand against the bank is not valid. His demand that the bank reconvey to him
X’s house presupposes that he has a real right over the house. All that Z has is a
personal right against X for damages for breach of the contract of loan.
The treatment of a house, even if built on rented land, as movable property is void
insofar as third persons, such as the bank, are concerned. On the other hand, the Bank
already had a real right over the house and lot when the mortgage was annotated at the
back of the Torrens title. The bank later became the owner in the foreclosure sale.
Z cannot ask the bank to pay for X’s loan plus interest. There is no privity of contract
between Z and the bank.
ALTERNATIVE ANSWER:
The answer hinges on whether or not the bank is an innocent mortgagee in good faith
or a mortgagee in bad faith. In the former case, Z’s demand is not valid. In the latter
case, Z’s demand against the bank is valid and sustainable.
35

Under the Torrens system of land registration, every person dealing with registered land
may rely on the correctness of the certificate of title and the law will not in any way
Page

oblige him to look behind or beyond the certificate in order to determine the condition of

CIVIL LAW REVIEW 2 – 4A BAR EXAMINATION Q & A (1980-2017)


the title. He is not bound by anything not annotated or reflected in the certificate. If he
proceeds to buy the land or accept it as a collateral relying on the certificate, he is
considered a buyer ora mortgagee in good faith. On this ground, the Bank acquires a
clean title to the land and the house.
However, a bank is not an ordinary mortgagee. Unlike private individuals, a bank is
expected to exercise greater care and prudence in its dealings. The ascertainment of
the condition of a property offered as collateral for a loan must be a standard and
indispensable part of its operation. The bank should have conducted further inquiry
regarding the house standing on the land considering that it was already standing there
before X acquired title to the land. The bank cannot be considered as a mortgagee in
good faith. On this ground, Z's demand against the Bank is valid and sustainable.

Distinguish a contract of chattel mortgage from a contract of pledge. (2%) (1999


Bar Question)
SUGGESTED ANSWER:
In a contract of chattel mortgage possession belongs to the creditor, while in a contract
of pledge possession belongs to the debtor.
A chattel mortgage is a formal contract while a pledge is a real contract.

A contract of chattel mortgage must be recorded in a public instrument to bind third


persons while a contract of pledge must be in a public instrument containing description
of the thing pledged and the date thereof to bind third persons.
Debtor purchased a parcel of land from a realty company payable in 5 yearly
installments. Under the contract of sale, title to the lot would be transferred upon
full payment of the purchase price.
But even before full payment, debtor constructed a house on the lot. Sometime
thereafter, debtor mortgaged the house to secure his obligation arising from the
issuance of a bond needed in the conduct of his business. The mortgage was
duly registered with the proper chattel mortgage registry.
5 years later after completing payment of the purchase price, debtor obtained title
to the lot. And even as the chattel mortgage on the house was still subsisting,
debtor mortgaged to a bank the lot and improvement thereon to secure a loan.
This real estate mortgage was duly registered and annotated at the bank of the
title.
Due to business reverses, debtor failed to pay his creditors. The chattel mortgage
was foreclosed when the debtor failed to reimburse the surety company for
payments made on the bond. In the foreclosure sale, the surety company was
awarded the house as the highest bidder.
Only after the foreclosure sale did the surety company learn of the real estate
mortgage in favor of the lending investor on the lot and the improvement thereon.
Immediately, it filed a complaint praying for the exclusion of the house from the
real estate mortgage. It was submitted that as the chattel mortgage was executed
and registered ahead, it was superior to the real estate mortgage.
On the suggestion that a chattel mortgage on a house—a real property—was a
nullity, the surety company countered that when the chattel mortgage was
executed, debtor was not yet the owner of the lot on which the house was built.
36

Accordingly, the house was a personal property and a proper subject of a


chattel mortgage. Discuss the validity of the position taken by the surety
Page

company.

CIVIL LAW REVIEW 2 – 4A BAR EXAMINATION Q & A (1980-2017)


Who has a better claim to the house, the surety company or the lending
investor? Explain.
Would the position of the surety company be bolstered by the fact that it
acquired title in a foreclosure sale conducted by the Provincial Sheriff. Explain.
(1999 Bar Question)
SUGGESTED ANSWER:

The house is always a real property even though it was constructed on a land not
belonging to the builder. However, the parties may treat it as personal property
and constitute a chattel mortgage thereon. Such mortgage shall be valid and
binding but only on the parties. It will not bind or affect third parties.
The lending investor has a better claim to the house. The real estate mortgage
covering the house and lot was duly registered and binds the parties and third
persons. On the other hand, the chattel mortgage on the house securing the credit
of the surety company did not affect the rights of third parties such as the lending
investor despite registration of the chattel mortgage.
No. The chattel mortgage over the house which was foreclosed did not affect the
rights of third parties like the lending investor. Since third parties are not bound by
the chattel mortgage, they are not also bound by any enforcement of its provisions.
The foreclosure of such chattel mortgage did not bolster or add anything to the
position of the surety company.
Ritz bought a new car on installments which provided for an acceleration clause
in the event of default. To secure payment of the unpaid installment, as and when
due, he constituted 2 chattel mortgages. i.e., one over his very old car and the
other covering the new car that he had just bought, as aforesaid, on installment.
After Ritz defaulted on 3 installments, the seller-mortgagee foreclosed on the old
car. The proceeds of the foreclosure were not enough to satisfy the due
obligation; hence, he similarly sought to foreclose on the new car. Would the
seller-mortgagee be legally justified in foreclosing on this second chattel
mortgage? (1997 Bar Question)
SUGGESTED ANSWER:
No. the 2 mortgages were executed to secure the payment of the unpaid installments
for the purchase of a new car. When the mortgage on the old car was foreclosed, the
seller-mortgagee is deemed to have renounced all other rights. A foreclosure of
additional property, that is, the new car covered by the second mortgage would be a
nullity.
Finding a 24-month payment plan attractive, Anjo purchased a Tamaraw FX from
Toyota Quezon City. He paid a down payment of P100,000, and obtained financing
for the balance from IOU Company. He executed a chattel mortgages over the
vehicle in favor of IOU. When Anjo defaulted, IOU foreclosed the chattel
mortgage, and sought to recover the deficiency.
May IOU still recover the deficiency? Explain. (1996 Bar Question)

SUGGESTED ANSWER:
IOU may no longer recover the deficiency. Under Article 1484 of the Civil Code, in a
contract of sale of personal property the price of which is payable in installments, the
vendor may, among several options, foreclose the chattel mortgage on the thing sold, if
one has been constituted, should the vendee’s failure to pay cover 2 or more
installments. In such case, however, the vendor shall have no further action against the
37

purchaser to recover any unpaid balance of the price and any agreement to the contrary
is void. While the given facts did not explicitly state that Anjo’s failure to pay covered 2
Page

CIVIL LAW REVIEW 2 – 4A BAR EXAMINATION Q & A (1980-2017)


or more installments, this may safely be presumed because the right of IOU to foreclose
the chattel mortgage under the circumstances is premised on Anjo’s failure to pay 2 or
more installment. The foreclosure would not have been valid if it were not so.
Lawrence, a retired air force captain, decided to go into the air transport
business. He purchased an aircraft in cash except for an outstanding balance of
P500,000.00. He incurred an indebtedness of P300.000.00 for repairs with an
aircraft repair company.
He also borrowed PI Million from a bank for additional capital and constituted a
chattel mortgage on the aircraft to secure the loan.
While on a test flight the aircraft crashed causing physical injuries to a third party
who was awarded damages of P200.000.00.
Lawrence’s insurance claim for damage to the aircraft was denied thus leaving
him nothing else but the aircraft which was then valued only at PI Million.
Lawrence was declared insolvent.
Assuming that the aircraft was sold for PI Million, give the order of preference of
the creditors of Lawrence and distribute the amount of PI Million. (1995 Bar
Question)
SUGGESTED ANSWER:
Assuming that the aircraft was sold for P1 Million, there is no order of preference. The
P1 Million will all go to the bank as a chattel mortgagee because a chattel mortgage
under Art. 2241 (4) NCC, defeats Art. 2244 (12) and (14). Art. 2241 (3) and (5) are not
applicable because the aircraft is no longer in the possession of the creditor.

A, about to leave the country on a foreign assignment, entrusted to B his brand


new car and its certificate of registration. Falsifying A's signature. B sold A's car
to C for P200.000.00. C then registered the car in his name. To complete the
needed amount, C borrowed P 100,000.00 from the savings and loan association
in his office, constituting a chattel mortgage on the car. For failure of C to pay the
amount owed, the savings and loan association filed in the RTC a complaint for
collection with application for issuance of a writ of replevin to obtain possession
of the vehicle so that the chattel mortgage could be foreclosed. The RTC issued
the writ of replevin. The car was then seized from C and sold by the sheriff at
public auction at which the savings and loan association was the lone bidder.
Accordingly, the car was sold to it. A few days later. A arrived from his foreign
assignment. Learning of what happened to his car, A sought to recover pos-
session and ownership of it from the savings and loan association.
Can A recover his car from the savings and loan association? Explain your
answer. (1993 Bar Question)
SUGGESTED ANSWER:
Under the prevailing rulings of the Supreme Court, A can recover the car from the
Savings and Loan Association provided he pays the price at which the Association
bought the car at a public auction. Under that doctrine, there has been an unlawful
deprivation by B of A of his car and. therefore, A can recover It from any person in
possession thereof.. But since it was bought at a public auction in good faith by the
Savings and Loan Association, he must reimburse the Association at the price for which
the car was bought.
ALTERNATIVE ANSWER:
38

Yes. A can recover his car from the Savings and Loan Association. In a Chattel
Mortgage, the mortgagor must be the absolute owner of the thing mortgaged.
Page

Furthermore, the person constituting the mortgage must have the free disposal of the

CIVIL LAW REVIEW 2 – 4A BAR EXAMINATION Q & A (1980-2017)


property, and in the absence thereof, must be legally authorized for the purpose. In the
case at bar, these essential requisites did not apply to the mortgagor B. hence the
Chattel Mortgage was not valid.
To secure the payment of an earlier loan of P20,000, as well as subsequent loans
which her friend, Noreen, would extend to her, Karen executed in favor of Noreen
a chattel mortgage over her (Karen) care.
Is the mortgage valid? (1991 Bar Question)

SUGGESTED ANSWER:
A chattel mortgage cannot effectively secure after-incurred obligations. While a
stipulation to include after -incurred obligations in a chattel mortgage is itself not invalid,
the obligation cannot, however, be deemed automatically secured by that mortgage until
after a new chattel mortgage or an addendum to the original chattel mortgage is
executed to cover the obligation after it has been actually incurred. Accordingly, unless
such supplements are made, the chattel mortgage in the problem given would be
deemed to secure only the loan of P20,000.
Eastern Motors, Inc. (EMI), an automotive dealer, sold a Toyota station wagon to
Alran Tuason, payable in 10 monthly installments. The installments were
evidenced by a promissory note and secured by a mortgage on the car. EMI
assigned the credit to Island Finance Corporation (IFC), subject to IFC’s right of
recourse to EMI if the car buyer (Tuason) was unable to pay the credit in full.
Upon Tuason’s default, IFC foreclosed on the mortgage. Since a deficiency
remained, IFC sought to collect the same from EMI.
Is IFC justified in doing so? Reason out your answer. (1987 Bar Question)

SUGGESTED ANSWER:
IFC is not justified in collecting the deficiency from EMI. An assignee is merely a
successor -in-interest of the assignor and, therefore, unless otherwise expressed in the
deed of assignment, the right of recourse stipulated in favor of IFC must be deemed
confined only to a case where the car buyer is unable to pay the credit in full. By
foreclosing on the car, the right to the deficiency is lost and no further amount is thus
due from the car buyer.
Benedicto executed a chattel mortagage on a Mercedes-Benz car in favor of
Silverio. The mortgage was duly registered on August 15. Upon the failure of
Benedicto to pay the obligation secured by the chattel mortgage, Silverio filed, on
October 3, an action for replevin to take possession of the mortgaged car. It
turned out that as early as August 20, Leopoldo had already filed an action to
recover a sum of money against Benedicto. Even before the repelvin case of
Silverio could be set for trial, Leopoldo caused a levy to be made on the Mercede-
Benz to satisfy the money judgment which a court had awarded on October 10
against Benedicto in favor of Leopoldo.
Whose claim to the Mercedes-Benz car will prevail, Leopoldo’s or Silverio’s?
explain. (1986 Bar Question)
SUGGESTED ANSWER:
Silverio’s claim will prevail. His mortgage was duly registered on August 15 or days
before Leopoldo filed his action and months before the judgment levy by him was made.
The time when Silverio sought to enforce the lien is not material; the date of registration
of the chattel mortgage is enough to bind, or make it effective as against, third persons.
Juan Royo is constructing his family home on a lot in Marikina which he leased
39

from Fernando Paz. Short of funds to finish the house, Juan borrowed from
Page

CIVIL LAW REVIEW 2 – 4A BAR EXAMINATION Q & A (1980-2017)


Traders Royal Bank the sum of P150,000. By mutual agreement, Juan Royo
executed a chattel mortgage over the residential house in favor of the bank.
Is a chattel mortgage over a residential house constructed on a rented land
belonging to another person valid and enforceable? Explain. (1985 Bar Question)
SUGGESTED ANSWER:
It depends. The chattel mortgage over a residential house constructed on a rented land
belonging to another person, may be valid and enforceable, or not. If between only the
contracting parties, mortgagor and mortgagee, the chattel mortgage of said building is
valid and enforceable, since the building, though a real property, has been considered a
chattel between the parties, and the validity of the contract between them has been
recognized principally upon the principle of estoppels. However, with respect to third
persons, who are not parties to said contract of chattel mortgage, the building is an
immovable property, and the chattel mortgage on the same is, to said third persons, a
complete nullity.
To secure a loan or P100,000, Mr. Pons executed in favor of Mr. Sy a chattel
mortgage on his house built on a leased property belonging to another. They
agreed to consider said house as a personal property for purposes of the
mortgage. In the ensuing foreclosure sale, the house was sold by the sheriff to
Mr. Sy, who in turn sold it to Mr. Roque. Subsequently, Mr. Manapla filed a
complaint and obtained judgment against Mr. Pons for the sum of P120,000. The
same house, formerly owned by Mr. Pons but, as above stated, acquired by Mr.
Sy and sold to Mr. Roque, was levied upon by Mr. Manapla. To prevent the sale at
public auction, Mr. Roque and Mr. Sy filed an action against Mr. Pons and Mr.
Manapla. Will such action prosper? (1982 Bar Question)
SUGGESTED ANSWER:
The action by Mr. Sy and Mr. Roque against Mr. Pons and Mr. Manapla will not prosper,
since plaintiffs have no cause of action against the defendants herein. Regardless of the
validity of a contract constituting a chattel mortgage on a house which is a real property,
as between the parties to said contract, the same cannot and does not bind third
persons, who are not parties to the aforementioned contract or their privies. As a
consequence, the sale of the house in question in the proceedings for extrajudicial
foreclosure or said chattel mortgage, is null and void in so far as Mr. Manapla is
concerned, and did not confer upon Mr. Sy, as buyer in said sale, any dominical right in
and to said house, so that he had not transmitted to his assignee, plaintiff Mr. Roque
any such right as against defendant Mr. Manapla. (Piansay, et al. v. David , et al., Oct.
30, 1964; 12 SCRA 228)

―W‖ constructs a residential house on a rented land belonging to ―X‖. To enable


him to finish the house, ―W‖ borrows P50,000.00 from Jet Savings bank and
mortgages his house as security.
By mutual agreement, ―W‖ executes a chattel mortgage on the residential house
in favor of the bank.
Is the chattel mortgage on ―W’s‖ residential house situated on a rented land
belonging to ―X‖ valid and enforceable? (1980 Bar Question)
SUGGESTED ANSWER:
The validity of the chattel mortgage on W’s residential house situated on a rented land
belonging to X may be recognized between the contracting parties, principally upon the
principle of estoppels; thus, the validity of the chattel mortgage in question cannot be
assailed by one of the parties to the contract of mortgage, principally on the doctrine of
40

estoppels. (Navarro v. Pineda, Nov. 30, 1963; 9 SCRA 636)


Page

CIVIL LAW REVIEW 2 – 4A BAR EXAMINATION Q & A (1980-2017)


Creditor A filed action for recovery of a sum of money against debtor B, and
secured a preliminary attachment on a personal property of B. Subsequently, B
executed a deed of chattel mortgage over the same property in favor of C, who
filed a third party claim over the property attached. A now files a motion for
disapproval of the third party claim. C opposes the motion on the ground that the
chattel mortgage being in the nature of a conditional sale, title passed to him in
the meantime and therefore he is entitled to possession of the property. Decide
the motion and opposition. (1979 Bar Question)
SUGGESTED ANSWER:
A’s motion for disapproval of C’s third party claim may be sustained, because C was
merely a chattel mortgagee. Chattel mortgage is merely a security for a loan and does
not transfer title of the property mortgaged to the chattel mortgagee. (Serra v.
Rodriguez, L-25546, April 22, 1974; 56 SCRA 538).
C’s allegation that the chattel mortgage being in the nature of a conditional sale, cannot
be sustained. The old view that a chattel mortgage is a conditional sale has been
expressly repudiated by the new Civil Code. (Art. 2140, Civil Code; Serra v. Rodriguez,
supra)
To guarantee the payment of his obligation, the defendant A mortgaged to the
plaintiff B his sugar, then stored in a warehouse in San Fernando, Pampanga
authorizing said plaintiff (B) to sell the sugar in case he (A) failed to pay. During
the initial days of martial rule in late 1972, all of A’s sugar were burned or looted
in the warehouse. Plaintiff B sued defendant A for payment of the obligation.
1. Will the suit prosper? Explain.

Who shall bear the loss of the mortgaged sugar? Give reasons. (1978 Bar
Question)
SUGGESTED ANSWER:
Yes. B’s suit will prosper. The mortgagee, B, after the loss of the sugar in a warehouse,
may still recover on the obligation of the mortgagor, as an ordinary creditor, he having
lost already his security. (Martinez v. Philippine National Bank, L-4080, Sept. 21, 1953)
Mr. A, the mortgagor, shall bear the loss of the mortgaged sugar. The mortgagee, not
being the owner of the mortgaged sugar, does not suffer the loss. Said goods are to be
regarded as lost on account of the real owner, the mortgagor. (Martinez v. Philippine
National Bank, L-4080, Sept. 21, 1953)
A, as guarantor, executed a real estate mortgage in the amount of P50,000.00 to
secure payment of the indebtedness of XYZ Transit Co. for the purchase of two
GM trucks with a total value of P152,000.00. XYZ Transit Co. paid BMC Motors
Co., the seller of the trucks, the sum of P92,000.00, thus leaving a balance of
P60,000.00. The obligation guaranteed is further secured by a deed of chattel
mortgage on the trucks executed by XYZ Transit Co., in favor of BMC Motors Co.
To collect the balance of P60,000.00, BMC Motors Co. later filed an action against
XYZ Transit Co. with CFI, Manila to foreclose the chattel mortgage. The suit
resulted in the sale of the trucks at public auction in the amount of P50,000.00.
A, the real estate mortgagor, filed an action for the cancellation of the real estate
mortgage above-mentioned.
Will the action prosper? Give reasons. (1978 Bar Question)
41
Page

CIVIL LAW REVIEW 2 – 4A BAR EXAMINATION Q & A (1980-2017)


SUGGESTED ANSWER:
Yes, A’s action for the cancellation of the real estate mortgage will prosper. The rule is
to the effect that the foreclosure of the chattel mortgage on the thing sold, bars further
recovery by the vendor of any unpaid balance of the price. Any agreement to the
contrary is void. A’s real estate mortgage is invalid, being a flagrant circumvention of the
prohibition of the law.
Include: Act 1508

Which phrase best completes the statement—The Deed of Chattel mortgage, if


not registered with the Register of Deeds where debtor resides: (2012 Bar
Question)

a. Is not valid, hence not binding between the mortgagor and the mortgagee;
b. Is binding between the mortgagor and the mortgagee but will not affect
third party;
c.To be valid between the mortgagor and the mortgagee, it must be
coupled with the delivery of the subject matter of the chattel mortgage;
d. Is as if a non-existent chattel mortgage.
SUGGESTED ANSWER:

b. Is binding between the mortgagor and the mortgagee but will not affect third party.

Which phrase best completes the statement—A chattel mortgage can cover:
(2012 Bar Question)
A. Only property described in the deed without exception;
B. Can also cover substituted property;
C. Properties described in the deed except in case of stock in trade being
a substitute;
D. After acquired property.
SUGGESTED ANSWER:

c. Properties described in the deed except in case of stock in trade being a substitute.

Which phrase best completes the statement—To bind third parties, a chattel
mortgage of shares of stock must be registered: (2012 Bar Question)
A. With the Register of Deeds where the debtor resides;
B. With the Register of Deeds where the principal office of the corporation is;
C. In the Stock and Transfer Book of the corporation with the Corporate
Secretary;
D. With the Register of Deeds where the debtor resides and the principal office
of the corporation.
SUGGESTED ANSWER:

With the Register of Deeds where the debtor resides and the principal office of the
corporation.
Which phrase best completes the statement—The affidavit of good faith in a Deed
of Chattel Mortgage is: (2012 Bar Question)
a. An oath where the parties swear that the mortgage is made for the purpose
42

of securing the obligations specified and that the obligation is just and valid;
Page

CIVIL LAW REVIEW 2 – 4A BAR EXAMINATION Q & A (1980-2017)


b. An affidavit, the absence of which will vitiate the mortgage between the
parties;
c. Necessary only if the chattel being mortgaged are growing crops;
d. A certification from the mortgagor that he is the mortgagor of the chattel.
SUGGESTED ANSWER:
An oath where the parties swear that the mortgage is made for the purpose of securing
the obligations specified and that the obligation is just and valid.
Armando, a resident of Manila, borrowed P3 M from Bernardo, offering as
security his 500 shares of stock worth P1.5 M in Xerxes Corporation, and his 2007
BMW sedan, valued at P2 M. the mortgage on the shares of stock was registered
in the Office of the Register of Deeds of Makati City where Xerxes Corporation
has its principal office. The mortgage on the car was registered in the Office of
the Register of Deeds of Manila. Armando executed a single Affidavit of Good
Faith, covering both mortgages.
Armando defaulted on the payment of his obligation; thus, Bernardo foreclosed
on the two chattel mortgages. Armando filed suit to nullify the foreclosure and
the mortgages, raising the following issues:
The execution of only one Affidavit of Good Faith for both mortgages invalidated
the two mortgages; and (2009 Bar Question)
SUGGESTED ANSWER:
The execution of only one Affidavit of Good Faith for both mortgages is not a ground to
nullify the said mortgages and the foreclosure thereof. Said mortgages are valid as
between immediate parties, although they cannot bind third parties.
The mortgage on the shares of stocks should have been registered in the Office
of the Register Deeds of Manila where he resides, as well as in the stock and
transfer book of Xerxes Corporation.
Rule on the foregoing issues with reasons. (2009 Bar Question)

SUGGESTED ANSWER:
The mortgage on the shares of stock should be registered in the chattel mortgage
registry in the register of Deeds of Makati City where the corporation has its principal
office and also in the Register of Deeds of Manila where the mortgagor resides.
Registration of chattel mortgage in the stock and transfer book is not required to make
the chattel mortgage valid. Registration of dealings in the stock and transfer book under
Section 63 of the Corporation Code applies only to sale or disposition of shares, and
has no application to mortgages and other forms of encumbrances.
Assume that Bernardo extrajudicially foreclosed on the mortgages, and both the
car and the shared of stock were sold at public auction. If the proceeds from such
public sale should be 1 -million short of Armando’s total obligation, can Bernardo
recover the deficiency? Why or why not? (2009 Bar Question)
SUGGESTED ANSWER:
Yes. Bernardo can recover the deficiency. Chattels are given as mere security, and not
as payment or pledge.
On January 1, 2008, Al obtained a loan of P10,000 from Bob to be paid on January
30, 2008, secured by a chattel mortgage on a Toyota motor car. On February 1,
2008, Al obtained another loan of P10,000 from Bob to be paid on February 15,
2008. He secured this by executing a chattel mortgage on a Honda motorcycle.
43

On the due date of the first loan Al failed to pay. Bob foreclosed the chattel
mortgage but the car was bidded for P6,000 only. Al also failed to pay the second
Page

loan due on February 15, 2008. Bob filed an action for collection of sum of money.

CIVIL LAW REVIEW 2 – 4A BAR EXAMINATION Q & A (1980-2017)


Al filed a motion to dismiss claiming that Bob should first foreclose the mortgage
on the Honda motorcycle before he can file the action for sum of money. Decide
with reasons. (2008 Bar Question)
SUGGESTED ANSWER:
Bob can file an action for collection of a sum of money without first foreclosing the
chattel mortgage on the motorcycle of Al. Bob has the right to abandon the chattel
mortgage and file instead an action for collection of a sum of money.
Are the right of redemption and the equity of redemption given by law to a
mortgagor the same? Explain. (2%) (1999 Bar Question)
SUGGESTED ANSWER:
The equity of redemption is different from the right of redemption. Equity of redemption
is the right of the mortgagor after judgment in a judicial foreclosure to redeem the
property by paying to the court the amount of the judgment debt before the sale or
confirmation of the sale. On the other hand, right of redemption is the right of the
mortgagor to redeem the property sold at an extra-judicial foreclosure by paying to the
buyer in the foreclosure sale the amount paid by the buyer within one year from such
sale.
On December 1, 1996, Borrower executed a chattel mortgage in favor of the bank
to secure a loan of P3 M. in due time the loan was paid.
On December 1, 1997, Borrower obtained another loan of P2 M which the Bank
granted under the same security as that which secured the first loan.
For the second loan, Borrower merely delivered a promissory note; no new
chattel mortgage agreement was executed as the parties relied on a provision in
the 1996 chattel mortgage agreement which included future debts as among the
obligations secured by the mortgage. The provision reads:
―In case the MORTGAGOR executes subsequent promissory note or notes either
as renewal, as an extension, or as a new loan, this mortgage shall also stand as
security for the payment of said promissory note or notes without the necessity
of executing a new contract and this mortgage shall have the same force and
effect as if the said promissory note or notes were existing on date hereof.‖
As borrower failed to pay the second loan, the Bank proceeded to foreclose the
Chattel Mortgage.
Borrower sued the Bank claiming that the mortgage was no longer in force.
Borrower claimed that a fresh chattel mortgage should have been executed when
the second loan was granted.
Decide the case and ratiocinate.
Supposed the chattel mortgage was not registered, would its validity and
effectiveness be impaired? Explain. (1999 Bar Question)
SUGGESTED ANSWER:
The foreclosure of the chattel mortgage regarding the second loan is not valid. A chattel
mortgage cannot validly secure after-incurred obligations. The affidavit of good faith
required under the chattel mortgage law expressly provides that ―the foregoing
mortgage is made for securing the obligation specified in the conditions hereof, and for
no other purpose.‖ The after-incurred obligation not being specified in the affidavit, it is
not secured by the mortgage.
44

Yes. The chattel mortgage is not valid as against any person, except the mortgagor, his
executors and administrators.
Page

CIVIL LAW REVIEW 2 – 4A BAR EXAMINATION Q & A (1980-2017)


Vini constructed a building on a parcel of land he leased from Andrea. He chattel
mortgaged the land to Felicia. When he could not pay Felicia, Felicia initiated
foreclosure proceedings. Vini claimed that the building he had constructed on the
leased land cannot be validly foreclosed because the building was, by law, an
immovable.
Is Vini correct? (1994 Bar Question)

SUGGESTED ANSWER:
The Chattel Mortgage is void and cannot be foreclosed because the building is an
immovable and cannot be an object of a chattel mortgage.
It depends. If the building was intended and is built of light materials, the chattel
mortgage may be considered as valid as between the parties and it may be considered
in respect to them as movable property, since it can be removed from one place to
another. But if the building is of strong material and is not capable of being removed or
transferred without being destroyed, the chattel mortgage is void and cannot be
foreclosed.
If it was the land which Vini chattel mortgaged, such mortgage would be void, or at least
unenforceable, since he was not the owner of the land.
If what was mortgaged as a chattel is the building, the chattel mortgage is valid as
between the parties only, on grounds of estoppel which would preclude the mortgagor
from assailing the contract on the ground that its subject-matter is an immovable.
Therefore Vini's defense is untenable, and Felicia can foreclose the mortgage over the
building, observing, however, the procedure prescribed for the execution of sale of
ajudgment debtor's immovable under Rule 39, Rules of Court, specifically, that the
notice of auction sale should be published in a newspaper of general circulation.
The problem that Vini mortgaged the land by way of a chattel mortgage is untenable.
Land can only be the subject matter of a real estate mortgage and only an absolute
owner of real property may mortgage a parcel of land. (Article 2085 (2) Civil Code).
Hence, there can be no foreclosure.
But on the assumption that what was mortgaged by way of chattel mortgage was the
building on leased land, then the parties are treating the building as chattel. A building
that is not merely superimposed on the ground is an immovable property and a chattel
mortgage on said building is legally void but the parties cannot be allowed to disavow
their contract on account of estoppel by deed. However, if third parties are involved
such chattel mortgage is void and has no effect.
To secure the payment of his loan of P200,000, A executed in favor of the Angeles
Banking Corp., in 1 document, a real estate mortgage over 3 lots registered in his
name and a chattel mortgage over his 3 cars and 1 Isuzu cargo truck.
Upon his failure to pay the loan on due date, the bank foreclosed the mortgage on
the 3 lots, which were subsequently sold for only P99,000 at the foreclosure sale.
Thereafter, the bank filed an ordinary action for the collection of the deficiency. A
contended that the mortgage contract he executed was indivisible and
consequently, the bank had no legal right to foreclose only the real estate
mortgage and leave out the chattel mortgage, and then sue him for a supposed
deficiency judgment.
If you were the Judge, would you sustain the contention of A? (1991 Bar Question)
45
Page

CIVIL LAW REVIEW 2 – 4A BAR EXAMINATION Q & A (1980-2017)


SUGGESTED ANSWER:
If I were the Judge, I would dismiss the action as being premature since the proper
remedy would be to complete the foreclosure of the mortgages and only thereafter can
there be an action for collection of any deficiency. In Caltex vs. Intermediate Appellate
Court (G.R. 74730, 25 August 1989). The remedies on a secured debt, said the court,
are either an action to collect or to foreclose a contract of real security. These remedies
are alternative remedies, although an action for any deficiency is not precluded, subject
to certain exception such as those stated in Article 1484 of the Civil Code, by a
foreclosure on the mortgages. While the factual settings in the case of Suria vs.
Intermediate Appellate Court (30 June 1987) are not similar to the facts given in the
problem, the Supreme Court implied that foreclosure as a remedy in secured obligations
must first be availed of by a creditor in preference to other remedies that might also be
invoked by him.
Zone, who lives in Bulacan, bought a 1988 model Toyota Corolla sedan on July 1,
1989 from Anadelaida, who lives in Quezon City, for P300,000, paying P150,000 as
down payment and promising to pay the balance in 3 equal quarterly installments
beginning October 1, 1989. Anadelaida executed a deed of sale of the vehicle in
favor of Zonee and, to secure the unpaid balance of the purchase price, had
Zonee execute a deed of chattel mortgage on the vehicle in Anadelaida’s favor.
10 days after the execution of the abovementioned documents, Zonee had the car
transferred and registered in her name. Contemporaneously, Anadelaida had the
chattel mortgage on the car registered in the Chattel Mortgage Registry of the
Office of the Register of Deed of Quezon City.
In September 1989, Zonee sold the sedan to Jimbo without telling the latter that
the car was mortgaged to Anadelaida. When Zonee failed to pay the first
installment on October 1, 1989, Anadelaida went to see Zonee and discovered
that the latter had sold the car to Jimbo.
Jimbo refused to give up the car on the ground that the chattel mortgage
executed by Zonee in favor of Anadelaida is not valid because it was
executed before the car was registered in Zonee’s name, i.e., before Zonee
became the registered owner of the car. Is the said argument meritorious?
Explain your answer.
Jimbo also argued that even if the chattel mortgage is valid, it cannot affect
him because it was not properly registered with the government offices
where it should be registered. What government office is Jimbo referring to?
(1990 Bar Question)
SUGGESTED ANSWER:
Jimbo’s argument is not meritorious. Zone became the owner of the property upon
delivery; registration is not essential to vest that ownership in the buyer. The execution
of the chattel mortgage by the buyer in favor of the seller, in fact, can demonstrate the
vesting of such ownership to the mortgagor.
Jimbo was referring to the Register of Deeds of Bulacan where Zonee was a resident.
The Chattel Mortgage Law requires the registration to be made in the Office of the
Register of Deeds of the province where the mortgagor resides and also in which the
property is situated as well as the LTO where the vehicle is registered.
A bought a car from Sigma Motors Co. for P80,000, P40,000 payable upon delivery
and P40,000 one year after such delivery. As security for the payment of such
balance, A executed a chattel mortgage in favor of Sigma Motors, which
mortgage was recorded in the proper chattel mortgage registry. After 6 months, A
46

sold the car to B who, unaware of the registered chattel mortgage on the car,
Page

CIVIL LAW REVIEW 2 – 4A BAR EXAMINATION Q & A (1980-2017)


registered the sale and obtained a certificate of registration in his own name from
the Bureau of Land Transportation, which was not aware of the mortgage either.
Seven months later, Sigma Motors brought a suit to foreclose the chattel
mortgage since A had failed to pay the balance of P40,000. B opposed the suit.
May the foreclosure be legally made? (1981 Bar Question)
SUGGESTED ANSWER:
No. Chattel mortgage of a car must be registered not only in the Chattel Mortgage
Registry but also with the Bureau of Land Transportation otherwise it will not affect third
persons who were not aware thereof. B bought the car in good faith and since there was
nothing in the BLT records which showed the existence of any lien on the car, he had
the right to assume that such car was free from any encumbrance. The sale of the car
to B would therefore prevail over Sigma’s defectively registered chattel mortgage.
To secure a debt to Y, X, the owner of Supreme Drugstore, executed a chattel
mortgage covering the goods contained in the drugstore. The deed of chattel
mortgage provides that ―all goods, stock-in-trade, furniture and fixtures hereafter
purchased by the mortgagor shall be included in and covered by the mortgage.‖
Upon default by X, Y sought to foreclose the mortgage on the goods then found
in the drugstore, half of which were admittedly acquired after the execution of the
chattel mortgage.
If you were the lawyer of X, what arguments would you advance to defeat the
foreclosure on the after acquired property? If you were the judge, how would you
decide? (1984 Bar Question)
SUGGESTED ANSWER:
As the lawyer of X, I would argue that the chattel mortgage is invalid in respect of the
after acquired property, being in contravention of the last paragraph of Section 7 of the
Chattel Mortgage Law which provides:
―A chattel mortgage shall be deemed to cover only the property described therein and
not like a substituted property thereafter acquired by the mortgagor and placed in the
same depository as the property originally mortgaged, anything in the mortgage to the
contrary notwithstanding‖.
The Supreme Court, confronted with the same issue, held in Torres v. Limjap, 5 Phil.
141, that a similar stipulation in the chattel mortgage extending its effect to after
acquired property is valid and binding, taking into consideration the spirit and intent of
the Chattel Mortgage law.
Bearing in mind the primary aim of the lawmaker was to promote business and trade in
the Philippine Islands and to give impetus to the economic development of the country,
it could not have been the intention of the lawmaker to apply the last provision of Sec. 7
to stores open for retail business, where the goods are constantly sold and substituted
with new stocks, such as drug stores, grocery stores, dry good stores, etc. if such
provision were intended to apply to this class of business, it would be practically
impossible to constitute a mortgage on such stores without closing them, contrary to the
very spirit and purpose of the said Act. (Dee Hao Kiam v. Busiang, 56 Phil. 181.)
Note that the mortgage in question expressly provides that future acquisitions of goods
to be added to the original stock mortgaged shall be held as included in the mortgage.
Moreover, X is estopped.
47

Accordingly, if I were the judge, I would decide in favor of Y.


Page

CIVIL LAW REVIEW 2 – 4A BAR EXAMINATION Q & A (1980-2017)


Alfonso borrowed P10,000.00 from Bernabe payable in 10 equal monthly
installments of P1,000.00 per month. To secure payment of the loan, Alfonso
executed a chattel mortgage on his sports car. The condition of the mortgage was
violated.
What right does Bernabe have against Alfonso?
How should he enforce that right?
The proceeds of the sale after deducting expenses was not sufficient to pay the
mortgage debt. Bernabe now wants to collect the deficiency. Is he correct? Why?
(1976 Bar Question)
SUGGESTED ANSWER:
A. Bernabe has the right to:
Exact fulfillment of the obligation; or
Foreclose the chattel mortgage on the car.

B. Should he elect fulfillment of the obligation, he should institute the proper case in
court; cause the chattel to be attached as provided under the Rules of Court, and after a
favorable judgment, have it sold at public auction in the manner prescribed by the Rules
of Court.
On the other hand, should he elect to foreclose the mortgage all he has to do is to
cause the car to be sold at public auction pursuant to Sec.14 of the Chattel Mortgage
Law.
C. Yes. The creditor is entitled to a deficiency judgment in a chattel mortgage contract.
Under Sec. 14 of the Chattel Mortgage Law, it can be inferred that if the price of the sale
of the thing mortgage is less than the amount of the principal obligation an action may
still be maintained by the creditor against the debtor for the deficiency. The only
exception to this rule is that which is provided for in Art 1484 (3) of the Civil Code.
According to this provision, should the vendee fail to pay two or more installments, the
vendor may foreclose the chattel mortgage, but he shall not have further action against
the vendee to recover any unpaid balance of the price. (Ablaza v. Ignacio, L-11463,
May 23, 1958 and Garrido v. Tuason, L-23768, August 23, 1968)

A borrowed P10,000.00 from X. To secure payment thereof, A executed a chattel


mortgage on his Toyota car. The condition of the mortgage having been breached
What right does the mortgage have?
How should he proceed to enforce that right?
If the proceeds of the sale after deducting expenses are not sufficient to pay the
mortgage debt, is A entitled to collect the deficiency? Reason. (1976 Bar Question)
SUGGESTED ANSWER:
He may foreclose the mortgage either judicially or extrajudicially.

Judicial foreclosure should follow the provision of the Rules of Court. Extrajudicial
foreclosure is made in accordance with the Chattel Mortgage Law and is by means
of sale at public auction conducted by a public officer in the municipality where the
mortgagor resides or where the property is situated. Notice of the time, place and
purpose of the sale should be given in accordance with law.
Yes, he may collect the deficiency since the chattel is given only as security and not
as payment of the debt. The provision of the Civil Code barring deficiency
judgments in case of pledge is not applicable to chattel mortgage, which is
48

governed by a special law. Neither is it a sale by installments. (Garrido v. Tuazon,


24 SCRA 727)
Page

CIVIL LAW REVIEW 2 – 4A BAR EXAMINATION Q & A (1980-2017)


A executed a chattel mortgage on certain personal properties in favor of B to
secure a debt. The chattel properties were attached by C, knowing that said
properties had been mortgaged to B. As between B and C who has a better right
to the properties? Reason. (1976 Bar Question)
SUGGESTED ANSWER:
C has a better right to the properties. The Chattel Mortgage Law expressly provides that
an unregistered chattel mortgage is not valid against any person. Registration is
essential for the validity of the chattel mortgage.
VIII. Quasi-contracts

DPO went to a store to buy a pack of cigarettes worth P225.00 only. He gave the
vendor, RRA, a P500-peso bill. The vendor gave him the pack plus P375.00
change. Was there a discount, an oversight, or an error in the amount given?
What would be DPO’s duty, if any, in case of an excess in the amount of change
given by the vendor? How is this situational relationship between DPO and RRA
denominated? Explain. (5%) (2004 Bar Question)
SUGGESTED ANSWER:
There was error in the amount of change given by RRA. This is a case of solutioindebiti
in that DPO received something that is not due him. He has the obligation to return the
P100.00; otherwise, he will unjustly enrich himself at the expense of RRA. (Art. 2154,
Civil Code)
ALTERNATIVE ANSWER:
DPO has the duty to return to RRA the excess P100 as trustee under Article 1456 of the
Civil Code which provides: If property is acquired through mistake or fraud, the person
obtaining it is, by force of law, considered a trustee of an implied trust for the benefit of
the person from whom the property comes. There is, in this case, an implied or
constructive trust in favor of RRA.
Armando owns, a row of residential apartments in San Juan, Metro Manila, which
he rents out to tenants. On 1 April 1991 he left for the United States without
appointing any administrator to manage his apartments such that uncollected
rentals accumulated for three (3) years. Amparo, a niece of Armando, concerned
with the Interest of her uncle, took it upon herself to administer the property. As a
consequence, she incurred expenses in collecting the rents and in some
instances even spent for necessary repairs to preserve the property.
What juridical relation between Amparo and Armando, if any, has
resulted from Amparo’s unilateral act of assuming the administration of
Armando’s apartments? Explain.

What rights and obligations, if any, does Amparo have under the
circumstances? Explain. (1995 Bar Question)
SUGGESTED ANSWER:
Negotiorum gestio existed between Amparo and Armando. She voluntarily took charge
of the agency or management of the business or property of her uncle without any
power from her uncle whose property was neglected. She is called the gestor
negotiorum or officious manager. (Art. 2144, NCC)
It is recommended by the Committee that an enumeration of any two (2) obligations and
two (2) rights as enumerated in Arts. 2145 to 2152, NCC, would entitle the examinee to
49

full credit.
Page

CIVIL LAW REVIEW 2 – 4A BAR EXAMINATION Q & A (1980-2017)


Art. 2145. The officious manager shall perform his duties with all the diligence of a
good father of a family, and pay the damages which through his fault or negligence may
be suffered by the owner of the property or business under management.
The courts may, however, increase or moderate the indemnity according to the
circumstances of each case.
Art. 2146. If the officious manager delegates to another person all or some of his
duties, he shall be liable for the acts of the delegate, without prejudice to the direct
obligation of the latter toward the owner of the business.
The responsibility of two or more officious managers shall be solidary, unless
management was assumed to save the thing or business from imminent danger.
Art. 2147. The officious manager shall be liable for any fortuitous event:

If he undertakes risky operations which the owner was not accustomed to


embark upon:
If he has preferred his own interest to that of the owner;
If he fails to return the property or business after demand by the owner;
If he assumed the management in bad faith.
Art. 2148. Except when the management was assumed to save the property or
business from imminent danger, the officious manager shall be liable for fortuitous
events
If he is manifestly unfit to carry on the management;
If by his intervention he prevented a more competent person from taking up the
management.

Art. 2149. The ratification of the management by the owner of the business
produces the effects of an express agency, even if the business may not have been
successful.
Art. 2150. Although the officious management may not have been expressly
ratified, the owner of the property business who enjoys the advantages of the same
shall be liable for obligations Incurred In his interest, and shall reimburse the officious
manager for the necessary and useful expenses and for the damages which the latter
may have suffered in the performance of his duties.
The same obligation shall be incumbent upon him when, the management had for
its purpose the prevention of an imminent and manifest loss, although no benefit may
have been derived.
Art. 2151. Even though the owner did not derive any benefit and there has been no
imminent and manifest danger to the property or business, the owner is liable as under
the first paragraph of the preceding article, provided:
The officious manager has acted in good faith, and
The property or business is intact, ready to be returned to the owner.
Art. 2152. The officious manager is personally liable for contracts which he has
entered into with third persons, even though he acted in the name of the owner, and
there shall be no right of action between the owner and third persons.. These provisions
shall not apply:
If the owner has expressly or tacitly ratified the management, or
50

When the contract refers to things pertaining to the owner of the business.
Page

CIVIL LAW REVIEW 2 – 4A BAR EXAMINATION Q & A (1980-2017)


(NOTE: It is recommended by the Committee that an enumeration of any two (2)
obligations and any two (2) rights as enumerated in Arts. 2145 to 2152. NCC would
entitle the examinee to full credit.)
In September, 1972, upon declaration of martial rule in the Philippines, A,
together with his wife and children, disappeared from his residence along A.
Mabini Street. Ermita, Manila. B. his immediate neighbor, noticing that mysterious
disappearance of A and his family, closed the doors and windows of his house to
prevent it from being burglarized. Years passed without B hearing from A and his
family. B continued taking care of A's house, even causing minor repairs to be
done at his house to preserve it. In 1976, when business began to perk up in the
area, an enterprising man, C, approached B and proposed that they build stores
at the ground floor of the house and convert its second floor into a pension
house. B agreed to C’s proposal and together they spent for the construction of
stores at the ground floor and the conversion of the second floor into a pension
house. While construction was going on, fire occurred at a nearby house. The
houses at the entire block, including A's, were burned. After the EDSA revolution

in February 1986. A and his family returned from the United States where they
took refuge in 1972. Upon learning of what happened to his house. A sued B for
damages. B pleaded as a defense that he merely look charge of his house under
the principle of negotiorum gestio. He was not liable as the burning of the house
is a fortuitous event.
Is B liable to A for damages under the foregoing circumstances? (1993 Bar
Question)
SUGGESTED ANSWER:
No, B is not liable for damages, because he is a gestor in negotiorum gestio (Art. 2144,
Civil Code).
Furthermore, B is not liable to A because Article 2147 of the Civil Code is not
applicable.
B did not undertake risky operations which the owner was not accustomed to embark
upon:
he has not preferred his own interest to that of the owner:
he has not failed to return the property or business after demand by the owner; and
he has not assumed the management in bad faith.
ALTERNATIVE ANSWER:
He would be liable under Art.2147 (1) of the Civil Code, because he used the property
for an operation which the operator is not accustomed to, and in so doing, he exposed
the house to increased risk, namely the operation of a pension house on the second
floor and stores on the first floor.
In fear of reprisals from lawless elements besieging his barangay, X abandoned
his fishpond, fled to Manila and left for Europe. Seeking that the fish in the
fishpond were ready for harvest, Y, who is in the business of managing fishponds
on a commission basis, took possession of the property, harvested the fish and
sold the entire harvest to Z.
Thereafter, Y borrowed money from W and used the money to buy new supplies
of fish fry and to prepare the fishpond for the next crop.
51

What is the Juridical relation between X and Y during X’s absence?


Page

CIVIL LAW REVIEW 2 – 4A BAR EXAMINATION Q & A (1980-2017)


Upon the return of X to the barangay, what are the obligations of Y to X as
regards the contract with Z?
Upon X’s return, what are the obligations of X as regards Y’s contract with W?
What legal effects will result If X expressly ratifies Ys management and what
would be the obligations of X in favor of Y? (1992 Bar Question)

SUGGESTED ANSWER:
The juridical relation is that of the quasi-contract of ―negotiorum gestio". Y Is the
―gestor" or ―officious manager" and X is the ―owner" (Art. 2144, Civil Code).
Y must render an account of his operations and deliver to X the price he received for
the sale of the harvested fish (Art. 2145, Civil Code).
X must pay the loan obtained by Y from W because X must answer for obligations
contracted with third persons in the interest of the owner (Art. 2150, Civil Code).
Express ratification by X provides the effects of an express agency and X is liable to
pay the commissions habitually received by the gestor as manager (Art. 2149, Civil
Code).
Distinguish an implied contract from a quasi- contract. (1989 Bar Question)

SUGGESTED ANSWER:
Any of the following answers should be given full credit:

A. An implied contract requires consent of the parties. A quasi-contract is not predicated


on consent, being a unilateral act.
B. The basis of an implied contract is the will of the parties. The basis of a quasi-
contract is law to the end that there be no unjust situation.
52
Page

CIVIL LAW REVIEW 2 – 4A BAR EXAMINATION Q & A (1980-2017)


TORTS AND DAMAGES

Jovencio operated a school bus to ferry his two sons and five of their
schoolmates from their houses to their school, and back. The parents of the five
schoolmates paid for the service. One morning, Porfirio, the driver, took a short
cut on the way to school because he was running late, and drove across an
unmanned railway crossing. At the time, Porfirio was wearing earphones because
he loved to hear loud music while driving. As he crossed the railway tracks, a
speeding PNR train loudly blared its horn to warn Porfirio, but the latter did not
hear the horn because of the loud music. The train inevitably rammed into the
school bus. The strong impact of the collision between the school bus and the
train resulted in the instant death of one of the classmates of Jovencio's younger
son. The parents of the fatality sued Jovencio for damages based on culpa
contractual alleging that Jovencio was a common carrier; Porfirio for being
negligent; and the PNR for damages based on culpa aquiliana. Jovencio denied
being a common carrier. He insisted that he had exercised the diligence of a good
father of a family in supervising Porfirio, claiming that the latter had had no
history of negligence or recklessness before the fatal accident.

Did his operation of the school bus service for a limited clientele render Jovencio
a common carrier? Explain your answer.

In accordance with your answer to the preceding question, state the degree of
diligence to be observed by Jovencio, and the consequences thereof. Explain
your answer.

Assuming that the fatality was a minor of only 15 years of age who had no
earning capacity at the time of his death because he was still a student in high
school, and the trial court is minded to award indemnity, what may possibly be
the legal and factual justifications for the award of loss of earning capacity?
Explain your answer. (2017)

SUGGESTED ANSWER:
The operation of the school bus service for a limited clientele rendered Jovencio a
common carrier because he offered his business of transporting passengers for
compensation to the public, as evidenced by the payment for such service by the
parents of the schoolmates.

The degree of diligence to be observed by Jovencio as a common carrier is


extraordinary diligence for the safety of the passengers transported by him, according to
all the circumstances of each case. In case of death of or injuries to passengers,
Jovencio is presumed to have been at fault or to have acted negligently, unless he
proves that he observed extraordinary diligence.

The legal and factual justification for the award of loss of earning capacity is to present
sufficient evidence to establish that the minor was reasonably certain to complete
training for a specific profession. Compensation should be allowed for loss of earning
capacity resulting from the death of a minor who has not yet commenced employment
or training for a specific profession.

Dr. Jack, a surgeon, holds clinic at the St. Vincent's Hospital and pays rent to the
hospital. The fees of Dr. Jack are paid directly to him by the patient or through the
cashier of the hospital. The hospital publicly displays in the lobby the names and
specializations of the doctors associated or accredited by it, including that of Dr.
Jack. Marta engaged the services of Dr. Jack because of recurring stomach pain.
53

It was diagnosed that she is suffering from cancer and had to be operated on.
Before the operation, she was asked to sign a "consent for hospital care," which
Page

CIVIL LAW REVIEW 2 – 4A BAR EXAMINATION Q & A (1980-2017)


reads: "Permission is hereby given to the medical, nursing and laboratory staff of
the St. Vincent's Hospital to perform such procedures and to administer such
medications and treatments as may be deemed necessary or advisable by the
physicians of this hospital for and during the confinement." After the surgery, the
attending nurses reported that two (2) sponges were missing. Later, Marta died
due to complications brought about by the sponges that were left in her stomach.
The husband of Marta sued the hospital and Dr. Jack for damages arising from
negligence in the medical procedure. The hospital raised the defense that Dr.
Jack is not its employee as it did not hire Dr. Jack nor pay him any salary or
compensation. It has absolutely no control over the medical services and
treatment being provided by Dr. Jack. Dr. Jack even signed an agreement that he
holds the hospital free and harmless from any liability arising from his medical
practice in the hospital.

Is St. Vincent's Hospital liable for the negligence of Dr. Jack? Explain your
answer. (2016)

SUGGESTED ANSWER:
Yes, the hospital is liable for the negligence of the doctor. Hospitals exercise significant
control in the hiring and firing of consultants and in the conduct of their work within the
hospital premises. Accordingly, the purpose of allocating responsibility in medical
negligence cases, an employer-employee relationship in effect exists between hospitals
and their attending and visiting physicians. The basis for holding an employer solidarily
responsible for the negligence of its employee is found in Article 2180 of the Civil Code
which considers a person accountable not only for his own acts but also for those of
others based on the former’s responsibility under a relationship of partia ptetas. (Ramos
v. CA, G.R. No. 124354, December 29, 1999)

Its liability is also anchored upon the agency principle of apparent authority or agency
by estoppel and the doctrine of corporate negligence. The hospital is estopped from
passing all the blame to Dr. Jack whose name it proudly paraded in the public directory
leading the public to believe that it vouched for his skill and competence. (Agana Case,
G.R. No. 126297, 126467, 127590, January 31, 2007)

Peter, a resident of Cebu City, sent through Reliable Pera Padala (RPP) the
amount of P20,000.00 to his daughter, Paula, for the payment of her tuition fee.
Paula went to an RPP branch but was informed that there was no money remitted
to her name. Peter inquired from RPP and was informed that there was a
computer glitch and the money was credited to another person. Peter and Paula
sued RPP for actual damages, moral damages and exemplary damages. The trial
court ruled that there was no proof of pecuniary loss to the plaintiffs but awarded
moral damages of P20,000.00 and exemplary damages of P5,000.00. On appeal,
RPP questioned the award of moral and exemplary damages.

Is the trial court correct in awarding moral and exemplary damages? Explain.
(2016)

SUGGESTED ANSWER:

No, the trial court is not correct in awarding moral and exemplary damages. The
damages in this case are prayed for based on the breach of contract committed by RPP
in failing to deliver the sum of money to Paula. Under the provisions of the Civil Code, in
breach of contract, moral damages may be recovered when the defendant acted in bad
faith or was guilty of gross negligence (amounting to bad faith) or in wanton disregard of
his contractual obligation. In the same fashion, to warrant the award of exemplary
54

damages, the wrongful act must be accomplished by bad faith, and an award of
damages would be allowed only if the guilty party acted in a wanton, fraudulent,
Page

reckless or malevolent manner (Article 2232 of the Civil Code). Bad faith does not

CIVIL LAW REVIEW 2 – 4A BAR EXAMINATION Q & A (1980-2017)


simply connote bad judgment or negligence. It imports a dishonest purpose or some
moral obliquity and conscious doing of a wrong, a breach of known duty through some
motive or interest or ill will that partakes of the nature of fraud. In this case, however,
RPP's breach was due to a computer glitch which at most can be considered as
negligence on its part, but definitely does not constitute bad faith or fraud as would
warrant the award of moral and exemplary damages.

A driver of a bus owned by company Z ran over a boy who died instantly. A
criminal case for reckless imprudence resulting in homicide was filed against the
driver. He was convicted and was ordered to pay P2 Million in actual and moral
damages to the parents of the boy who was an honor student and had a bright
future. Without even trying to find out if the driver had assets or means to pay the
award of damages, the parents of the boy filed a civil action against the bus
company to make it directly liable for the damages.

Will their action prosper?

If the parents of the boy do not wish to file a separate civil action against the bus
company, can they still make the bus company liable if the driver cannot pay the
award for damages? If so, what is the nature of the employer's liability and how
may civil damages be satisfied? (2015)

SUGGESTED ANSWER:
Yes, the action will prosper. The liability of the employer in this case may be based on
quasi-delict and is included within the coverage of an independent civil action. It is not
necessary to enforce the civil liability based on culpa aquiliana that the driver or
employee be proven to be insolvent since the liability of the employer for the quasi-delict
committed by their employees is direct and primary subject to the defense of due
diligence on their part.

Yes, the parents of the boy can enforce the subsidiary liability of the employer in the
criminal case against the driver. The conviction of the driver is a condition sine qua non
for the subsidiary liability of the employer to attach. Proof must be shown that the driver
is insolvent.

Mabuhay Elementary School organized a field trip for its Grade VI students in Fort
Santiago, Manila Zoo, and Star City. To be able to join, the parents of the students
had to sign a piece of paper that reads as follows: "I allow my child (name of
student), Grade – Section, to join
the school’s field trip on February 14, 2014. I will not file any claim against the
school, administrator or teacher in case something happens to my child during
the trip." Joey, a 7-year-old student of Mabuhay Elementary School was bitten by
a snake while the group was touring Manila Zoo. The parents of Joey sued the
school for damages. The school, as a defense, presented the waiver signed by
Joey’s parents.

Was there a valid waiver of right to sue the school? Why? (2014)

SUGGESTED ANSWER:
No, there was no valid waiver of the right to sue the school. A waiver to be valid must
have three requisites 1) existence of the right; 2) legal capacity of the person waiving
the right and 3) the waiver must not be contrary to law, morals, good customs, public
order or public policy or prejudicial to a third person with a right recognized by law. In
the case presented, the waiver may be considered contrary to public policy as it
exonerates the school from liability for future negligence. The waiver in effect allows the
55

school to not exercise even ordinary diligence.


Page

CIVIL LAW REVIEW 2 – 4A BAR EXAMINATION Q & A (1980-2017)


A collision occurred at an intersection involving a bicycle and a taxicab.
Both the bicycle rider (a businessman then doing his morning exercise) and the
taxi driver claimed that the other was at fault. Based on the police report, the
bicycle crossed the intersection first but the taxicab, crossing at a fast clip from
the bicycle's left, could not brake in time and hit the bicycle's rear wheel, toppling
it and throwing the bicycle rider into the sidewalk 5 meters away. The bicycle
rider suffered a fractured right knee, sustained when he fell on his right side on
the concrete side walk. He was hospitalized and was subsequently operated on,
rendering him immobile for 3 weeks and requiring physical rehabilitation for
another 3 months. In his complaint for damages, the rider prayed for the award of
P1,000,000 actual damages, P200,000 moral damages, P200,000 exemplary
damages, P100,000 nominal damages and P50,000 attorney's fees.
Assuming the police report to be correct and as the lawyer for the bicycle
rider, what evidence (documentary and testimonial) and legal arguments will you
present in court to justify the damages that your client claims? (1994, 2002, 2013)

SUGGESTED ANSWER:
I will base the claim of my client on quasi-delict. The requisites for a claim under quasi-
delict to prosper are as follows:
a) Act or omission, there being fault or negligence;
b) Damage or injury; and
c) Causal connection between the damage and the act or omission.
The case clearly involves a quasi-delict where my client, the bicycle rider,
suffered injury as a result of the negligence of the over-speeding taxi driver, without fault
on my client’s part.
To prove actual damages, aside from the testimony of my client, I will present his
hospital and medical bills. Receipts of the fees paid on the rehabilitation will also be
presented. Furthermore, I will present income tax returns, contracts and other
documents to prove unrealized profits as a result of this temporary injury. I will also call
the attending physician to testify as to the extent of the injuries suffered by my client,
and to corroborate the contents of the medical documents.
In quasi-delicts, the defendant shall be liable for all damages which are the
natural and probable consequences of the act or omission complained of. It is not
necessary that such damages have been foreseen or could have been foreseen by the
defendant.
Unlike actual damages, no proof of pecuniary loss is necessary in order that
moral, nominal, temperate, liquidated or exemplary damages may be adjudicated. The
assessment is left to the discretion of the Court. There must still be proof of pecuniary
estimation, however.
Moral damages can be recovered by my client. Moral damages may be
recovered in case of a quasi-delict causing physical injuries. Additionally, it must be
proved that such damages were the proximate result of the act complained of. Medical
certificates will be presented, along with the testimony from my client and other
eyewitness accounts, in order to support the award for moral damages.
Exemplary damages may be granted if the defendant acted in wanton,
fraudulent, reckless, oppressive, or malevolent manner. While the amount of exemplary
damages need not be proved, the plaintiff must show that he is entitled to moral or
compensatory damages. In support of this, I will present the police report showing the
circumstances under which the accident took place, taking into account the actions of
the parties. I will ask the officials who responded to the accident to testify as to the
conduct of the parties at the time of the accident in order to determine whether
defendant was guilty of gross negligence.
Finally, attorney’s fees may be recovered when exemplary damages are
awarded.
56

Roberto was in Nikko Hotel when he bumped into a friend who was then on
Page

her way to a wedding reception being held in said hotel. Roberto alleged that he

CIVIL LAW REVIEW 2 – 4A BAR EXAMINATION Q & A (1980-2017)


was then invited by his friend to join her at the wedding reception and carried the
basket full of fruits which she was bringing to the affair. At the reception, the
wedding coordinator of the hotel noticed him and asked him, allegedly in a loud
voice, to leave as he was not in the guest list. He retorted that he had been invited
to the affair by his friend, who however denied doing so. Deeply embarrassed by
the incident, Roberto then sued the hotel for damages under Articles 19 and 21 of
the Civil Code.
Will Roberto’s action prosper? Explain. (2012)

SUGGESTED ANSWER:
It depends. While the hotel has the right to exclude an uninvited guest from the
wedding reception, that does not give the hotel the license to humiliate Roberto. If the
wedding coordinator of the hotel acted wrongfully e.g. with abuse of right, unfairly, or in
a manner that exposed Roberto to unnecessary ridicule or shame, his action will
prosper. Otherwise, Roberto’s action will prosper. The hotel is liable for the wrongful
acts of its employees.
Alternative: No, Roberto’s action will not prosper. From the facts given in the
problem, the wedding coordinator did not abuse her right when she asked him to leave
the wedding reception because he was not in the guest list. Hotel Nikko could not be
held liable for damages as its liability spring from the liability of its employee (Nikko
Hotel Manila Garden v. Reyes, G.R. No. 154259, February 28, 2005).

Liwayway Vinzons-Chato was then the Commissioner of Internal Revenue


while Fortune Tobacco Corporation is an entity engaged in the manufacture of
different brands of cigarettes, among which are "Champion," "Hope," and "More"
cigarettes. Fortune filed a complaint against Vinzons-Chato to recover damages
for the alleged violation of its constitutional rights arising from Vinzons-Chato’s
issuance of Revenue Memorandum Circular No. 37-934 (which re-classified
Fortune cigarettes as locally manufactured with foreign brands and thereby
imposed higher taxes), which the Supreme Court later declared invalid. Vinzons-
Chato filed a Motion to dismiss arguing that she cannot be held liable for
damages for acts she performed while in the discharge of her duties as BIR
Commissioner.
Is she correct? Explain (2012)

SUGGESTED ANSWER:
Yes. As a general rule, a public officer is not liable for acts performed in the
discharge of his duties. The exceptions are when he acted with malice, bad faith, or
gross negligence in the performance of his duty, or when his act is in violation of the
constitutionally guaranteed rights and liberties of a person under Art. 32.
The public officer is not automatically considered to have violated the rights or
liberties of a person simply because the rule the public officer issued was declared
invalid by the Court. The complainant must still allege and prove the particular injury or
prejudice he has suffered from the violation of his constitutional right by the issuance of
the invalidated rule.
The problem does not state any fact from which any malice, bad faith or gross
negligence on the part of Vinzons-Chato may be inferred, or the particular injury or
prejudice the complainant may have suffered as a result of the violation of his
constitutional rights. Hence, she cannot be held liable. The facts presented are similar
to the facts of the case of Vinzons-Chato v. Fortune, (G.R. No. 141309, December 23,
2008).

The liability of the school, its administrators and teachers, or the individual,
entity or institution engaged in child care over the minor child or damage caused
57

by the acts or omissions of the unemancipated minor while under their


supervision, instruction or custody shall be:
Page

a) Joint and subsidiary

CIVIL LAW REVIEW 2 – 4A BAR EXAMINATION Q & A (1980-2017)


b) Principal and solidary
c) Principal and joint
d) Subsidiary and solidary. (2012)

Virgilio owned a bare and simple swimming pool in his garden. MB, a 7-
year old child, surreptitiously entered the garden and merrily romped around the
ledges of the pool. He accidentally tripped, fell into the pool, and drowned. MB’s
parents sued Virgilio for damages arising from their child’s death, premised on
the principle of "attractive nuisance". Is Virgilio liable for the death of MB? (2011)
a) No, the child was 7 years old and knew the dangers that the pool offered.
b) Yes, being an attractive nuisance, Virgilio had the duty to prevent children from
coming
c) near it.
d) No, since the pool was bare and had no enticing or alluring gadgets, floats,
or devices in it that would attract a 7-year old child.
e) Yes, since Virgilio did not cover the swimming pool while not in use to prevent
children from falling into it.

Lennie bought a business class ticket from Alta Airlines. As she checked
in, the manager downgraded her to economy on the ground that a Congressman
had to be accommodated in the business class. Lennie suffered the discomfort
and embarrassment of the downgrade. She sued the airlines for quasi-delict but
Alta Airlines countered that, since her travel was governed by a contract between
them, no quasi-delict could arise.
Is the airline correct? (2011)
a) No, the breach of contract may in fact be tortious as when it is tainted as in
this case with arbitrariness, gross bad faith, and malice.
b) No, denying Lennie the comfort and amenities of the business class as provided
in the ticket is a tortious act.
c) Yes, since the facts show a breach of contract, not a quasi-delict.
d) Yes, since quasi-delict presupposes the absence of a pre-existing contractual
relation between the parties.

Define quasi tort. Who are the persons liable under quasi torts and what
are the defenses available to them? (2010)

SUGGESTED ANSWER:
Quasi-tort is a legal concept upholding the doctrine that some legal duty exists that
cannot be classified strictly as a personal duty (that is, resulting in a tort), nor as a
contractual duty (thus resulting in a breach of contract) but rather some other kind of
duty recognizable by the law. ―Tort‖ or ―Quasi-Tort‖ is an Anglo American or Common
Law concept, while ―Delict‖ or ―Quasi-Delict‖ is a Civil Law concept.

Alternative:
1. Quasi-tort is considered as the equivalent of quasi-delict. Hence the rules of the
latter pertaining to persons who can be held liable and their defenses would also
apply.
a. Those liable for quasi-delict include:
2. The tortfeasor or the person causing damage to another through fault or
negligence; and
3. Persons vicariously liable under Article 2180.
4. That the defendant was not negligent or that he exercised due diligence.
5. That although the defendant is negligent, his negligence is not the proximate
cause of the injury.
6. That the plaintiffs own negligence was the immediate and proximate cause of his
58

7. injury.
Page

CIVIL LAW REVIEW 2 – 4A BAR EXAMINATION Q & A (1980-2017)


8. That the person vicariously liable has observed all the diligence of a good father
of a family to prevent damage.
9. That the cause of action has prescribed after the lapse of 4 years.
10. The fact that the plaintiff had committed contributory negligence is a partial
11. defense.

On May 5, 1989, 16-year old Rozanno, who was issued a student permit,
drove to school a car, a gift from his parents. On even date, as his class was
scheduled to go on a field trip, his teacher requested him to accommodate in his
car, as he did, four (4) of his classmates because the van rented by the school
was too crowded. On the way to a museum which the students were scheduled to
visit, Rozanno made a wrong maneuver, causing a collision with a jeepney. One
of his classmates died. He and the three (3) others were badly injured.
Who is liable for the damage to the jeepney? Explain.
Under the same facts, except the date of occurrence of the incident, this
time in mid-1994, what would be your answer? Explain. (2010)

SUGGESTED ANSWER:
With respect to the damages caused to the jeepney, only Rozanno should be
held liable because his negligence or tortious act was the sole, proximate, and
immediate cause thereof.
Since Rozanno was 16 years old in 1989, if the incident happened sometime in
the middle of 1994, Rozanno would have been 21 years old at that time. Hence, he was
already of legal age. The law reducing the age of majority to 18 years took effect in
December 1989.
Being of legal age, Articles 218, 219, and 221 of the Family Code are no longer
applicable. In such case, only Rozanno will be personally responsible for all the
consequences of his act unless the school or his parents were themselves also
negligent and such negligence contributed to the happening of the incident. In that
event, the school or his parents are not liable under Article 218, 219 or 221 of the
Family Code, but will be liable under the general provisions of the Civil Code on quasi-
delict.

Primo owns a pet iguana which he keeps in a man- made pond enclosed by
a fence situated in his residential lot. A typhoon knocked down the fence of the
pond and the iguana crawled out of the gate of Primo’s residence. N, a neighbour
who was passing by, started throwing stones at the iguana, drawing the iguana to
move toward him. N panicked and ran but tripped on something and suffered a
broken leg.
Is anyone liable for N’s injuries? Explain. (2010)
SUGGESTED ANSWER:
No one is liable. The possessor of an animal or whoever may make use of the
same is responsible for the damage which it may cause, although it may escape or be
lost. This responsibility shall cease only in case the damage should come from force
majeure or from the fault of the person who has suffered damage.

Rodolfo, married to Sharon, had an illicit affair with his secretary, Nanette,
a 19-year old girl, and begot a baby girl, Rona. Nanette sued Rodolfo for
damages: actual, for hospital and other medical expenses in delivering the child
by caesarean section; moral, claiming that Rodolfo promised to marry her,
representing that he was single when, in fact, he was not; and exemplary, to
teach a lesson to like-minded Lotharios.
If you were the judge, would you award all the claims of Nanette? Explain.
(2009)
59
Page

CIVIL LAW REVIEW 2 – 4A BAR EXAMINATION Q & A (1980-2017)


SUGGESTED ANSWER:
If Rodolfo’s marriage could not have been possibly known to Nanette or there is
no gross negligence on the part of Nanette, Rodolfo could be held liable for moral
damages. If there is gross negligence in a suit for quasi-delict, exemplary damages
could be awarded.

Rommel’s private car, while being driven by the regular family driver,
Amado, hits a pedestrian causing the latter’s death. Rommel is not in the car
when the incident happened.
Is Rommel liable for damages to the heirs of the deceased? Explain.
Would your answer be the same if Rommel was in the car at the time of the
accident? Explain. (2009)

SUGGESTED ANSWER:
Yes, Rommel may be held liable for damages if he fails to prove that he
exercised the diligence of a good father of a family (Art. 2180, par. 5, NCC) in selecting
and supervising his family driver. The owner is presumed liable unless he proves the
defense of diligence. If the driver was performing his assigned task when the incident
happened, Rommel shall be solidarily liable with the driver. In case the driver is
convicted of reckless imprudence and cannot pay the civil liability, Rommel is
subsidiarily liable for the damages awarded against the driver and the defense of
diligence is not available.
Yes, my answer would be the same. Rommel, who was in the car, shall be liable
for damages if he could have prevented the misfortune by the use of due diligence in
supervising his driver but failed to exercise it (Art. 2184, NCC). In such a case, his
liability is solidary with his driver.
Alternative: Yes, my answer will be the same except that in such a case the
liability of the owner is not presumed. When the owner is inside the vehicle, he becomes
liable only when it is shown that he could have prevented the misfortune by the use of
due diligence (Article 2184, NCC). For the owner to be held liable, the burden of proving
that he could have prevented the misfortune rests on the shoulder of the victim.

Explain the doctrine of discovered peril (last clear chance). (2007)

SUGGESTED ANSWER:
The doctrine of last clear chance states that where the plaintiff was guilty of prior
or antecedent negligence but the defendant, who had the ultimate opportunity to avoid
the impending harm failed to do so, it is the defendant who is liable for all the
consequences of the accident notwithstanding the prior negligence of the plaintiff.
An example is where a person was riding a pony on a bridge and improperly
pulled the pony to the wrong side when he saw a car coming. The driver of the car did
not stop or change direction, and nearly hit the horse, and, the frightened animal
jumped to its death. The driver of the car is guilty of negligence because he had a fair
opportunity to avoid the accident and failed to avail himself of that opportunity. He is
liable under the doctrine of last clear chance (Picart v. Smith, 37 Phil. 809 [1918]).

Tony bought a Ford Expedition from a car dealer in Muntinlupa City. As


payment, Tony issued a check drawn against his current account with Premium
Bank. Since he has a good reputation, the car dealer allowed him to immediately
drive home the vehicle merely on his assurance that his check is sufficiently
funded. When the car dealer deposited the check, it was dishonored on the
ground of ―Account Closed‖. After an investigation, it was found that an
employee of the bank misplaced Tony's account ledger. Thus, the bank
erroneously assumed that his account no longer exists. Later, it turned out that
Tony’s account has more than sufficient funds to cover the check. The dealer
60

however, immediately filed an action for recovery of possession of the vehicle


against Tony for which he was terribly humiliated and embarrassed.
Page

Does Tony have a cause of action against Premium Bank? Explain. (2006)

CIVIL LAW REVIEW 2 – 4A BAR EXAMINATION Q & A (1980-2017)


SUGGESTED ANSWER:
Yes, Tony has a cause of action against Premium Bank. According to Art. 2176,
whoever by act or omission causes damages to another, there being fault or
negligence, is obliged to pay for the damage done. The proximate cause of the injury
which is the dishonor of Tony’s check, was the bank’s negligence in misplacing his
account ledger. The fiduciary nature of banking requires high standards of integrity and
performance necessitating banks to treat the accounts of its depositors with meticulous
care.

Arturo sold his Pajero to Benjamin for P1 Million. Benjamin took the vehicle
but did not register the sale with the Land Transportation Office. He allowed his
son Carlos, a minor who did not have a driver’s license, to drive the car to buy
pan de sal in a bakery. On the way, Carlos, driving in a reckless manner,
sideswiped Dennis, then riding a bicycle. As a result, he suffered serious physical
injuries. Dennis filed a criminal complaint against Carlos for reckless imprudence
resulting in serious physical injuries.
Can Dennis file an independent civil action against Carlos and his father
Benjamin for damages based on quasi-delict? Explain.
Assuming Dennis' action is tenable, can Benjamin raise the defense that he
is not liable because the vehicle is not registered in his name? Explain. (2006)

SUGGESTED ANSWER:
Yes, Dennis can file an independent civil action against Carlos and his father,
Benjamin. The independent civil action against Carlos can be based on Article 2176 of
the Civil Code, which states that, "whoever by act or omission causes damage to
another, there, being fault or negligence, is obliged to pay for the damage done." The
proximate cause of the injury suffered by Dennis, was the negligent driving of Carlo. He
can thus be held personally liable by the former for said injuries.
No, Benjamin cannot raise the defense that he is not liable because the vehicle is
not registered in his name. Had Dennis sued Benjamin based on the latter’s liability as
the owner of the vehicle, the non-registration of the vehicle in his name would have
been a valid defense. As held in the case of BA Finance Corporation v. CA (215 SCRA
715 [19921), it is the registered owner of any vehicle, who should be primarily
responsible to the public or third persons for injuries caused the latter while the vehicle
is being driven. In this case, Arturo was not sued. If sued, Arturo should be held liable
for the injury incurred by Dennis.
However, Benjamin is not being sued based on his ownership of the registered
vehicle, but rather for his responsibility as the parent of a minor child whose negligent
act resulted to damage or injury to another. As provided in Article 2180 of the Civil
Code, as amended by Article 221 of the Family Code, the father and mother are
responsible for the damages caused by the fault and negligence of the minor children
who live in their company. This liability is imposed upon the parents on the presumption
that they have failed in their duty of supervision over their children. Regardless of the
ownership of the vehicle, Dennis, therefore, has a cause of action against Benjamin.

Spouses Biong and Linda wanted to sell their house. They found a
prospective buyer, Ray. Linda negotiated with Ray for the sale of the property.
They agreed on a fair price of P2 Million. Ray sent Linda a letter confirming his
intention to buy the property. Later, another couple, Bemie and Elena, offered a
similar house at a lower price of P1.5 Million. But Ray insisted on buying the
house of Biong and Linda for sentimental reason. Ray prepared a deed of sale to
be signed by the couple and a manager’s check of P2 Million. After receiving the
P2 Million, Biong
signed the deed of sale. However, Linda was not able to sign it because she was
61

abroad. On her return she refused to sign the document saying she changed her
mind. Linda filed suit for nullification of the deed of sale and for moral and
Page

exemplary damages against Ray.

CIVIL LAW REVIEW 2 – 4A BAR EXAMINATION Q & A (1980-2017)


Does Ray have any cause of action against Biong and Linda? Can he also
recover damages from the spouses? Explain. (2006)

SUGGESTED ANSWER:
Yes, Ray has a cause of action against Linda and Biong for the return of the 2
million pesos he paid for the property. He may recover damages from the spouses, if it
can be proven that they were in bad faith in backing out from the contract, as this is an
act contrary to morals and good customs under Articles 19 and 21 of the Civil Code.
Alternative: Assuming that the contract of sale has been perfected, Ray may file
a counterclaim against Linda and Biong for specific performance or rescission, with
damages in either case. Linda has breached the obligation created by the contract
when she filed an action for nullification of sale. On account of Linda’s bad faith or fraud,
Ray may ask for damages under Article 1170 of the Civil Code.

Under Article 2219 of the Civil Code, moral damages may be recovered in
the cases specified therein, several of which are enumerated below.
Choose the case wherein you cannot recover moral damages. Explain.
a. A criminal offense resulting in physical injuries
a) Quasi-delicts causing physical injuries
b) Immorality or dishonesty
c) Illegal search
d) Malicious prosecution. (2006)

SUGGESTED ANSWER:
Moral damages may not be recovered in (c) immorality or dishonesty because it
is not included in the enumeration in Article 2219 of the Civil Code.

Alternative:
Moral damages may be recovered in all of the five instances enumerated above. While
―immorality‖ and ―dishonesty" are not included in the ten instances enumerated in Article
2219 of the Civil Code, the same article provides that moral damages may be recovered
―in the following and analogous cases". Article 2219(10) provides and includes: ―Acts
and actions referred to in Article 21...". Article 21 in turn provides: ―Any person who
willfully causes loss or injury to another in a manner that is contrary to morals, good
customs or public policy shall compensate the latter for damages. Immorality or
dishonesty is analogous to acts contrary to morals, and therefore covered by Article
2219.

Dr. and Mrs. Almeda are prominent citizens of the country and are frequent
travelers abroad. In 1996, they booked round-trip business class tickets for the
Manila-Hongkong-Manila route of the Pinoy Airlines, where they are holders of
Gold Mabalos Class Frequent Flier cards. On their return flight, Pinoy Airlines
upgraded their tickets to first class without their consent and, inspite of their
protestations to be allowed to remain in the business class so that they could be
with their friends, they were told that the business class was already fully
booked, and that they were given priority in upgrading because they are elite
members/holders of Gold Mabalos Class cards. Since they were embarrassed at
the discussions with the flight attendants, they were forced to take the flight at
the first class section apart from their friends who were in the business class.
Upon their return to Manila, they demanded a written apology from Pinoy Airlines.
When it went unheeded, the couple sued Pinoy Airlines for breach of contract
claiming moral and exemplary damages, as well as attorney’s fees.
Will the action prosper? Give reasons. (2005)

SUGGESTED ANSWER:
62

Yes, Pinoy Airlines breached its contract of carriage by upgrading the seat
accommodation of the Almedas without their consent. The object of their contract was
Page

CIVIL LAW REVIEW 2 – 4A BAR EXAMINATION Q & A (1980-2017)


the transportation of the Almedas from Manila to Hongkong and back to Manila, with
seats in the business class section of the aircraft. They should have been consulted first
whether they wanted to avail themselves of the privilege and would consent to a change
of seat accommodation. It should not have been imposed on them over their vehement
objection. By insisting on the upgrade, Pinoy Airlines breached its contract of carriage
with the Almedas.
However, the upgrading or the breach of contract was not attended by fraud or
bad faith. They were not induced to agree to the upgrading through insidious words or
deceitful machination or through willful concealment of material facts. Bad faith does not
simply connote bad judgment or negligence; it imports a dishonest purpose or some
moral obliquity and conscious doing of a wrong, a breach of a known duty through some
motive or interest or ill will that partakes of the nature of fraud.
Neither is Pinoy Airlines in bad faith since Section 3 of the Economic Regulation
No.7 of
the Civil Aeronautics Board provides that an overbooking that does not exceed ten
percent is not considered deliberate and therefore does not amount to bad faith.
As a result, the Almedas are not entitled to recover moral damages. Moral
damages predicated upon a breach of contract of carriage may only be recoverable in
instances where the carrier is guilty of fraud or bad faith or when the mishap resulted in
the death of a passenger. Where in breaching the contract of carriage the airline is not
shown to have acted fraudulently or in bad faith, liability for damages is limited to the
natural and probable consequences of the breach of the obligation which the parties
had foreseen or could have reasonably foreseen. In such a case the liability does not
include moral and exemplary damages.
It is a requisite in the grant of exemplary damages that the act of the offender be
accompanied by bad faith or done in wanton, fraudulent or malevolent manner. Such
requisite is absent in this case. Moreover, to be entitled thereto the claimant must first
establish his right to moral, temperate, or compensatory damages. Since the Almedas
are not entitled to any of these damages, the award for exemplary damages has no
legal basis. And where the awards for moral and exemplary damages are eliminated, so
must the award for attorney’s fees.
The most that can be awarded for the breach of contract is an award for nominal
damages. Pinoy Airlines may be said to have disturbed the spouses' wish to be with
their companions at the Business Class on their flight to Manila. (Cathay Pacific v.
Spouses Vazquez, 399 SCRA 207 [2003]).

Under the law on quasi-delict, aside from the persons who caused injury to
persons, who else are liable under the following circumstances:
1. When a 7-year-old injures his playmate while playing with his father's rifle.
Explain.
2. When a domestic helper, while haggling for a lower price with a fish vendor
in the course of buying foodstuffs for her employer’s family, slaps the fish
vendor, causing her to fall and sustain injuries. ExplainC. A carpenter in a
construction company accidentally hits the right foot of his co-worker with
a hammer. Explain
3. A 15-year-old high school student stabs his classmates who is his rival for
a girl while they were going out of the classroom after their last class.
Explain
4. What defense, if any, is available to them? (2005)

SUGGESTED ANSWER:

a) Under Article 221 of the Family Code, parents and other persons exercising
parental authority shall be civilly liable for the injuries and damages caused by the acts
or omissions of their unemancipated children or wards living in their company and under
63

their parental authority subject to the appropriate defenses provided by law.


Page

CIVIL LAW REVIEW 2 – 4A BAR EXAMINATION Q & A (1980-2017)


b) Under Article 2180, employers shall be liable for the damages caused by their
employees and household helpers acting within the scope of their assigned tasks, even
though the former are not engaged in any business or industry. As the domestic helper
was then in the exercise of her duties and acting within the scope of her assigned tasks,
her employer is also liable for the damage she has caused to the fish vendor.
Alternative: The act of ―slapping the fish vendor" is not "within the scope of the assigned
tasks" of the domestic helper. Hence, under Article 2180, the employer is not liable for
the damages caused by the domestic helper to the fish vendor.
c) His employer, the construction company, is also liable for the damages that
the carpenter caused to the latter’s co-worker. Under Article, 2176 and 2180 of the Civil
Code, liability is based on culpa aquiliana which holds the employer primarily liable for
tortious acts of its employees subject, however, to the defense that the former exercised
all the diligence of a good father of a family in the selection and supervision of his
employees. (Franco v. IAC, 178 SCRA 331 [1989]).
d) Under Section 218 of the Family Code, the school, its administrators and
teachers, or the individual, entity or institution engaged in child care shall have special
parental authority and responsibility over the minor child while under their supervision,
instruction or custody. Authority and responsibility shall apply to all authorized activities
whether inside or outside the premises of the school, entity or institution.
e) These persons identified by law to be liable may raise the defense that they
exercised proper diligence required under the circumstances. Their responsibility will
cease when they prove that they observed all the diligence of a good father of a family
to prevent damage. As regards the employer, if he shows to the satisfaction of the court
that in the selection and in the supervision of his employees he has exercised the care
and diligence of a good father of a family, the presumption is overcome and he is
relieved from liability. (Layugan v. 1AC, 167 SCRA 363 11988]).

DT and MT were prominent members of the frequent travelers’ club of FX


Airlines. In Hongkong, the couple were assigned seats in Business Class for
which they had bought tickets. On checking in, however, they were told they were
upgraded by computer to First Class for the flight to Manila because the Business
Section was overbooked. Both refused to transfer despite better seats, food,
beverage and other services in First Class. They said they had guests in
Business Class they should attend to. They felt humiliated, embarrassed and
vexed, however, when the stewardess allegedly threatened to offload them if they
did not avail of the upgrade. Thus they gave in, but during the transfer of luggage
DT suffered pain in his arm and wrist. After arrival in Manila, they demanded an
apology from FX’s management as well as indemnity payment. When none was
forthcoming, they sued the airline for a million pesos in damages.
Is the airline liable for actual and moral damages? Why or why not? Explain
briefly. (2004)

SUGGESTED ANSWER:
FX Airlines committed breach of contract when it upgraded DT and MT, over their
objections, to First Class because they had contracted for Business Class passage.
However, although there is a breach of contract, DT and MT are entitled to actual
damages only for such pecuniary losses suffered by them as a result of such breach.
There seems to be no showing that they incurred such pecuniary loss. There is no
showing that the pain in DT's arm and wrist resulted directly from the carrier's acts
complained of. Hence, they are not entitled to actual damages. Moreover, DT could
have avoided the alleged injury by requesting the airline staff to do the luggage transfer
as a matter of duty on their part. There is also no basis to award moral damages for
such breach of contract because the facts of the problem do not show bad faith or fraud
on the part of the airline. (Cathay Pacific v. Vazquez, 399 SCRA 207 [20031). However,
64

they may recover moral damages if the cause of action is based on Article 21 of the
Civil Code for the humiliation and embarrassment they felt when the stewardess
Page

threatened to offload them if they did not avail of the upgrade.

CIVIL LAW REVIEW 2 – 4A BAR EXAMINATION Q & A (1980-2017)


Alternative: If it can be proved that DT’s pain in his arm and wrist occasioned by
the transfer of luggage was caused by fault or negligence on the part of the airline’s
stewardess, actual damages may be recovered. The airline may be liable for moral
damages pursuant to Art. 2219 (10) if the cause of action is based on Article 21 or an
act contrary to morals in view of the humiliation suffered by DT and MT when they were
separated from their guests and were threatened to be offloaded.

If a pregnant woman passenger of a but were to suffer an abortion


following a vehicular accident due to the gross negligence of the bus driver, may
she and her husband claim damages from the bus company for the death of their
unborn child? Explain. (2003)

SUGGESTED ANSWER:

No, the spouses cannot recover actual damages in the form of indemnity for the
loss of life of the unborn child. This is because the unborn child is not yet considered a
person and the law allows indemnity only for loss of life of person. The mother, however
may recover damages for the bodily injury she suffered from the loss of the fetus which
is considered part of her internal organ. The parents may also recover damages for
injuries that are inflicted directly upon them, e.g., moral damages for mental anguish
that attended the loss of the unborn child. Since there is gross negligence, exemplary
damages can also be recovered.

As a result of a collision between a taxicab owned by A and another taxicab


owned by B, X, a passenger of the first taxicab, was seriously injured. X later filed
a criminal action against both drivers.
a. Is it necessary for X to reserve his right to institute a civil action for
damages against both taxicab owners before he can file a civil action for
damages against them? Why?
b. May both taxicab owners raise the defense of due diligence in the
selection and supervision of their drivers to be absolved from liability for
damages to X? Reason. (2003)

SUGGESTED ANSWER:
a. It depends. If the separate civil action is to recover damages arising from the
criminal act, reservation is necessary. If the civil action against the taxicab
owners is based on culpa contractual, or on quasi-delict, there is no need for
reservation.

Alternative:
(a) No, such reservation is not necessary. Under Section 1 of Rule 111 of the 2000
Rules on Criminal Procedure, what is ―deemed instituted‖ with the criminal action is only
the action to recover civil liability arising from the crime or ex delicto. All the other civil
actions under Articles 32, 33, 34, and 2176 of the New Civil Code are no longer
―deemed instituted‖, and may be filed separately and prosecuted independently even
without any reservation in the criminal action (Section 3, Rule 111, Ibid.), The failure to
make a reservation in the criminal action is not a waiver of the right to file a separate
and independent civil action based on these articles of the New Civil Code.
a) It depends. If the civil action is based on a quasi-delict the taxicab
owners may raise the defense of diligence of a good father of a family in the
selection and supervision of the driver; if the action against them is based on
culpa contractual or civil liability arising from a crime, they cannot raise the
defense.

OJ was employed as professional driver of MM Transit bus owned by Mr.


65

BT. In the course of his work, OJ hit a pedestrian who was seriously injured and
later died in the hospital as a result of the accident. The victim’s heirs sued the
Page

driver and the owner of the bus for damages

CIVIL LAW REVIEW 2 – 4A BAR EXAMINATION Q & A (1980-2017)


Is there a presumption in this case that Mr. BT, the owner, had been
negligent? If so, is the presumption absolute or not? Explain. (2003)

SUGGESTED ANSWER:
Yes, there is a presumption of negligence on the part of the employer. However,
such presumption is rebuttable. The liability of the employer shall cease when they
prove that they observed the diligence of a good father of a family to prevent damage.
When the employee causes damage due to his own negligence while performing
his own duties, there arises the juris tantum presumption that the employer is negligent,
rebuttable only by proof of observance of the diligence of a good father of a family.
Likewise, if the driver is charged and convicted in a criminal case for criminal
negligence, BT is subsidiarily liable for the damages arising from the criminal act.

If a pregnant woman passenger of a bus were to suffer an abortion


following a vehicular accident due to the gross negligence of the bus driver, may
she and her husband claim damages from the bus company for the death of their
unborn child? Explain. (2003)

SUGGESTED ANSWER:
No, the spouses cannot recover actual damages in the form of indemnity for the
loss of life of the unborn child. This is because the unborn child is not yet considered a
person and the law allows indemnity only for loss of life of persons. The mother,
however, may recover damages for the bodily injury she suffered from the loss of the
fetus which is considered part of her internal organs. The parents may also recover
damages for injuries that are inflicted directly upon them, e.g., moral damages for
mental anguish that attended the loss of the unborn child. Since there is gross
negligence, exemplary damages can also be recovered.

If during class hours, while the teacher was chatting with other teachers in
the school corridor, a 7-year old male pupil stabs the eye of another boy with a
ballpen during a fight, causing permanent blindness to the victim, who could be
liable for damages for the boy's injury: the teacher, the school authorities, or the
guilty boy's parents? Explain (2003)

SUGGESTED ANSWER:
The school, its administrators, and teachers have special parental authority and
responsibility over the minor child while under their supervision, instruction or custody
(Article 218, FC). They are principally and solidarity liable for the damages caused by
the acts or omissions of the unemancipated minor unless they exercised the proper
diligence required under the circumstances (Article 219, FC). In the problem, the
teacher and the school authorities are liable for the blindness of the victim, because the
student who caused it was under their special parental authority and they were
negligent. They were negligent because they were chatting in the corridor during the
class period when the stabbing incident occurred. The incident could have been
prevented had the teacher been inside the classroom at that time. The guilty boy’s
parents are subsidiarily liable under Article 219 of the Family Code.

Ortillo contracts Fabricate, Inc. to supply and install tile materials in a


building he is donating to his province. Ortillo pays 50% of the contract price as
per agreement. It is also agreed that the balance would be payable periodically
after every 10% performance until completed. After performing about 93% of the
contract, for which it has been paid an additional 40% as per agreement,
Fabricate, Inc. did not complete the project due to its sudden cessation of
operations. Instead, Fabricate, Inc. demands payment of the last 10% of the
contract despite its non-completion of the project. Ortillo refuses to pay, invoking
66

the stipulation that payment of the last amount of 10% shall be upon completion.
Fabricate, Inc. brings suit for the entire 10%, plus damages, Ortillo counters with
Page

claims for (a) moral damages for Fabricato, Inc.’s unfounded suit which has

CIVIL LAW REVIEW 2 – 4A BAR EXAMINATION Q & A (1980-2017)


damaged his reputation as a philanthropist and respected businessman in his
community, and (b) attorney’s fees.
a. Does Ortillo have a legal basis for his claim for moral damages?
b. How about his claim for attorney’s fees, having hired a
lawyer to defend him? (2002)

SUGGESTED ANSWER:
a. There is no legal basis to Ortillo's claim for moral damages. It does not fall
under the coverage of Article 2219 of the Civil Code.
b. Ortillo is entitled to attorney’s fees because Fabricato’s complaint is a case of
malicious prosecution or a clearly unfounded civil action. (Art 2208 [4] and [11], NCC).

Stockton is a stockholder of Core Corp. He desires to sell his shares in


Core Corp. in view of a court suit that Core Corp. has filed against him for
damages in the amount of P10 million, plus attorney’s fees of P1 million, as a
result of statements published by Stockton which are allegedly defamatory
because it was calculated to injure and damage the corporation’s reputation and
goodwill. The articles of incorporation of Core Corp. provide for a right of first
refusal in favor of the corporation. Accordingly, Stockton gave written notice to
the corporation of his offer to sell his shares of P10 million. The response of Core
Corp. was an acceptance of the offer in the exercise of its rights of first refusal,
offering for the purpose payment in form of compensation or set-off against the
amount of damages it is claiming against him, exclusive of the claim for
attorney's fees. Stockton rejected the offer of the corporation, arguing that
compensation between the value of the shares and the amount of damages
demanded by the corporation cannot legally take effect.
Is Stockton correct? Give reasons for your answer. (2002)

SUGGESTED ANSWER:
Stockton is correct. There is no right of compensation between his price of P10
million and Core Corp.’s unliquidated claim for damages, in order that compensation
may be proper, the two debts must be liquidated and demandable. The case for the P10
million damages being still pending in court, the corporation has as yet no claim which is
due and demandable against Stockton.
Alternative: The right of first refusal was not perfected as a right for the reason
that there was a conditional acceptance equivalent to a counter-offer consisting in the
amount of damages as being credited on the purchase price. Therefore, compensation
did not result since there was no valid right of first refusal.
Alternative: Even assuming that there was a perfected right of first refusal,
compensation did not take place because the claim is unliquidated.

A van owned by Orlando and driven by Diego, while negotiating a downhill


slope of a city road, suddenly gained speed, obviously beyond the authorized
limit in the area, and bumped a car in front of it, causing severe damage to the car
and serious injuries to its passengers. Orlando was not in the car at the time of
the incident. The car owner and the injured passengers sued Orlando and Diego
for damages caused by Diego's negligence, in their defense, Diego claims that
the downhill slope caused the van to gain speed and that, as he stepped on the
brakes to check the acceleration, the brakes locked, causing the van to go even
faster and eventually to hit the car in front of it. Orlando and Diego contend that
the sudden malfunction of the van’s brake system is a fortuitous event and that,
therefore, they are exempt from any liability.
Is this contention tenable? Explain.
Does the presence of the owner inside the vehicle causing damage to a
67

third party affect his liability for his driver’s negligence? Explain.
Explain the concept of vicarious liability in quasi-delicts. (2002)
Page

CIVIL LAW REVIEW 2 – 4A BAR EXAMINATION Q & A (1980-2017)


SUGGESTED ANSWER:
No. Mechanical defects of a motor vehicle do not constitute fortuitous event,
since the presence of such defects would have been readily detected by diligent
maintenance check. The failure to maintain the vehicle in safe running condition
constitutes negligence.
In motor vehicle mishaps, the owner is made solidarily liable with his driver if he
(the owner) was in the vehicle and could have, by the use of due diligence, prevented
the mishap. (Caedo v. Yu Khe Thai, 26 SCRA 410 [1968]). However, this question has
no factual basis in the problem given, in view of the express given fact that ―Orlando
was not in the car at the time of the incident‖
The doctrine of vicarious liability is that which renders a person liable for the
negligence of others for whose acts or omission the law makes him responsible on the
theory that they are under his control and supervision.

After working overtime up to midnight, Alberto an executive of an


insurance company drove a company vehicle to a favorite Videoke bar where he
had some drinks and sang some songs with friends to "unwind". At 2:00 a.m., he
drove home, but in doing so, he bumped a tricycle, resulting in the death of its
driver.
May the insurance company be held liable for the negligent act of Alberto?
Why? (2001)

SUGGESTED ANSWER:
The insurance company is not liable because when the accident occurred,
Alberto was not acting within the assigned tasks of his employment. It is true that under
Art. 2180 (par. 5), employers are liable for damages caused by their employees who
were acting within the scope of their assigned tasks. However, the mere fact that
Alberto was using a service vehicle of the employer at the time of the injurious accident
does not necessarily mean that he was operating the vehicle within the scope of his
employment. In Castilex Industrial Corp. v. Vasquez, Jr. (321 SCRA393 [1999]), the
Supreme Court held that notwithstanding the fact that the employee did some overtime
work for the company, the former was, nevertheless, engaged in his own affairs or
carrying out a personal purpose when he went to a restaurant at 2:00 a.m. after coming
out from work. The time of the accident (also 2:00 a. m.) was outside normal working
hours.
Alternative: The insurance company is liable if Alberto was negligent in the
operation of the car and the car was assigned to him for the benefit of the insurance
company, and even though he was not within the scope of his assigned tasks when the
accident happened. In one case decided by the Supreme Court, where an executive of
a pharmaceutical company was given the use of a company car, and after office hours,
the executive made personal use of the car and met an accident, the employer was also
made liable under Art. 2180 of the Civil Code for the injury caused by the negligent
operation of the car by the executive, on the ground that the car which caused the injury
was assigned to the executive by the employer for the prestige of the company. The
insurance company was held liable even though the employee was not performing
within the scope of his assigned tasks when the accident happened.

Silvestre leased a car from Avis-Rent-A-Car Co. at the Mactan International


Airport. No sooner had he driven the car outside the airport when, due to his
negligence, he bumped an FX taxi owned and driven by Victor, causing damage
to the latter in the amount of P 100,000.00. Victor filed an action for damages
against both Silvestre and Avis, based on quasi-delict. Avis filed a motion to
dismiss the complaint against it on the ground of failure to state a cause of
action.
68

Resolve the motion. (2000)


Page

CIVIL LAW REVIEW 2 – 4A BAR EXAMINATION Q & A (1980-2017)


SUGGESTED ANSWER:
The motion to dismiss should be granted. AVIS is not the employer of Silvestre;
hence, there is no right of action against AVIS under Article 2180 of the Civil Code. Not
being the employer, AVIS has no duty to supervise Silvestre. Neither has AVIS the duty
to observe due diligence in the selection of its customers. Besides, it was given in the
problem that the cause of the accident was the negligence of Silvestre.
Alternative: The motion should be denied. Under the Public Service Law, the
registered owner of a public utility is liable for the damages suffered by third persons
through the use of such public utility. Hence, the cause of action is based in law, the
Public Service Law.

Despite a warning from the police that an attempt to hijack a PAL plane will
be made in the following week, the airline did not take extra precautions, such as
frisking of passengers, for fear of being accused of violating human rights. Two
days later, an armed hijacker did attempt to hijack a PAL flight to Cebu. Although
he was subdued by the other passengers, he managed to fire a shot which hit and
killed a female passenger. The victim’s parents sued the airline for breach of
contract, and the airline raised the defense of force majeure.
Is the airline liable or not? (2000)

SUGGESTED ANSWER:
The airline is liable. In case of death of a passenger, common carriers are
presumed to have been at fault or to have acted negligently, unless they prove that they
observed extraordinary diligence. The failure of the airline to take extra precautions
despite a police warning that an attempt to hijack the plane would be made, was
negligence on the part of the airline. Being negligent, it is liable for the death of the
passenger. The defense of force majeure is not tenable since the shooting incident
would not have happened had the airline taken steps that could have prevented the
hijacker from boarding the plane.
Alternative: Under Article 1763 of the Civil Code, the common carrier is not
required to observe extraordinary diligence in preventing injury to its passengers on
account of the wilful acts or negligence of other passengers or of strangers. The
common carrier, in that case, is required to exercise only the diligence of a good father
of a family; hence, the failure of the airline to take EXTRA precautions in filsking the
passengers and by leaving that matter to the security personnel of the airport, does not
constitute a breach of that duty so as to make the airline liable. Besides, the use of
irresistible force by the hijackers
was force majeure that could not have been prevented even by the observance of
extraordinary diligence.

A Galant driven by John and owned by Art, and a Corolla driven by its
owner, Gina, collided somewhere along Adriatico Street. As a result of the
accident, Gina had a concussion. Subsequently, Gina brought an action for
damages against John and Art. There is no doubt that the collision is due to
John’s negligence.
Can Art, who was in the vehicle at the time of the accident, be held
solidarlly liable with his driver, John? (1998)

SUGGESTED ANSWER:
Yes. Art may be held solidarily liable with John, if it was proven that the former
could have prevented the misfortune with the use of due diligence. Article 2184 of the
Civil Code states: ―In motor mishaps, the owner is solidarily liable with his driver, if the
former, who was in the vehicle, could have, by the use of due diligence, prevented the
misfortune, x x x"
69

Alternative: It depends. The Supreme Court In Chapman vs. Underwood (27 Phil
374), held: ―An owner who sits in his automobile, or other vehicle, and permits his driver
Page

to continue in a violation of law by the performance of negligent acts, after he has had a

CIVIL LAW REVIEW 2 – 4A BAR EXAMINATION Q & A (1980-2017)


reasonable opportunity to observe them and to direct that the driver cease therefrom,
becomes himself responsible for such acts, x x x On the other hand, if the driver, by a
sudden act of negligence, and without the owner having a reasonable opportunity to
prevent the act or its continuance, injures a person or violates the criminal law, the
owner of the automobile, although present therein at the time the act was committed is
not responsible, either civilly or criminally, therefor. The act complained of must be
continued in the presence of the owner for such a length of time that the owner, by his
acquiescence, makes his driver’s act his own."

A. When would an employer's liability for damage, caused by an employee


in the performance of his assigned tasks, be primary and when would it be
subsidiary in nature?
B. Would the defense of due diligence in the selection and supervision of
the employee be available to the employer in both instances? (1997)

SUGGESTED ANSWER:

a) The employer's liability for damage based on culpa aquiliana under


Art. 2176 and 2180 of the Civil Code is primary, while that under Art. 103 of the
Revised Penal Code is subsidiary.
b) The defense of diligence in the selection and supervision of the
employee under Article 2180 of the Civil Code is available only to those primarily
liable thereunder, but not to those subsidiarily liable under Article 103 of the
Revised Penal Code.

Rosa was leasing an apartment in the city. Because of the Rent Control
Law, her landlord could not increase the rental as much as he wanted to, nor
terminate her lease as long as she was paying her rent. In order to force her to
leave the premises, the landlord stopped making repairs on the apartment, and
caused the water and electricity services to be disconnected. The difficulty of
living without electricity and running water resulted in Rosa's suffering a nervous
breakdown. She sued the landlord for actual and moral damages.
Will the action prosper? Explain. (1996)

SUGGESTED ANSWER:
Yes, based on quasi-delict under the human relations provisions of the New Civil
Code (Articles 19, 20 and 21) because the act committed by the lessor is contrary to
morals. Moral damages are recoverable under Article 2219 (10) in relation to Article 21.
Although the action is based on quasi-delict and not on contract, actual damages may
be recovered if the lessee is able to prove the losses and expenses she suffered.

On 8 December 1991 Vanessa purchased from the Manila office of Euro-


Aire an airline ticket for its Flight No. 710 from Dallas to Chicago on 16 January
1992. Her flight reservation was confirmed. On her scheduled departure Vanessa
checked in on time at the Dallas airport. However, at the check-in counter she
discovered that she was waitlisted with some other passengers because of
intentional overbooking, a Euro-Aire policy and practice. Euro-Aire admitted that
Vanessa was not advised of such policy when she purchased her plane ticket.
Vanessa was only able to fly two days later by taking another airline. Vanessa
sued Euro-Aire in Manila for breach of contract and damages. Euro-Aire claimed
that it cannot be held liable for damages because its practice of overbooking
passengers was allowed by the U.S. Code of Federal Regulations. Vanessa on the
other hand contended that assuming that the U.S. Code of Federal Regulations
allowed intentional overbooking, the airline company cannot invoke the U.S.
70

Code on the ground that the ticket was purchased in Manila, hence, Philippine
law should apply, under which Vanessa can recover damages for breach of
Page

contract of carriage.

CIVIL LAW REVIEW 2 – 4A BAR EXAMINATION Q & A (1980-2017)


Decide. Discuss fully. (1995)

SUGGESTED ANSWER:
Vanessa can recover damages under Philippine law for breach of contract of
carriage.
Philippine law should govern as the law of the place where the plane tickets were
bought and the contract of carriage was executed. In Zalamea v. Court of Appeals (G.R.
No. 104235. Nov. 10. 1993) the Supreme Court applied Philippine law in recovery of
damages for breach of contract of carriage for the reason that it is the law of the place
where the contract was executed.
If the violation of the contract was attended with bad faith, there is a ground to
recover
moral damages. But since there was a federal regulation which was the basis of the act
complained of, the airline cannot be in bad faith. Hence, only actual damages can be
recovered. The same is true with regards to exemplary damages.

On January 5, 1992, Nonoy obtained a loan of Pl,000,000.00 from his friend


Rafly. The promissory note did not stipulate any payment for Interest. The note
was due on January 5, 1993 but before this date the two became political
enemies. Nonoy, out of spite, deliberately defaulted in paying the note, thus
forcing Raffy to sue him.
What actual damages can Rafly recover?
Can Rafly ask for moral damages from Nonoy?
Can Rafly ask for nominal damages?
Can Raffy ask for temperate damages?
Can Rafly ask for attorney's fees? (1994)

SUGGESTED ANSWER:
Raffy may recover the amount of the promissory note of P1 million, together with
interest at the legal rate from the date of judicial or extrajudicial demand. In addition,
however, inasmuch as the debtor is in bad faith, he is liable for all damages which may
be reasonably attributed to the non-performance of the obligation. (Art. 2201(2). NCC).
Yes, under Article 2220, NCC moral damages are recoverable in case of breach
of contract where the defendant acted fraudulently or in bad faith.
Nominal damages may not be recoverable in this case because Rafly may
already be indemnified of his losses with the award of actual and compensatory
damages. Nominal damages are adjudicated only in order that a right of the plaintiff,
which has been violated or invaded by the defendant may be vindicated or recognized,
and not for the purpose of indemnifying the plaintiff for any loss suffered by him. (Article
2231. Civil Code)
Raffy may ask for, but would most likely not be awarded temperate damages, for
the reason that his actual damages may already be compensated upon proof thereof
with the promissory note. Temperate damages may be awarded only when the court
finds that some pecuniary loss has been suffered but its amount cannot, from the nature
of the case, be proved with certainty. (Article 2224, Civil Code)
Yes. under paragraph 2, Article 2208 of the Civil Code, considering that Nonoy's
act or omission has compelled Raffy to litigate to protect his interests. Furthermore.
attorneys' fees may be awarded by the court when it is just and equitable. (Article
2208(110) Civil Code).

Johnny Maton’s conviction for homicide was affirmed by the Court of


Appeals and, in addition, although the prosecution had not appealed at all, the
appellate court increased the indemnity for death from P30.000.00 to P50.000.00.
On his appeal to the Supreme Court, among the other things Johnny Maton
brought to the high court’s attention, was the increase of indemnity imposed by
71

the Court of Appeals despite the clear fact that the People had not appealed from
Page

the appellate court’s judgment.

CIVIL LAW REVIEW 2 – 4A BAR EXAMINATION Q & A (1980-2017)


Is Johnny Maton correct? (1994)

SUGGESTED ANSWER:
In Abejam v. Court of Appeals, the Supreme Court said that even if the issue of
damages were not raised by the appellant in the Court of Appeals but the Court of
Appeals in its findings increased the damages, the Supreme Court will not disturb the
findings of the Court of Appeals.
Alternative: No, the contention of the accused is not correct because upon
appeal to the Appellate Court, the court acquired jurisdiction over the entire case,
criminal as well as civil. Since the conviction of homicide had been appealed, there is no
finality in the amount of indemnity because the civil liability arising from the crime and
the judgment on the crime has not yet become final.
Alternative: Yes. Since the civil indemnity is an award in the civil action arising
from the criminal offense, the rule that a party cannot be granted affirmative relief unless
he himself has appealed should apply. Therefore, it was error for the Court of Appeals
to have expanded the indemnity since the judgment on the civil liability had become
final.
Alternative: No. Courts can review matters not assigned as errors. (Hydro
Resource vs. CA, 204 SCRA 309).

Julio and Lea, both 18 years old, were sweethearts. At a party at the house
of a mutual friend. Lea met Jake, also 18 years old, who showed interest in her.
Lea seemed to entertain Jake because she danced with him many times. In a fit of
jealousy, Julio shot Jake with his father’s 38 caliber revolver which, before going
to the party he was able to get from the unlocked drawer inside his father’s
bedroom. Jake died as a result of the lone gunshot wound he sustained. His
parents sued Julio’s parents for damages arising from quasi-delict. At the time of
the incident, Julio was 18 years old living with his parents. Julio’s parents moved
to dismiss the complaint against them claiming that since Julio was already of
majority age, they were no longer liable for his acts.
Should the motion to dismiss be granted? Why?
What is the liability of Julio’s parents to Jake’s parents? Explain your
answer. (1993)

SUGGESTED ANSWER:
No, the Motion to Dismiss should not be granted. Article 236 of the Family Code
as amended by Republic Act 6809, provides in the third paragraph that ―nothing in this
Code shall be construed to derogate from the duty or responsibility of parents and
guardians for children and wards below twenty-one years or age mentioned in the
second and third paragraphs of Article 2180 of the Civil Code.
The liability of Julio’s parents to Jake’s parents arises from quasi-delict (Arts.
2176 and 2180 Civil Code) and shall cover specifically the following: a. P50.000.00 for
the death of the son; b. such amount as would correspond to lost earning capacity; and
c. moral damages.

As the result of a collision between a public service passenger bus and a


cargo truck owned by D. X sustained physical injuries and Y died. Both X and Y
were passengers of the bus. Both drivers were at fault, and so X and Z, the only
heir and legitimate child of the deceased Y, sued the owners of both vehicles.
May the owner of the bus raise the defense of having exercised the
diligence of a good father of a family?
May D raise the same defense?
May X claim moral damages from both defendants?
May Z claim moral damages from both defendants? Give reasons for all
your answers.(1992)
72
Page

CIVIL LAW REVIEW 2 – 4A BAR EXAMINATION Q & A (1980-2017)


SUGGESTED ANSWER:

No. The owner of the bus cannot raise the defense because the carrier's liability is
based on breach of contract.

Yes. D can raise the defense because his liability is based on a quasi-delict. Because X
suffered physical injuries, X can claim moral damages against D. But as against the
owner of the bus, X can claim moral damages only if X proves reckless negligence of
the carrier amounting to fraud.

Z can claim moral damages against both defendants because the rules on damages
arising from death due to a quasi-delict are also applicable to death of a passenger
caused by breach of contract by a common carrier (Arts. 1755. 1756, 1764, 2206 and
2219. Civil Code).

Romano was bumped by a minivan owned by the Solomon School of


Practical Arts (SSPA). The minivan was driven by Peter, a student assistant
whose assignment was to clean the school passageways daily one hour before
and one hour after regular classes, in exchange for free tuition. Peter was able to
drive the school vehicle after persuading the regular driver, Paul, to turn over the
wheel to him (Peter). Romano suffered serious physical injuries. The accident
happened at night when only one headlight of the vehicle was functioning and
Peter only had a student driver’s permit.
As a consequence, Peter was convicted in the criminal case. Thereafter.
Romarjo sued for damages against Peter and SSPA.
A. Will the action for damages against Peter and SSPA prosper?
B. Will your answer be the same if, Paul, the regular driver, was impleaded
as party defendant for allowing Peter to drive the minivan without a regular
driver's license.
C. Is the exercise of due diligence in the selection and supervision of Peter
and Paul a material issue to be resolved in this case? (1991)

SUGGESTED ANSWER:
A. Yes. It will prosper (Art. 2180) because at the time he drove the vehicle, he
was not performing his assigned tasks as provided for by Art. 2180. With respect to
SSPA, it is not liable for the acts of Peter because the latter was not an employee as
held by Supreme Court in Filamer Christian Institute vs. CA, (190 SCRA 485).
Peter belongs to a special category of students who render service to the school
in exchange for free tuition fees.
B. I would maintain the same answer because the incident did not occur while
the employee was in the performance of his duty as such employee. The incident
occurred performing his duties as a driver.
C. In the case of Peter, if he were to be considered as employee, the exercise of
due diligence in the selection and supervision of peter would not be a material issue
since the conviction of Peter would result in a subsidiary liability where the defense
would not be available by the employer.
In the case of Paul, since the basis of subsidiary liability is the paterfamilias rule
under Art. 2180, the defense of selection and supervision of the employee would be a
valid defense.
Alternative: A. In the case of Peter, if he were to be considered an employee, the
exercise of due diligence in the selection and supervision of Peter would not be a
material issue since the conviction of Peter would result in a subsidiary liability where
the defense would not be available by the employer.
In the case of Paul, since he was in the performance of his work at the time the
incident occurred, the school may be held subsidiarily liable not because of the
73

conviction of Peter, but because of the negligence of Paul under Art. 2180.
Page

CIVIL LAW REVIEW 2 – 4A BAR EXAMINATION Q & A (1980-2017)


Mr. and Mrs. R own a burned-out building, the firewall of which collapsed
and destroyed the shop occupied by the family of Mr. and Mrs. S, which resulted
in injuries to said couple and the death of their daughter. Mr. and Mrs. S had been
warned by Mr. Mrs. R to vacate the shop in view of its proximity to the weakened
wall but the former failed to do so. Mr. and Mrs. S filed against Mr. and Mrs. R an
action for recovery of damages the former suffered as a result of the collapse of
the firewall. In defense, Mr. and Mrs. R rely on the doctrine of ―last clear chance"
alleging that Mr. and Mrs. S had the last clear chance to avoid the accident if only
they heeded the former’s warning to vacate the shop, and therefore Mr. and Mrs.
R’s prior negligence should be disregarded.
If you were the Judge, how would you decide the case? State your reasons.
(1990)

SUGGESTED ANSWER:
I would decide in favor of Mr. and Mrs. S. The proprietor of a building or structure
is responsible for the damages resulting from its total or partial collapse, if it should be
due to the lack of necessary repairs. (Article 2190, Civil Code).
As regards the defense of Mr. and Mrs. R relying on the doctrine of ―last clear
chance.‖ the same is not tenable because according to the Supreme Court in one case
(DeRoy v. Court of Appeals. G.R. L-80718. January 29, 1988, 157 SCRA 757) the
doctrine of ―last clear chance‖ is not applicable to instances covered by Art. 2190, Civil
Code.
Further, in Phoenix Construction, Inc. v. Intermediate Appellate Court (G.R. L-
65295, March 10, 1987. 148 SCRA 353). the Supreme Court held that the role of the
common law "last clear chance‖ doctrine in relation to Article 2179 of the Civil Code is
merely to mitigate damages within the context of contributory negligence.

What are the requisites in order that the defendant can be held liable for
damages in a quasi-delict case? (1988)

SUGGESTED ANSWER:
In actions based on quasi-delicts, before the person injured can recover
damages from the defendant, it is necessary that he must be able to prove the following
facts:
a) The fault or negligence of the defendant;
b) The damages suffered or incurred by the plaintiff; and
c) The relation of cause and effect between the fault or negligence of the defendant
and the damage incurred by the plaintiff.

Ato was the registered owner of a passenger jeepney, which was involved
in a collision accident with a vegetable truck, resulting in the death of four
passengers and injuries to three. At the time of the accident, Ato was legally
married to Maria but was cohabiting with Tonia in a relationship akin to that of
husband and wife.
Could the heirs of the dead passengers and the injured persons recover
damages from:
a) Ato?
b) Maria?
c) Tonia?
Explain each case. (1987)

SUGGESTED ANSWER:
A. Ato — Yes. Insofar as the dead passengers are concerned, the heirs can
recover damages on the basis of culpa contractual. If the injured persons are also
passengers, Ato is likewise liable on the same basis of culpa contractual.
74

However, if the injured persons are not passengers, then the liability for damages
of Ato will be on the basis of a quasi-delict.
Page

CIVIL LAW REVIEW 2 – 4A BAR EXAMINATION Q & A (1980-2017)


B. Maria — In view of the ruling in Juaniza v. Jose (89 SCRA 306) that the
passenger jeepney acquired by the husband during an illicit cohabitation with the
paramour is conjugal property, Maria is liable to the same extent as Ato insofar as the
conjugal property in the marriage between Ato and Maria could be answerable. But as
regards her paraphernal property, Maria cannot be held answerable.
C. Tonia — No. In Juaniza v. Jose the paramour of the owner of the passenger
jeepney that figured in an accident was held to be not a co-owner, and therefore not
liable for damages. Article 144 is inapplicable.

The X Electric Cooperative services a small town where the roads are lined
with lush acacia trees- Normally these trees are pruned before the onset of the
rainy season by the cooperative itself since the power lines of the cooperative are
not infrequently affected by falling branches. This year, for financial reasons, the
electric cooperative omitted the pruning in spite of reminders from the
townspeople. In August this year a strong typhoon hit the town and live wires fell
to the ground. While the cooperative made a preliminary survey of the damages,
it did not immediately take precautionary measures against possible harm. Thus,
the attention of one of its employees was called to the fallen wire in the center of
the town. Before the cooperative could make the necessary repairs, a four-year
old boy crossed the street and was electrocuted by the live wire. His parents sued
the electric cooperative for damages.
If you were counsel for the parents, what arguments would you advance to
support your claim for damages and how much damages would you demand?
If you were counsel for the electric cooperative, what defenses
would you offer? If you were judge, how would you decide the
case? (1987)

SUGGESTED ANSWER:
The damages that can be claimed by the parents are the following: civil
indemnity for death - P30,000.00; actual and compensatory damages; moral damages
for mental anguish; exemplary or corrective damages.
As counsel for the electric cooperative, I would offer the defense of "fortuitous
event," because the strong typhoon could not be foreseen and even if foreseen, could
not be avoided.
As judge, I would rule for the parents. The attention of the cooperative through its
employee was called to the fallen live wire. If there had been care and diligence, the
death could have been avoided. The cooperative could have made the necessary
repairs before the 4 year old boy crossed the street and was electrocuted by the live
wire. It failed to do so, hence it is liable.

A entered into a twenty- year lease contract with 6 for the use of B's
warehouse in connection with his (A's) business. After ten years, A's business
had so prospered that he needed to move to a larger place. Upon learning of As
intention to transfer his business elsewhere, B offered to terminate the lease
contract, as the rental rate for the warehouse had by then tripled the stipulated
rental. A, whose relationship with B had soured over the years, refused the offer
so as to prevent B from leasing the premises to another party. A padlocked the
warehouse after he had transferred his business to another place, although he
continued paying B the stipulated rental.
Under the circumstances, does B have any cause of action against A?
Explain. (1984)

SUGGESTED ANSWER:
B can file an action for damages and other relief, including a declaration of the
termination of the lease contract, against B for abuse of rights under Article 19, which
75

provides that every person must, in the exercise of his rights and in the performance of
his duties, act with justice, give everyone his due, and observe honesty and good faith.
Page

CIVIL LAW REVIEW 2 – 4A BAR EXAMINATION Q & A (1980-2017)


Alternative: B can file an action against A for damages, including a declaration of
the termination of the lease contract. The act of A is not only willful but also contrary to
morals, good customs and public policy.
Alternative: B can file an action against A for ejectment. A's act constitutes a
violation of one of the essential conditions of the contract -of lease to use the thing
leased for the purpose intended.

"KK" sued "LL" for damages for breach of contract. At the trial "KK"
proved the breach of contract while "LL" proved that he acted in good faith.
Give the law governing the damages which "KK" is entitled to recover.
If "KK" proved that "LL" acted in bad faith, give the law governing the
damages which "KK" is entitled to recover. (1980)

SUGGESTED ANSWER:
If "LL" acted in good faith, "KK" is entitled to recover from him as compensatory
damages those consequences which are natural, probable and which the parties had
foreseen at the time when the obligation was constituted.
However, if "LL" acted in bad faith, "KK" is entitled to recover from him as
compensatory damages those consequences which may be reasonably attributed to the
non -performance of the obligation. Additionally, he may even be entitled to recover
moral damages, exemplary damages and attorney's fees.
76
Page

CIVIL LAW REVIEW 2 – 4A BAR EXAMINATION Q & A (1980-2017)

Das könnte Ihnen auch gefallen